PDA

Archiv verlassen und diese Seite im Standarddesign anzeigen : Licht: Welle oder Teilchen?


Seiten : [1] 2

ingeniosus
17.04.08, 16:20
Da Lichtmodell schon vergeben ist, will ich diesen Neuansatz wagen.

Jünste Beiträge sind leider nur emotionell geworden.

Ich ersuche alle Teilnehmer rein wissenschaftlich zu argumentieren und keine Emotionen aufkommen zu lassen.

ingeniosus
17.04.08, 16:43
Kurz meine jüngsten Ideen dazu, jede wissenschaftliche Diskussion - ohne persönliche Emotionen (=Entgleisungen) - ist hier willkommen.
Für die Welle: Frequenz, Beugung,
Für den Körper: Photonen-Quantelung nach Planck, Impuls, Ausbreitungsgeschwindigkeit
Verbindungen: Ausbreitung als mögliche Interferrenzwellenfront
Fragen: woher kommt die Frequenz?
woher kommt die konstante Lichtgeschwindigkeit? Einstein sagt: ist eben eine Naturkonstante, es ändert sich die Masse bzw bei Licht die Frequenz?
Warum schwingt Licht?
Setzt das nicht ein vorhandenes Verteilungsfeld voraus, das wir noch nicht kennen (Verschränkung nach Zeillinger)?
Teilchen-Hierarchie: Himmelskörper, Gegenstand, Molekül, Atom, Elektron, Photon, Atomkern: Quarks
Theoretische Elemente von Beziehungen: "Eichbosonen" (Photonen, Gluonen, W- und Z-Bosonen)
Eigenes Masse-Teilchen: Higgs-Boson
Ist die makroskopische Physik doch auch in die Quantenmechanik überführbar?
Scheitern wir in der Quantenmechanik nur an der Unfähigkeit, einflusslose Messungen durchzuführen? Wie wären solche zu erreichen?

sapienti sat

Querkopf
17.04.08, 21:14
Was ist Licht?
Klassisch gesehen eine Lösung der Maxwellgleichungen, genauer der sogenannten Wellengleichung, einer linearen Differentialgleichung zweiter Ordnung in Räumlichen und Zeitlichen Variablen, also eine Welle.
Gott sprach es werde Maxwell und es ward Licht. Nun zumindest für den Ingenieur.
Für den theoretischen Physiker (zumindest Feldtheoretiker) steht die Lagrangedichte am Anfang aller Naturbetrachtung. Aus dieser lassen sich dann die Feldgleichungen ableiten.
Soweit die klassische Theorie. Teilchen gibt es in ihr keine. Daran ändern auch ad hoc Hypothesen von Planck oder Einstein nichts. Diese zeigen nur, dass die Maxwelltheorie unvollständig ist. Ich brauche eine Feldgleichung, die mir explizit Teilchenlösungen gibt, denn das ist z.B. das Ergebnis des Photoelektrischen Effektes. Ich brauche Energiepakete von h-quer Omega.
Wie bekomme ich so etwas (es gibt natürlich viele Wege zum Ziel)? Mein Ausgangspunkt ist die Lagrangedichte (also die Feldvariante dessen was du als Lagrangefunktion vermutlich in der Ingenieursausbildung gelernt hast) des freien Feldes (also keine Ströme und Ladungen). Durch Legendretransformation erhalte ich meine Hamiltondichte.
Meine Bewegungsgleichung ist eine lineare Differentialgleichung, ich kann einen Ebenen Welleansatz machen (bzw. eine Darstellung des Feldes durch Fouriermoden). Ich habe also im Grunde genommen eine Summation über jede menge harmonische Oszillatoren mit verschienen Frequenzen.

Der Harmonische Oszillator ist ein wohlbekanntes Problem, für jeden, der in seinem Leben einmal mit Differentialgleichungen zu tun bekommen hat. Den quantenmechanischen Oszillator hingegen kann ich zwar auch als Differentialgleichung lösen, viel effektiver ist aber die algebraische Variante des Problems. Dabei werden Anregungszustände des Oszillators durch auf und Absteigeoperatoren ineinander überführt. Jeder Anregungszustand lässt sich durch mehrfaches Anwenden eines Aufsteigeoperators aus dem Grundzustand aus dem Grundzustand erzeugen. Durch Rekursion bekomme ich die Polynome, die auch meine Differentialgleichung lösen.
Das möchte ich aber eigentlich gar nicht mehr. Interessanter Weise hat mein QM harmonischer Oszillator ein äquidistantes Energiespektrum mit den Energieschritten von h quer Omega. Das ist genau das diskrete Energiepaket, das Lichtteilchen nachdem ich suche. Meine Operatoren erzeugen also Photonen aus dem Grundzustand bzw. vernichten diese wieder.

Ich ersetze nun also meine ganzen Fouriermoden durch Quantenmechanische Oszillatoren die ihrerseits aus Operatoren bestehen, die jeweils Photonen der Energie h-quer omega erzeugen, bzw. vernichten.

Ich habe nun ein Quantisiertes Elektromagentisches Feld. Im klassischen Grenzübergang habe ich ein kontinuierliches Spektrum, dass ich wieder in meine Fouriermoden und letzten Endes in Maxwell überführen kann. Makroskopisch macht habe ich meine gute alte Maxwelltheorie und meine bekannten elektromagnetischen Wellen.
Makroskopisch habe ich also Wellen, aber meine Theorie ist nun nicht mehr fundamental. Was ist nun aber Licht auf fundamentaler Ebene.

Ich bin gestartet mit einem klassischen Feld, also einer Funktion (Orts und zeitvariablen, bzw. Ortsableitungen). Nun habe ich aber meine Erzeugungs - und Vernichtungsoperatoren eingefügt. Mein Feld ist nun keine Funktion mehr, sondern ein Feldoperator, der das Entstehen von Photonen in Raum und Zeit beschreibt.
Es gibt also in dieser Theorie nur noch Teilchen. Das sind aber keine klassischen Teilchen mehr, also irgendwelche kleine Massekügelchen oder ähnliches, sondern Anregungszustände des Grundzustandes meines Systems (auch Vakuum genannt).
Noch einmal zur Gegenüberstellung der gegensätzlichen Konzepte:

Klassische Theorie: Ich habe Felder und geladene Teilchen. Geladenen Teilchen Wechselwirken über Felder.
QFT: Felder sind Operatoren die Teilchen aus dem Vakuum erzeugen und in dieses vernichten. Teilchen sind Anregungszustände des Vakuums. Die Teilchenzahl muss nicht erhalten bleiben!

Bis jetzt habe ich in meiner Theorie noch keine Wechselwirkung. Ich habe nur ein freies Maxwellfeld. Meine Theorie hat keinen Selbstwechselwirkungsterm. Für Wechselwirkungen brauche ich Ströme und Ladungen. Diese ladungsträger (Elektronen, Positronen, ...) werden durch das Diracfeld, ein anderes Quantisiertes Feld beschrieben.

Meine Wechselwirkung findet auch über meine Operatoren statt, wobei der Viererimpuls erhalten bleiben muss (also aus dem Nichts kann ich auch nichts erzeugen). Ein typischer Wechselwirkungsprozess wäre also: Ein Photon wird erzeugt und dafür ein Elektron - Positron paar vernichtet.
Ich kann mein Quantisiertes elektromagnetisches Feld auch mit einem Elektron z.B. in einem Wasserstoffatom koppeln. Dann erzeuge ich ein Elektron höherer Energie und vernichte dafür ein Photon und ein Elektron niedrigerer Energie. Ich weiß plötzlich wo meine Photonen stecken wenn ich ein Wasserstoffatom anrege. Ich habe nämlich plötzlich einem Mechanismus der mir eine Verletzung der Teilchenzahlerhaltung ausdrücklich erlaubt. Klassisch kann ich Teilchen nicht vernichten und schon gar nicht erzeugen (da kommt die Kontinuitätsgleichung her).

Für mich gibt es daher wie z.B. auch für QED Mitschöpfer Richard Feynman nur Teilchen.

All die ganzen Schönen Theorien enthalten aber immer die Konstanz der Lichtgeschwindigkeit. Warum? Weil es funktioniert. Weil es die natur nun einmal beschreibt. Aber warum ist sie konstant? Keiner weiß es. Vielleicht sagt uns eine spätere Theorie etwas darüber. Vielleicht werden wir es nie wissen.

Scheitern wir in der Quantenmechanik nur an der Unfähigkeit, einflusslose Messungen durchzuführen? Wie wären solche zu erreichen?
Das ist natürlich eine Interessante Überlegung, mit denen sich die Väter der QM auch herumgeschlagen haben. Sind QM Effekte fundamental, ist z.B. die Heisenbergsche Unschärfe Fundamental, oder lassen wir uns von unserer Unfähigkeit im Mikroskopischen Bereich vernünftige Experimente zu machen hinters Licht Führen.

Dazu kann man aber mittlerweile sagen: Die Quantenmechanik ist Fundamental. Es gibt eine ganze Reihe von Experimenten bei denen der die maximale Störung durch die Messung um ein vielfaches geringer ist, als der Quantenmechanische Effekt. Man ist mittlerweile auf einem technischen niveau, bei dem man Verschränkung an Makroskopischen Objekten beobachten kann. Es gibt natürlich die berühmten Zeilingerexperimente , aber die Kohlenstofffußbälle sind immer noch ziemlich klein. Weniger gut vermarktet aber eigentlich viel Spektakulärer sind Versuche zur Verschränkung von strömen in Supraleiterelementen. Da kann man durchaus Ströme im Mikroamper Bereich (also das was man in der Schule z.T an Strömen hat) miteinander verschränken. Das ist recht interessant für den Bau von Festköperquantencomputern weil man eben nicht so ein kleines schlecht zu kontrollierendes Atom hat sondern einen Makroskopische Strom.

quick
18.04.08, 04:46
Hallo Querkopf,

Interessanter Weise hat mein QM harmonischer Oszillator ein äquidistantes Energiespektrum mit den Energieschritten von h quer Omega. Das ist genau das diskrete Energiepaket, das Lichtteilchen nachdem ich suche. Meine Operatoren erzeugen also Photonen aus dem Grundzustand bzw. vernichten diese wieder.
Wenn Du virtuelle Photonen entstehen läßt, was genau oszilliert dabei?
Oszillieren die Photonen selbst dann auch noch?

All die ganzen Schönen Theorien enthalten aber immer die Konstanz der Lichtgeschwindigkeit. Warum? Weil es funktioniert. Weil es die natur nun einmal beschreibt. Aber warum ist sie konstant? Keiner weiß es. Vielleicht sagt uns eine spätere Theorie etwas darüber. Vielleicht werden wir es nie wissen.
Die Konstanz der Geschwindigkeit finde ich gar nicht so besonders, mich würde eher interessieren, welcher "Mechanismus" hinter dieser hohen Geschwindigkeit steckt. Was hält ein Photon zusammen und auf der "Spur"?

mfg
quick

Uranor
18.04.08, 06:26
Hallo quick,
Die Konstanz der Geschwindigkeit finde ich gar nicht so besonders, mich würde eher interessieren, welcher "Mechanismus" hinter dieser hohen Geschwindigkeit steckt. Was hält ein Photon zusammen und auf der "Spur"?
wir kennen noch nicht alle Eigenschaften des Vakuums und des Energiepotentials. Es ist so lange Potential, bis es h erreicht. Virtuell wird nun ein Phtonenpaar konkretisiert. Die Art der Resonanz kann noch nicht beschrieben werden.

Die Teilcheneigenschaften bleiben noch akstrakt. Immerhin ergänzen sich beide Partner unter Befolgung der Erhaltungen. Vielleicht kann man sich komplexe, etwa spiegelbildliche Rotation nach dem Möbiusband oder anderes vorstellen.

Aber wieso macht es das? Energie war so lange Potential, bis es bei h die Aktionsschwelle überschritt. Verlängert man die Beobachtungen, mag man etwas auseinanderdrückendes annehmen. Impuls würde es unter Erhaltung in die Wirlichkeit drücken.

Damit haben wir wieder die Situation wie beim angeregten Elektron. Impulstausch, und wir haben ein Elektron und ein Photon. Physiker klopfen dem Wissbegierigen :p auf die Finger, wenn man was von Scheingungen auf einer Sinuslinie erzählt. Oszillation beschreibt die Situation offenbar besser. Es hat lang gedauert, bis ich das zumindest vom Grobschema her nachvollziehen konnte.

Und wieder die Frage: Wieso oszilliert es? Macht es das von sich aus als herausgehobene Dynamik? Oder verhält es sich aus eigener intrinsischer Veranlassung heraus? Das Bestreben mag die Beruhigung sein. Doch das kann es nicht. Die Dynamik unterhält sich, Teilung ist nicht möglich.

Selbst wenn man die Zeitdynamik als Energie auffasst, hat man die Fragestellung nur weiter verlagert...

Nun ja. Über die Teilcheneigenschaften und deren Verhalten mag man nach und nach vielleicht "deutlicher" erkennen, oder besser gesagt schließen können, welche Eigenschaften das Vakuum hat. Allerdings sind nur abstrakte Herleitungen denkbar. Wir werden niemals erkennen können, was es tatsächlich ist. Vakuum und Energie, das Quantenvakuum sind das System. Wie weit das Reinschauen gelingt, bleibt aber spannend.

Gruß Uranor

quick
18.04.08, 15:03
Hallo Uranor,


wir kennen noch nicht alle Eigenschaften des Vakuums und des Energiepotentials. Es ist so lange Potential, bis es h erreicht. Virtuell wird nun ein Photonenpaar konkretisiert. Die Art der Resonanz kann noch nicht beschrieben werden.
Man kann mit Hilfe von Formeln ganz gut beschreiben, was aus dem Vakuum "herauszuholen" ist. Das Vakuum selbst ist dabei aber wie eine "Black Box".
Seit über 100 Jahren ist der Äther nun "tot". Aber irgendeine Grundlage müssen die Eigenschaften des Vakuums doch haben! Auf welchem "Boden" errichten Physiker ihre Felder? Jeder Bauer müßte das eigentlich wissen...

Vakuum ist etwas Merkwürdiges, ähnlich merkwürdig, wie diese Flüssigkeiten (http://de.youtube.com/watch?v=vCHPo3EA7oE&feature=related).

mfg
quick

ingeniosus
18.04.08, 19:04
Teilchen sind Anregungszustände des Vakuums. Die Teilchenzahl muss nicht erhalten bleiben!
Dies scheint mir der kernige physikalische Neuwert zu sein.
Ihre mathematischen Theorien in Ehren, aber Mathematik hat ihre Grenzen -durch seine Axiome und Definitionen (Maxwell durch die Vektorrechnung und die Wellengleichungen durch die Differenzialgleichungsregeln).
Physik geht zunächst von der Natur aus. Es ist natürlich schön, wenn man mathematische Gesetze aufstellen kann wie De Brogly oder Max Planck, aber wir haben eine Dimension erreicht in der wir keine eindeutigen Grössen messen können.
Wir sind stolz, dass wir Atome sehen können, aber wir sind weit davon entfernt, damit nach den klassischen mechanischen Regeln etwas zu manipulieren ( rollen, stossen, beschleunigen und bremsen) . Eine Steuerung eines radioaktiven Zerfalles wäre doch ein lohnendes Ziel. Erst dann können wir doch von Quanten"mechanik" sprechen, aber das wird sicherlich noch Jahrzehnte dauern.

Nun aber zu Ihrem "Anregungszustand", der mich interessiert. Das geht konsequent weiter in die Richtung Zusammenfluss der Photonen und Wellentheorie. Das Planksche Wirkungsquantum hat eventuell ein Äquivalent im Quantensprung, was offen bleibt ist die Frage : Woher kommt die Frequenz. Ist Sie nur eine Rechengrösse? Nein, sie wurde ja offensichtlich nachgewiesen. Wie kann man sich einen Impuls eines masselosen Teilchens vorstellen? Sind noch kleinere Teilchen knapp vor der Entdeckung?
Ich habe mir erlaubt, einen neuen Ordnungsbegriff zu prägen: das Energon in seinem Energium, als Bezeichnung eines eigenständigen physikalischen Teiles oder Teilchens.
Man kann dann so schön eine "Energon-Kette" bilden vom Himmelkörper, beliebigen physikalischen Gegenstand, Molekül, Atom, Elektron, Atomkern, Quark bis zum masselosen Teilchen Photon. Diese "Kette" kann beliebig fortgesetzt gedacht werden, je nach Stand der Forschung und Technik. Das ist natürlich nur ein informationswissenschaftliches Paradigma, hilft mir aber sehr.

Uli
18.04.08, 20:04
Was ist Licht?
...


Hallo Querkopf,

danke für die exzellente und interessante Darstellung deiner Sicht zum Welle-Teilchen-Dualismus des Lichts. Ich denke, es ist alles logisch und richtig, was du geschrieben hast, und du bringst - vielleicht ein wenig aus der Sicht eines Theoretikers - diese Fragestellung auf den Punkt.
Vermutlich braucht es aber zumindest einen Physik-Studenten, um von deinem Beitrag profitieren zu können.

Gruss,
Uli

quick
18.04.08, 20:15
Hallo ingeniosus,

Eine Steuerung eines radioaktiven Zerfalles wäre doch ein lohnendes Ziel. Erst dann können wir doch von Quanten"mechanik" sprechen, aber das wird sicherlich noch Jahrzehnte dauern.
Ansatzweise hat mich dieses Problem auch schon beschäftigt.
http://www.quanten.de/forum/showthread.php5?p=18611#post18611
Ich vermute nur, solange es für die Kernkraftwerksbetreiber billiger ist, ihren strahlenden Abfall wegzusperren, tut sich in dieser Richtung nichts.


Woher kommt die Frequenz. Ist Sie nur eine Rechengrösse? Nein, sie wurde ja offensichtlich nachgewiesen. Wie kann man sich einen Impuls eines masselosen Teilchens vorstellen? Sind noch kleinere Teilchen knapp vor der Entdeckung?
Meines Wissens gibt es kein Experiment, das die Frequenz eines Photons nachweisen kann. Es ist immer nur eine Frequenz, durch welche dieses Photon erzeugt wurde oder selbst erzeugen kann. E = h*f charakterisiert also eher die Wirkungsweise bei der Erzeugung oder Vernichtung und nicht den Zustand des Photons selbst.


Ich habe mir erlaubt, einen neuen Ordnungsbegriff zu prägen: das Energon in seinem Energium, als Bezeichnung eines eigenständigen physikalischen Teiles oder Teilchens.
Das hört sich an, wie die Energiedichte-Oszillationen in den "Archen" von Uwebus (http://uwebus.de/rzg6/020.htm). Er meint damit alles erklären zu können, leider kann ich es größtenteils nicht nachvollziehen.

mfg
quick

Lambert
18.04.08, 20:49
Hallo Querkopf,

Vermutlich braucht es aber zumindest einen Physik-Studenten, um von deinem Beitrag profitieren zu können.

Gruss,
Uli

Hallo Uli,

darf man auch exmatrikulierter Elektroing. mit Physik als Nebenfach sein, um den Beitrag zu schätzen?

Ich plädiere heftig für Dialog der Disziplinen :D -- auch wenn das manchmal für die Gesprächspartner arg anstrengend ist.

Gruß,
Lambert

Uli
18.04.08, 22:05
Hallo Uli,

darf man auch exmatrikulierter Elektroing. mit Physik als Nebenfach sein, um den Beitrag zu schätzen?

...
Gruß,
Lambert

Ich schrieb "können" und nicht "dürfen". Wenn dir Querkopfs Beitrag einige Ideen vermitteln konnte, dann freu dich doch einfach darüber anstatt dich bei mir zu beschweren.

Gruss,
Uli

Uranor
18.04.08, 22:10
Hallo quick,
Man kann mit Hilfe von Formeln ganz gut beschreiben, was aus dem Vakuum "herauszuholen" ist. Das Vakuum selbst ist dabei aber wie eine "Black Box".
Ja, man schabt aus der Eisbox Eisballen, besteht aber selbst aus Eis. Man kann nun die Eigenschaften von Eisballen erforschen. Aber was ist es? Könnte man das feststellen, wüsste man auch, was sich in der Box befindet. So meinte ich meinen obigen Versuch.

Seit über 100 Jahren ist der Äther nun "tot". Aber irgendeine Grundlage müssen die Eigenschaften des Vakuums doch haben! Auf welchem "Boden" errichten Physiker ihre Felder? Jeder Bauer müßte das eigentlich wissen...
google ist gefüllt mit "Eigenschaften des Vakuums". Und dann liest man über den Inhalt wie etwa Wörmestrahlung.

Das Vakuum ist polarisierbar. Selbst solche klaren Definitionen entdecke ich nicht. Und das wäre auch schon die einzige Eigenschaft, die mir bekannt ist.

Der schwache Trost: Ein Backstein hat die Eigenschaft, kühl sein zu können. Na toll, ich habe den Backstein verstanden. :p

Der wirkliche Trost: Das von Kurt Gödel entdeckte Unvollständigkeitstheorem.

Ole. Wenn für uns als Bestandteile des Systems unverstehbar bleiben muss, was es ist, dann können wir doch immerhin versuchen, die Eigenschaften zu erkunden. Also, es ist polarisierbar. Welche Eigenschaften wurden ansonsten bekannt?

Gruß Uranor

quick
20.04.08, 01:37
Hallo Uranor,


Ole. Wenn für uns als Bestandteile des Systems unverstehbar bleiben muss, was es ist, dann können wir doch immerhin versuchen, die Eigenschaften zu erkunden. Also, es ist polarisierbar. Welche Eigenschaften wurden ansonsten bekannt?
Bei ähnlich lautenden Fragen habe ich schon öfter den Casimir-Effekt erwähnt.
Den kennst Du sicherlich. Der soll angeblich auf die Fluktuationen des Vakuums zurückzuführen sein. Sind damit virtuelle Teilchen gemeint? -Ich weiß es nicht.
Wie immer, wenn es "eng" wird, kann man diese mit der Unschärferelation begründen.
Letztere wiederum soll dafür verantwortlich sein, dass nichts zur vollkommenen Ruhe kommen kann, alles vollführt eine Nullpunktsschwingung, also auch das Vakuum.

mfg
quick

Uranor
20.04.08, 03:48
moin - moin, quick!

Vielleicht kann man sich das Vakuum bildanalog wie eine Art Flüssigpulvergasförmigkeit vorstellen? Das Zeug kan niemals perfekt entropisch sei. Denn wäre es so, wäre wegen der exakten Gleichförmigkeit nichts messbar, es wäre keine Existenz. Da offenbar etwas ist, müssen wir fordern, dass niemals und nirgends perfekte Entropie möglich sein kann. Denn wäre sie möglich, wäre sie wegen der günstigsten Energiesituation realisiert.

Somit ist die Heisenbergsche Unschärferelation offenbar naturfundamental. Wir haben also eine stets lokal unscharfe Grundsituation. Messbar ist sie nicht, sie liegt unterhalb des Qirkungsquantums h. Offensichtlich sind messbare Potentiale möglich. Wir erkennen das physikalisch offenbar minimalste, die Vakuum-Fluktuation auf Basis einer konkretisierenden Paarerzeugung. Solche virtuellen Entstehungen sind kohärente Photonenpaare. Bei WW verlieren sie die kohärente Eigenschaft.

Wie hoch ist Fluktuation realisierbar? Entstehen auch Elektron- Positron-Paare?

Der Casimier-Effekt zeigt sich als unterdruck-basiert. Überall sonst sind alle energetisch möglichen (Frequenzen) realisiert. Zwischen extrem eng benachbarten Objekten können sich nicht mehr alle möglichen Fluktuationen resonant realisieren. Also entstehen sie gar nicht. Die rundum realisierte Situation übt Druck aus, dem Casimir-Druck.


Hier wird auch gleich ein Hinweis erkennbar: Größte Wennenlängen, wie sie für die Gravitation postuliert werden, sollten bei zunehmender Fluktuationendichte immer weniger möglich sein. Fermionen werden also wohl ab einer gewissen Dichte nicht mehr realisiert. Genau so erleben wir das.

Man muss die Situation nun nicht als statisch auffassen. Imerhin erfahren wir fermionische Dynamik, offenbar als beschleunigte Expansion bekannt. Fermionische Objekte sorgen für Raumkrümmung. Wird Raumkrümmung lokal so stark, dass weitere Fermionen virtuell gebildet werden können, so mag es sein, dass genau das wieder geschieht. Irgendwo will einleuchten, dass sich dadurch bestehende, weiträumige Strukturen immer weiter voneiander entfernen. Damit das so klappen kann, müsste unendlich Krümmbarkeit des Vakuuks gefordert werden. Und ob das sein kann... also gel, also hier :p , ich habe nur einen Gedankengang ausgesprochen. Sowas scheint nicht diskussionsfähig zu sein, denn man hört derartige Überlegungen nicht in der Fachwelt.

Auf jeden Fal scheint es eine natürlicherweise maximale Fluktuationsdichte zu geben. Höherenergetische Fluktuationen würden somit immer unwahrscheinlicher. Es stellt sich eine flexible Grundsituation ein. - Möglicherweise wird genau deswegen die Casimirdruck überall gleich gemessen. Ein quasieuklidisches Vakuum muss es denoch nicht geben. Die Großstrukturen dürften durchaus das Vakuum strukturell krümmend formen. -- Jo, auch so betrachtet kann das Vakuum expansiv sein. Man hat aber wohl allenfalls überlegungen.

Hmmm, der Faden spann sich jetzt anders, als ich es wollte. Macht aber vielleicht nix. Was weiß ich schon von der Virtualität und von der Physik?

Gruß Uranor

ingeniosus
21.04.08, 14:07
Das hört sich an, wie die Energiedichte-Oszillationen in den "Archen" von Uwebus (http://uwebus.de/rzg6/020.htm). Er meint damit alles erklären zu können, leider kann ich es größtenteils nicht nachvollziehen.


Ein kurzer Blick in das Labyrith von uwebus bringt mich zur selben Meinung!

Nein! Nein! Energon-Energium soll nur ein verallgemeinender Name sein für alle Teilchen in der Physik sein.
Energon für Teilchen
Energium für das "Reich" drumherum

Fast noch wie verbale science fiction, eine Vereinfachung, aber schon mit dem Hintergedanken klassische Mechanik und Quantenmechanik theoretisch zu verbinden. Nur nicht zu schwierig denken! Nicht mehr als das! Besten Dank für Ihre Antwort!

ingeniosus
21.04.08, 16:03
Da offenbar etwas ist, müssen wir fordern, dass niemals und nirgends perfekte Entropie möglich sein kann. Denn wäre sie möglich, wäre sie wegen der günstigsten Energiesituation realisiert.

Hallo Uranor

Der Begriff Entropie scheint mir persönlich wenig hilfreich zu sein. Wo gibt es eine praktisches Beispiel dafür in der reelen Natur?
Man hat einen Begriff definiert vor etwa 100 jahren und glaubte eine Weltformel zu haben, heute sieht man das schon differenzierter.
Man kennt die Chaosforschung, man kennt die Systemtheorie, in der Mathematik hat Gödel zugeschlagen und ich frage mich, wo ist die Entropie geblieben. Eine Massengleichverteilung ist offensichtlich nicht erreichbar, wo wäre dann die schwarze Materie geblieben?
Auch der Endzustand der Entropie scheint mir heute sehr konstruiert.

mbG

ingeniosus
21.04.08, 16:54
Und wieder die Frage: Wieso oszilliert es? Macht es das von sich aus als herausgehobene Dynamik? Oder verhält es sich aus eigener intrinsischer Veranlassung heraus? Das Bestreben mag die Beruhigung sein. Doch das kann es nicht. Die Dynamik unterhält sich, Teilung ist nicht möglich.
Selbst wenn man die Zeitdynamik als Energie auffasst, hat man die Fragestellung nur weiter verlagert...


Hallo Uranor!

Das ist auch für mich die entscheidende Frage der gegenwärtigen Theoretischen Physik. Da schlummert sicherlich ein Nobelpreis dahinter!

Als praktischer Informatiker sehe ich die Analogie: Planeten kreisen um die Sonne - Elektronen kreisen um den Atomkern. Da stimmt ja noch alles.
Energieniveaus sind im Makrokosmos eigentlich nicht bekannt, es gibt nur Massen.
Eine Quantelung ist aber eben durch diese Niveaus begründet.
Da wäre die Quantelung nur durch die Menge an Teilchen erklärbar, also ein Ordnungskriterium, aber warum gerade um diese Plancksche Wirkungsquantum?

Da wäre der logische Pfade doch, dieses Wirkungsquantum genauer zu untersuchen....es muss zu weiteren kleineren Teilchen (Energoi oder Energons ) führen!

Uranor
21.04.08, 17:41
Hallo ingeniosus!
Man kennt die Chaosforschung, man kennt die Systemtheorie, in der Mathematik hat Gödel zugeschlagen und ich frage mich, wo ist die Entropie geblieben. Eine Massengleichverteilung ist offensichtlich nicht erreichbar, wo wäre dann die schwarze Materie geblieben?
Auch der Endzustand der Entropie scheint mir heute sehr konstruiert.
Man kent allerdings auch die Unschärferelation nach Heisenberg. In der Natur kann es keine Determiniertheit geben. Es sieht doch so aus, als müsse bereits die Möglichkeit zu einem solchen Zustand die Folge beinhalten, dass es die Natur gar nicht geben kann. Denn das wäre die perfekte Entropie = keinerlei Unterschied, keinerlei Messbarkein, eben keinerlei Natur.

Ich werte die bekanten Parameter so aus:

Was angestrebt wird, ist eine Sache. So wie wir die Natur kennen, wird der energiegünstigste Zusand, die perfekte Entropie, also E=0 angestrebt.
Erreichbar ist nicht das angestrebte sondern das mögliche. Möglich sind:

-- Unbestimmtheit gem. QT. Darauf resultiert der echte, der Quantenzufall.
-- Darauf sind die Erhaltungssätze als tiefste Grundlagen gestützt und abgesichert.
-- Die Befähigung zur Selbstorganisation basiert auf den beiden Urgrundlagen.
-- Ermöglichte Freiheitsgrade, offenbar als Folge der Selbstorganisation.

Ich denke, das sind die Grundlagen, auf welche die Natur baut, welche wir auch nachvollziehen können. Was braucht es anderes bzw. gar mehr?


Gruß Uranor

Uranor
21.04.08, 18:44
Hallo ingeniosus!

Hmmm. Ich denke, solche Kleinsteinheiten muss es gar nicht geben. Nach der Unschärferelation ist ohnehin jeder offenbar beliebig kleine Betrag, jedes beliebig kleine Potential möglich.

Gibt es tatsächlich etwas feinstes, wie sollte das feststellbar sein? Am Wirkungsquantum h endet unsere Nachweismöglichkeit. Feineres mag der Mengenlehre direkt gehorchen, ggf. je nach relativem Bezug ineinander übergehen können. Schließlich sind die größeren Potentiale sicher nicht ad hoc vorhanden.

Als praktischer Informatiker sehe ich die Analogie: Planeten kreisen um die Sonne - Elektronen kreisen um den Atomkern. Da stimmt ja noch alles.
Übrig bleibt allerdings nur die Dimensionierung in der Symmetrie. In der Quantenwelt haben wir es mit den ElMag-Potentialen +Hadron und -Elektron bzw. mit den gegenpolaren Antiteilchen-Situationen zu tun. Dort hat die schwächste der Kräfte, die Gravitation, nichts zu melden. Oberhalb der molekularen Verbünde gewinnt die Gravitation an Bedeutung. Kosmische Bahnen gründen auf dem Äquavilenzprinzip zwischen Masse und Beharrung.

Im Grunde haben wir es zu tun mit:

-- Quarkwelt (Starke und Schwache WW)
-- Elektrodymaische Welt (Orbitale und Bindungskräfte)
-- Gravitative Makrowelt.

Energieniveaus sind im Makrokosmos eigentlich nicht bekannt, es gibt nur Massen.
Eine Quantelung ist aber eben durch diese Niveaus begründet.
Da wäre die Quantelung nur durch die Menge an Teilchen erklärbar, also ein Ordnungskriterium, aber warum gerade um diese Plancksche Wirkungsquantum?
Energieniveaus kenen wir in der Praxis auf die 4 Grundkräfte basiert. Die Einteilung muss aber grundsätzlich nicht relevant sein. Forscher erwarten eine große Vereinigung aller Kräfte bei extrem hohen Energien. Auch nur annähernd vergleichbares zeigen allerdings nicht mal die Quasare als Neutronensterne, plasmatische Situationen. Auf dem Hintergrund müht/mühte man sich um eine TOE, man postulierte das Teilchen Gottes usw.

Das Plancksche Wirkungsquantum betrachte ich mal so "freihand" auf den Grundeigenschaften basierend. Die Polarisierbarkeit des Quantenvakuums wurde festgestellt. Vielleicht gehört auch eine Massebedingte Krümmbarkeit zu den Grundeigenschaften?

Unterhalb von h scheinen keine Wirklichkeiten, keine "Heraushebungen" stattzufinden. Ein minimales Energieniveau scheint für die Realisiebarkeit von Objekteigenschaften notwendig zu sein. Was keine Eigenschaften realisiert hat, scheint entspr. den fundamentalen Erhaltungssätzen nicht messbar sein zu können. Via Messung erfassbar sind minimal nur Eigenschaftenwerte.

Etwa auf der Basis wird man sich die grundsätzliche Wirklichkeits-Trennung vorstellen können/müssen. Die Physik ist bis runter zum Wirkungsquantum h definiert. Darunter kann sie ganz einfach nichts tun. Herleitungen, Folgerungen sind allerdings durchaus möglich. Nur kann das dann nicht weiter nachprüfbar sein.


Gruß Uranor

Querkopf
21.04.08, 20:35
Hallo Querkopf,

danke für die exzellente und interessante Darstellung deiner Sicht zum Welle-Teilchen-Dualismus des Lichts. Ich denke, es ist alles logisch und richtig, was du geschrieben hast, und du bringst - vielleicht ein wenig aus der Sicht eines Theoretikers - diese Fragestellung auf den Punkt.
Vermutlich braucht es aber zumindest einen Physik-Studenten, um von deinem Beitrag profitieren zu können.
Vielen Dank für den Lob.
Meine Zielgruppe waren auch nicht Laien völlig ohne Hintergund. Ich denke ohne bestimmte Grundlagen macht es ohne keinen Sinn sich über die frage nach dem Licht Gedanken zu machen, es muss unverständlich sein (Obwohl es ein recht gutes Buch von Feynman gibt, allerdings auf Grundlage des Pfadintegralformalismus).

Mein Ziel war es vielmehr interessierten Laien mit Hintergrundwissen z.B. aus der Chemie, der Elektrotechnik oder dem Maschinenbau einen Gewissen Einblick in die Struktur der QED zu geben, also Leuten die mit einer Differentialgleichung, vielleicht auch mit den Maxwellgleichungen und ein wenig mit Quantenmechanik vertraut sind.
Deshalb auch die Starke Anlehnung an die Maxwellgleichungen.
Ich denke viele Elektroingenieure z.B. hegen ein gewisses Misstrauen der Modernen Physik gegenüber, benutzen aber die Maxwellgleichung oder ihre Ableitungen mit großer Selbstverständlichkeit.
Daher war es mein Ziel zu zeigen, das QED keine Magie ist, sondern die Konsequente Fortentwicklung der Maxwellgleichungen, mit dem Ziel, die im Experiment gefundenen Energiepaketlösungen zu erhalten. Bestimmte Aspekte wie Kommutatorrelationen habe ich daher auch unter den Tisch fallen lassen.
Ich werde im folgenden meine „Philosophie“ anhand einiger Kommentare erleutern.

Ihre mathematischen Theorien in Ehren, aber Mathematik hat ihre Grenzen -durch seine Axiome und Definitionen (Maxwell durch die Vektorrechnung und die Wellengleichungen durch die Differenzialgleichungsregeln).
Physik geht zunächst von der Natur aus. Es ist natürlich schön, wenn man mathematische Gesetze aufstellen kann wie De Brogly oder Max Planck, aber wir haben eine Dimension erreicht in der wir keine eindeutigen Grössen messen können.
Dann argumentieren wir unabhängig ob in der klassischen Physik oder in der Quantenphysik von einer gänzlich anderen erkenntnistheoretischen Grundposition aus.
Meine Ausgangsposition ist in wenigen Worten etwa folgende:
Eine physikalische Theorie stellt mathematische Beziehungen zwischen Messgrößen her.
Einige dieser Messgroßen sind fundamental, ihre Einheiten sind durch einen Messapparat und ein Messverfahren definiert (z.B. in Deutschland durch die PTB in Braunschweig). Andere Größen lassen sich dann daraus ableiten.
Wenn ich also von einer Länge spreche, dann verstehe ich darunter etwas was man erhält, wenn man einen Laser und eine Atomuhr nehme einer bestimmten Messvorschrift folge und dass Ergebnis auf eine Standartmessung (Definition des Meters) normiere.
Ziel der Theoretischen Physik, ist es dabei mit einem Minimum an konsistenten mathematischen Aussagen auszukommen, aus denen sich alle Messresultate ableiten lassen.
Die übliche vorgehensweise dabei ist, das ein System solcher Aussagen als Axiome (oder first principles) postuliert wird und ihre Richtigkeit durch die Überprüfung der Richtigkeit der aus ihnen abgeleiteten Vorhersagen gezeigt wird.

Ein Beispiel sind die Maxwellgleichungen der Elektrodynamik. Sie wurden von Maxwell einfach postuliert (genauer gesagt, den bekannten experimentellen aussagen wurde aus Symmetrieüberlegungen der Maxwellsche Verschiebungsström hinzugefügt).

Aus diesen Axiomen lassen sich die Knoten und Maschenregeln für Stromkreise, die Existenz Elektromagnetischer Wellen, deren Konstante Ausbreitungsgeschwindigkeit im Vakuum und die gesamte klassische Optik ableiten.
Folglich ist die Theorie im klassischen Fall ein sehr gute Theorie.

In Quantentheorien gehe ich nicht anders vor. Ich postuliere bestimmte mathematische Beziehungen, in einer Quantentheorie kann das z.B. der fundamentale Kommutator sein (also die mathematische Version der Heisenbergschen Unschärfe) und bekomme dann bestimme abgeleitete aussagen, z.B. Streuwinkel zwischen Teilchen oder Energieniveaus in einem Wasserstoffatom.
Zu anderen Dingen kann ich ohnehin nichts sagen. Was ein Elektron macht wenn ich nicht hingucke kann ich nicht sagen. Zumindest nicht in der Physik, die mathematische Beziehungen zwischen Messgrößen beschreibt.
Von daher ist en Ansatz der Natur beschreibt (im Wörtlichen Sinne) ziemlich naiv und durch die Quantenphysik eigentlich widerlegt. Abgesehen davon stellt es einen Rückschritt in die Zeit vor 1687 (Philosophiae Naturalis Principia Mathematica von Sir Isaac Newton) dar.
Es mag vielleicht in der Schule oder auch im Ingenieurstudium nicht klar werden, aber der gute Isaac Newton hat einfach die Differential und Integralrechnung entwickelt und in diesem neuen mathematischen Kalkül drei physikalische Axiome aufgestellt, aus denen sich die komplette klassische Mechanik ableiten lässt. Das ist seit 321 Jahre der Weltstandard in Sachen Konstruktion physikalischer Theorien. Deshalb ist der Mann so wichtig für das abendländische denken. Nicht wegen ein bisschen läppischer Mechanik, die man anders viel effektiver betreiben kann, sondern weil er die Art und Weise über Physik nachzudenken verändert hat.

Die Zeit in die du Zurück willst ist eine in der Europa mal gerade die Folgen des dreißigjährigen Krieges überwunden hat, Hexen noch fleißig verbrannt wurden und in Spanien die Inquisition wütete.



Wir sind stolz, dass wir Atome sehen können, aber wir sind weit davon entfernt, damit nach den klassischen mechanischen Regeln etwas zu manipulieren ( rollen, stossen, beschleunigen und bremsen) . Eine Steuerung eines radioaktiven Zerfalles wäre doch ein lohnendes Ziel. Erst dann können wir doch von Quanten"mechanik" sprechen, aber das wird sicherlich noch Jahrzehnte dauern.
Der Punkt ist, das sich Atome offensichtlich nicht nach klassischen Regeln verhalten. Die Quantenmechanik postuliert dieses Quantenverhalten, daher ich führe ausgehend von der klassischen Mechanik (im Lagrange oder Hamilton Formalismus) eine Quantisierung durch.
Entscheidend ist, dass ich im Grenzfall die klassische Mechanik erhalte. Die geometrische Struktur der Quantenmechanik ist sogar weitestgehend identisch mit der der Mechanik (Stichwort Phasenraum als durch eine Lie -Klammer definierte Symplektische Mannigfaltigkeit, ein Aspekt der aber im Ingenieurstudium wohl vernachlässigt wird).

Im rahmen dieser Regeln für unsere Atommechanik kann ich natürlich auch Manipulationen durchführen.
Vorreiter dabei ist IBM in Almaden und Zürich Rüschlikon, vorllem mit späktakulären Experimenten in den 90ern:
Hübschem Atomarem Firmennamen: http://www.almaden.ibm.com/vis/stm/images/stm10.jpg
Kenji Zeichen (Eisenatoma auf Kupfer)
http://www.almaden.ibm.com/vis/stm/images/stm11.jpg
Quantenkorallen: Atom für Atom zusammengesetzt:
http://www.almaden.ibm.com/vis/stm/images/stm16.jpg
Molecule Cascades: http://www.almaden.ibm.com/st/past_projects/molecule_cascades/

... und vielem mehr http://www.almaden.ibm.com/almaden/projects/#Science%20&%20Technology




Soviel zu meinem Ausgangspunkt.


Nun zu einigen Punkten, die zu zeigen mir wichtig ist und die ich in vielen anderen Beiträgen schon angesprochen habe

1. Teilchen sind Wellen?
Die in der Schule oder der Populärwissenschaft üblicherweise angesprochenen Wellenfunktionen sind nur eine spezielle Darstellungsweise der Quantenmechanik. Ich kann Quantenmechanik sehr gut ohne Wellenfunktionen beschreibe, wenn ich zu komplexeren Problemen (Spinprobleme oder Verschränkung z.B.) übergehe, ist das sogar fast unvermeidbar.
Deshalb sollte man diesen Punkt nicht überinterpretieren.
In der Tat sagt mir die Quantenfeldtheorie dann auch, was ein Teilchen ist.

Selbst wenn ich mit Wellenfunktionen rechne habe ich nicht unbedingt sinusförmige wellen mit einem Wellenberg und einem Tal die sich dann zu einem Interferenzmuster überlagern. Die Schrödingergleichung in der Ortsdarstellung ist eine Wellengleichung, daher ein spezieller Typ Differentialgleichung. Alle ihre Lösungen werden wellen genannt, unabhängig davon wie grottig sie aussehen. Abhängig Vom Potential und den Randbedingungen, kann eine solche „Welle“ z.B. auch einen Exponentiellen Zerfall darstellen (Tunneleffekt).
Durch ein zu starkes klammern an der Anschauung steht man sich hier nur selbst im Weg.

Querkopf
21.04.08, 20:36
2. Was ist ein Vakuum?
Das sagt Wikipedia dazu: (über das Vakuum der Experimentalphysiker)
http://de.wikipedia.org/wiki/Vakuum
Ein Paar Zitate aus diesem Thread:


wir kennen noch nicht alle Eigenschaften des Vakuums und des Energiepotentials. Es ist so lange Potential, bis es h erreicht. Virtuell wird nun ein Phtonenpaar konkretisiert. Die Art der Resonanz kann noch nicht beschrieben werden.

Nun ja. Über die Teilcheneigenschaften und deren Verhalten mag man nach und nach vielleicht "deutlicher" erkennen, oder besser gesagt schließen können, welche Eigenschaften das Vakuum hat. Allerdings sind nur abstrakte Herleitungen denkbar. Wir werden niemals erkennen können, was es tatsächlich ist. Vakuum und Energie, das Quantenvakuum sind das System. Wie weit das Reinschauen gelingt, bleibt aber spannend

Man kann mit Hilfe von Formeln ganz gut beschreiben, was aus dem Vakuum "herauszuholen" ist. Das Vakuum selbst ist dabei aber wie eine "Black Box".
Seit über 100 Jahren ist der Äther nun "tot". Aber irgendeine Grundlage müssen die Eigenschaften des Vakuums doch haben! Auf welchem "Boden" errichten Physiker ihre Felder? Jeder Bauer müßte das eigentlich wissen...
Der Vakuumzustand in der Quantenfeldtheorie ist ein Wohldefiniertes mathematisches Objekt, nämlich der Grundzustand des Systems!
Daher: Die Anwendung eines Vernichtungsoperators auf den Vakumzustand resultiert in einer Null. Jede Rechnung, die einen Solchen Prozess mit einschließt, resultiert in der Wahrscheinlichkeit Null für diesen Prozess.
Eine wichtige folge ist, das als eine Art Kausalitätsforderung in der Rechnung immer erst Teilchen erzeugt und dann Teilchen vernichtet werden können.
Das Vakuum hat keinen Impuls, keine Ladung, keinen Spin, ... denn dies sind alles Teilcheneigenschaften, stecken also in den Operatoren.
Es macht keinen Sinn sich darüber zu streiten was dieses Vakuum in Realität ist, denn seine Existenz ist nur eine sinnvolle mathematische Forderung (das Spektrum meines Hamiltonoperators soll noch unten beschränkt sein, daher auch seine Energie), hat also nichts mit einem Äther zu tun.
Auf einer gewissen Ebene ist Theoretische Physik weitgehend Mathematik. Begriffe werden Mathematisch definiert. Mann kann da nicht einfach Irgendetwas reininterpretieren, was in der Definition nicht steht. Dafür hat man nämlich Definitionen.

3. Virtuelle Teilchen
Virtuelle Teilchen klingt furchtbar geheimnisvoll. Der bessere Name wäre Wechselwirkungsteilchen.
Zunächst ein Blick auf die Maxwellgleichungen, bzw. deren Lagrangedichte. Diese enthält einen Term für das freie Feld und einen Wechselwirkungsterm, der mein Feld an Ladungen und Ströme koppelt. Mein Freises elektromagnetisches Feld wechselwirkt nicht, sondern es braucht ein anderes Feld dazu, es ist nicht Selbstwechselwirkend. Ich kann in der folge elektrische und magnetische Felder Superponieren, meine Theorie ist linear.

In der QED habe ich mein Maxwellfeld für die Photonen und mein Diracfeld für geladene Teilchen bzw. deren Antiteilchen, der Einfachheit halber im folgenden nur Elektronen und Positronen.
Beide Felder sind nicht Selbstwechselwirkend. Der wechselwirkungsterm koppelt das elektromagnetische Feld an mein Elektron - und mein Positronfeld. Ich habe also drei Felder in meinem Term. Die einfachste Mögliche Wechselwirkung ist als Fyenmandiagramm dargestellt ein Dreibein, also ein Wechselwirkungspunkt mit drei freien Beinen für ein Elektron, ein Positron und ein Photon.

Angenommen, ich habe einen Prozess, bei dem ein Elektron und ein Positron aneinander gestreut werden. Klassisch ist das eine Art Billiardstoß. Meine Teilchen fliegen unter einem bestimmten Winkel aufeinander zu, kollidieren und entfernen sich unter einem bestimmten anderen Winkel wieder.

Wie sieht das in der QED aus? Die einfachste Möglichkeit einen solchen Prozess darzustellen ist durch zwei Wechselwirkungspunkte. Ich klebe im Diagramm also die Arme für die Photonen aus zwei Dreibeinen zusammen.
Meine Felder beschreiben das erzeugen und vernichten von Teilchen, also müssen auch Wechselwirkungen immer durch Erzeugung und Vernichtung beschrieben werden.

Im Obigen Prozess wird ein Photon erzeugt und ein Elektron – Positron – Paar vernichtet, worauf ein neues paar mit anderen Impulsen erzeugt und das Photon vernichtet wird.
Das Photon existiert nur zwischen den zwei Wechselwirkungspunkten, dient also nur zur Beschreibung dieser, während die freien arme die echten messbaren Teilchen sind (also eine entsprechende Energie und einen entsprechenden Impuls haben). Deshalb nennt man diese Photonen Wechselwirkungsteilchen oder virtuelle Teilchen und spricht eigentlich nichts dagegen sie als ein Artefakt des Formalismus (Störungstheorie) zu sehen.
In der Tat habe ich gleichartige Strukturen auch wenn ich Störungstheorie für eine klassische Feldtheorie mache, nur habe ich dort keine Teilcheninterpretation.

Soviel erst einmal zu den Grundlagen. Kommentare zu Anwendungen, wie dem Casimireffekt: http://www.casimir.rl.ac.uk/default.htm werden folgen.

Eine Anmerkung nur:

Als praktischer Informatiker sehe ich die Analogie: Planeten kreisen um die Sonne - Elektronen kreisen um den Atomkern. Da stimmt ja noch alles.
Elektronen kreisen keineswegs wie Planeten um den Atomkern. Das ist seit fast 100 Jahren überholt.

quick
22.04.08, 01:40
Hallo Querkopf,

den Dankesworten von Uli schließe ich mich gern an.
Ich bin schon richtig gespannt auf Deinen Kommentar zum Casimir-Effekt.
Hier würde mich insbesondere die Abgrenzung zu anderen Oberflächenkräften interessieren.

mfg
quick

uwebus
22.04.08, 12:19
Querkopf,
----
Es macht keinen Sinn sich darüber zu streiten was dieses Vakuum in Realität ist, denn seine Existenz ist nur eine sinnvolle mathematische Forderung.
----
Was soll das? Wenn Du das Universum zu verstehen suchst, mußt Du von der Realität ausgehen und nicht von irgendwelchen mathematischen Formeln. Das Vakuum existiert nun mal als Realität, oder meinst Du, wir bilden uns den Abstand zwischen den Gestirnen nur ein?

Erst wenn Du das reale Vakuum zur Kenntnis nimmst und dessen Funktionen verstehst, kommst Du weiter im Weltverständnis. Alles andere ist nichts anderes als mathematischer Frühsport.

Gruß

Querkopf
22.04.08, 12:41
uwebus,

Du hast offensichtlich nicht gelesen. Ich rede vom Grundzustand eines Systems, den man manchmal auch Vakuum nennt. Was soll das mit dem Raum zwischen den Sternen zu tun haben???

ingeniosus
22.04.08, 14:20
Eine Anmerkung nur:
Elektronen kreisen keineswegs wie Planeten um den Atomkern. Das ist seit fast 100 Jahren überholt.

Da scheint etwas in falsche Bahnen zu laufen. Es ist wohl dem kleinsten Physik-Sprössling klar, das das nicht exakt so ist.

Es geht um die Analogie der Bewegung : das Kreisen - nicht mehr!

Querkopf
22.04.08, 17:15
Da scheint etwas in falsche Bahnen zu laufen. Es ist wohl dem kleinsten Physik-Sprössling klar, das das nicht exakt so ist.
Es geht um die Analogie der Bewegung : das Kreisen - nicht mehr!
Wenn ich ein wasserstoffähnliches Atom betrachte, dann ist das Problem zentralsymmetrisch, also Isotrop und daher gilt Drehimpulserhaltung.
Mehr kann ich darüber nicht sagen. Wo du dein kreisen hernimmst, ist mir schleierhaft.

uwebus
23.04.08, 13:59
Querkopf,
----
Du hast offensichtlich nicht gelesen. Ich rede vom Grundzustand eines Systems, den man manchmal auch Vakuum nennt. Was soll das mit dem Raum zwischen den Sternen zu tun haben???
----
Das "System" ist das Universum und den Raum zwischen den Sternen nennt man Vakuum, oder ist bei Dir der interplanetare, interstellare oder intergalaktische Raum kein Vakuum?

http://de.wikipedia.org/wiki/Vakuumenergie

"Die Vakuumenergie ist die Energie des „leeren Raumes“ bei vollständiger Abwesenheit von Feldern und Teilchen des Standardmodells der Elementarteilchenphysik."

Ich würde sagen, laßt Euch erst einmal etwas vernünftiges zum Vakuum einfallen, ehe Ihr damit rechnet!

Gruß

Querkopf
23.04.08, 15:45
Dann sollte man den Wiki Artikel überarbeiten (aber das steht ja auch in dem Roten Kasten am Anfang). Wikipedia ist auch nicht der Ersatz für ein Fachbuch.

Ein Vakuumzustand ist nur der Grundzustand eines Systems in der Vielteilchenquantenmechanik bzw. der Quantenfeldtheorie.
Dieser Vakuumzustand, kann durchaus ein Kollektiv von Elektronen Atomrümpfen oder Spins sein, also alles andere als ein leerer Raum.
Im Falle des Leeren Raumes zwischen den Sternen gebrauche ich den Begriff Vakuum in anderem Zusammenhang!
Gegebenenfalls kann ich postulieren, mein leerer Raum, also mein Vakuum sei gleichzeitig der Vakuumzustand einer QFT.
Man muss Begriffe sehr sauber auseinander halten.
Selbst im Standardmodell habe ich verschiedene Vakua. Wenn ich nämlich Störungsrechnung mache, betrachte ich ein Interagierendes Vakuum, nämlich einen Grundzustand, in dem ich ganz langsam eine Wechselwirkung angeschaltet habe (adiabatische Näherung) und das ist ein andere Vakuum als in einem völlig leeren Raum (den es ohnehin nicht gibt).

Querkopf
23.04.08, 16:34
Casimir Effekt:
Der Quantenmechanische Harmonische Oszillator hat im Grundzustand eine nichtverschwindende Energie von ½ h-quer Omega. Also auch die Nullmode schwingt noch mit einer gewissen Energie.

Im Falle eines Quantisierten Feldes summieren wir über eine unendliche Anzahl von Harmonischen Oszillatoren. Da steckt also selbst im Grundzustand eine Menge Energie drin. Die lässt sich aber leider nur in SciFI Filmen nutzen.
Warum? Weil ein Vakuumzustand per Definition keine Teilchen enthält die irgendwie Wechselwirken können.

Wir haben unseren Vakuumzustand (der Grundzustand unseres quantisierten Elektromagnetischen Feldes um Missverständnissen zu vermeiden) und unser Maxwellfeld.
Eine Frage die man sich stellen kann ist, wie hoch ist die Wahrscheinlichkeit im Vakuum spontan ein elektrisches oder magnetisches Feld anzutreffen.
Die Wahrscheinlichkeit ist Null. Denn unser quantisiertes Feld ist ja ein Operator der Erzeuger und Vernichter von Teilchen enthält. Im Vakuum ist die Wahrscheinlichkeit für das Erzeugen und Vernichten von Teilchen aber Null, denn wir haben per Definition genau Null Teilchen.
Wir müssen uns also keine sorgen machen, das es plötzlich aus dem nichts zu Blitzen anfängt.
Der Erfahrene Statistiker weiß aber, dass es neben einem Wert Für die Wahrscheinlichkeit auch noch Streumaße und ähnliches gibt. Wir rechen also weiter.
Wenn ich nämlich die Wahrscheinlichkeit z.B. für das Quadrat eines elektrischen Feldes ausrechne (das zweite stochastische Moment, das eng mit der Varianz verbunden ist), bekomme ich je nach Randbedingungen für mein Feld einen von Null verschiedenen Wert heraus. Ich habe also im mittel kein Feld und kann auch keines messen aber zwischendurch brodelt es kräftig und ich habe Fluktuationen.

Wie kann ich das interpretieren? Nun, ich habe mit der Quantisierung des Feldes einen Formalismus gefunden, in dem die Erhaltung der Teilchenzahl verletzt ist. Mein Feld beschreibt das erzeugen und vernichten von Teilchen. Ich kann also durchaus zwischendrin virtuelle Teilchen erzeugen und wieder vernichten, solange diese nicht direkt messbar sind (denn dann wären sie real und ich müsste ihnen Energie zur Verfügung stellen).

Aber zurück zum Casimireffekt.
Ich kann meine Nullpunktsenergie nicht Direkt messen. Die Energie hängt aber von den Randbedingungen des Feldes ab. Also müsste sich die Energiedifferenz zwischen Feldern unterschiedlicher Randbedingungen bemerkbar machen. Das ist genau was der Casimireffekt zeigt. Das ausrechnen des Effektes ist sehr technisch und relativ anspruchsvoll.

uwebus
23.04.08, 18:05
Querkopf,
----
Aber zurück zum Casimireffekt.
----
Ich will mich nicht in Eure Betrachtungen vertiefen, da bei mir das Vakuum eine Aufgabe im Universum erfüllt, nämlich die, Gravitation (actio) zu erzeugen und damit überhaupt erst EM-Felder (reactio) zu ermöglichen. Das läuft bei Physikern erfahrungsgemäß unter Blödsinn. Aber beim o.a. Effekt kommt mein “Blödsinn-Modell“ wieder zu einer ziemlich überraschenden Übereinstimmung mit dem empirischen Wert der Casimir-Kraft, schau doch mal nach unter http://uwebus.de/rzg6/023.htm “Adsorptionseffekt bei Metalloberflächen“. Ich will Euch ja nicht von Euren Theorien abbringen, aber irgendwie müßte es Euch doch stutzig werden lassen, wenn man mit einem endlichen Feldmodell so viele unterschiedliche Übereinstimmungen mit den empirischen Werten der Physik erzielt.

Querkopf
23.04.08, 18:40
Ich durfte mich mal mit Elektronen an den Oberfläche von Edelmetallen befassen. Die Jungs machen ziemlich krasse Sachen (z.B. Confinement), aber mit deinem Modell hat das überhaupt nichts zu tun. Und wenn ich eine Metalloberfläche poliere sieht das Mikroskopisch immer noch aus wie eine Kraterlandschaft, die außerdem noch eine starke Dynamik hat.
Und der Casimireffekt tritt nicht nur zwischen zwei Metallplatten auf, sondern z.B. auch zwischen Einem BEC und einer Glassplatte (also einem Quantenkondensat und einer viskosen Flüssigkeit)

ingeniosus
23.04.08, 18:54
Wenn ich ein wasserstoffähnliches Atom betrachte, dann ist das Problem zentralsymmetrisch, also Isotrop und daher gilt Drehimpulserhaltung.

Und nun langsam -- ohne Emotion : zentralsymmetrisch ist für mich kreisförmig oder denke ich chinesisch? Drehimpulserhaltung heisst für mich : es dreht sich etwas um einen Drehpunkt !!!

Es ist schon klar dass die Planetenbewegung seit Kepler genau berechnet ist und die Bahnen keine Kreise sind, aber dafür kann man ja auch die Summe der universellen Massekräfte als verantwortlich sehen, oder?

Es geht mir nur um die naivistische (= nicht naiv) Betrachtungsweise der analogen Bewegung der Planeten um die Sonne oder des Mondes um die Erde und diejenige der Elektronen um den Atomkern (nach Nils Bohr).

Physik ist eine beobachtende Wissenschaft, die Mathematik macht sie zu kleinlich!

Das Planksche Wirkungsquantum wurde nicht errechnet, sondern physikalisch festgestellt!

ingeniosus
23.04.08, 19:06
Ich durfte mich mal mit Elektronen an den Oberfläche von Edelmetallen befassen. Die Jungs machen ziemlich krasse Sachen (z.B. Confinement), aber mit deinem Modell hat das überhaupt nichts zu tun. Und wenn ich eine Metalloberfläche poliere sieht das Mikroskopisch immer noch aus wie eine Kraterlandschaft, die außerdem noch eine starke Dynamik hat.


OK, dann habt Ihr damals aber eine Oberfläche betrachtet, ich meine die Struktur innerhalb eines Körpers (Metalles).

Irgendwo habe ich gelesen, dass das BOHRsche Atommodell (1913) nicht ganz so einfach ist, wie wir es noch Ende 60er Jahre gelernt haben. Die wesentlichen Bewegungen innerhalb eines einigermassen homogenen Körpers sind aber schon so, oder gilt das nicht mehr?

ingeniosus
23.04.08, 19:25
Und der Casimireffekt tritt nicht nur zwischen zwei Metallplatten auf, sondern z.B. auch zwischen Einem BEC und einer Glassplatte (also einem Quantenkondensat und einer viskosen Flüssigkeit)

Der Casimireffekt wurde vor kurzem im Fernsehen als das neueste Forschungsresultat deutscher Physiker dargestellt, ist aber doch schon in den 50er Jahren entstanden. Die "Virtuellen Teilchen" sind ja ein echter Hammer, heute meint man doch zumindest Fermionen oder Bosonen....

Hoch interessant finde ich auch etwa die Verschränkung bei Photonen oder deren Teilchen, die ja bekanntlich von einem Wiener od. genauer Tiroler wiedergefunden und genauer erforscht wurde und wird.

ingeniosus
23.04.08, 19:52
Ein Vakuumzustand ist nur der Grundzustand eines Systems in der Vielteilchenquantenmechanik bzw. der Quantenfeldtheorie.
Dieser Vakuumzustand, kann durchaus ein Kollektiv von Elektronen Atomrümpfen oder Spins sein, also alles andere als ein leerer Raum.

..... und später....
Wir haben unseren Vakuumzustand (der Grundzustand unseres quantisierten Elektromagnetischen Feldes um Missverständnissen zu vermeiden) und unser Maxwellfeld.


Zunächst meinen Respekt vor Ihrem komplexen und umfangreichen Fachwissen - Sie scheinen also ein echter Quantenphysiker zu sein..

Ich bin ein studierter Elektroingenieur, der immer in der Informatik tätig war und nun seinen messerscharfen Struktur- und Systemverstand auf die Naturwissenschaft loslässt....


Also ich kombiniere:

Es bleibt beim Objekt Vakuumzustand: Es ist ein Zustand eines Quantenfeldes und elektromagnetischen Feldes, das quantisiert ist. Das wäre dann ganz grob also gleichzusetzen.

Mathematisch wäre dann äquivalent Quantenfeldtheorie und quantisierte Maxwellfeldtheorie (, die für mich aus der Vektorrechnung folgt)

Wo stecken dann dabei die Schwache und Starke Wechselwirkung? Sind die im "Quantenfeld"?

uwebus
24.04.08, 13:16
Querkopf,
-----
..aber mit deinem Modell hat das überhaupt nichts zu tun. Und wenn ich eine Metalloberfläche poliere sieht das Mikroskopisch immer noch aus wie eine Kraterlandschaft, die außerdem noch eine starke Dynamik hat.
----
Das glaube ich Dir ja. Wesentlich ist jedoch nicht die von mir dargestellte Symmetrie, sondern die Kontaktfläche und die dürfte in etwa zutreffen.

----
Und der Casimireffekt tritt nicht nur zwischen zwei Metallplatten auf, sondern z.B. auch zwischen Einem BEC und einer Glassplatte (also einem Quantenkondensat und einer viskosen Flüssigkeit)
----
Das mag ja ebenfalls zutreffen, aber das heißt doch nicht, daß nicht auch hier freie Elektronen eine Rolle spielen können. Ich habe etwas gegen “Verbote“ in der Natur, denn Verbote sind eine menschliche Erfindung.

http://de.wikipedia.org/wiki/Casimir-Effekt
“Damit sind zwischen den Platten bestimmte Zustände virtueller Teilchen verboten, die außerhalb angenommen werden können. Alle erlaubten virtuellen Teilchen werden aber an den Platten reflektiert.“

Was mich eben wundert sind die Werte, die sich da ergeben zwischen Modell und Empirie; querbeet durch diverse Gebiete der Physik komme ich zu zahlenmäßigen Übereinstimmungen mit deren empirischen Werten, von der Gravitation über die Coulombkraft zu den Atom- und Molekülgrößen einschließlich Bindungsabständen, Planck-Konstante, c=(fast)konstant, Zeitdilatation etc., und das alles mit einer einzigen Annahme, wobei ich sogar noch Erklärungen liefere, die in der Physik noch ausstehen.

Ich nehme an, daß irgendwann mal ein Physiker, der auch ein wenig philosophisch denkt, sich zu fragen beginnt, was eigentlich der Raum zwischen den Gestirnen ist, welche Funktion er ausübt, und dann wird in der Physik ein Umdenken einsetzen. Solange jedoch das Vakuum lediglich als Aufbewahrungsbehälter für Galaxien betrachtet wird, solange werden die Urknallideologen die Weltsicht bestimmen. Bezüglich der Zeit hat ja inzwischen ein Umdenken begonnen, für den Raum steht dies noch aus. Als ich vor Jahren Zeit als psychisches und nicht physisches Phänomen an den Mann zu bringen versuchte, bin ich nur als Blödel abgestempelt worden, heute habe ich zumindest hier eine Art Wiedergutmachung erfahren (bild der wissenschaft, Ausgabe 1/2008). Vielleicht erlebe ich es ja auch noch, daß auch ein endliches Feldmodell in Physikerhirne Einzug hält, dann hören diese absurden Unendlichkeiten endlicher Größen in der Physik auf.

Ich kann es nicht beweisen, aber ich bin sicher recht zu haben mit der Aussage:
Eine endliche Energiemenge kann nur endliche Werte in Ausdehnung und Dichte annehmen.
Daraus folgt ein Energiequantenfeldmodell und daß ein solches prinzipiell funktioniert führe ich mit meiner HP vor. Nicht mehr und nicht weniger.

Gruß

Uranor
24.04.08, 14:54
Und der Casimireffekt tritt nicht nur zwischen zwei Metallplatten auf, sondern z.B. auch zwischen Einem BEC und einer Glassplatte (also einem Quantenkondensat und einer viskosen Flüssigkeit)
----
Das mag ja ebenfalls zutreffen, aber das heißt doch nicht, daß nicht auch hier freie Elektronen eine Rolle spielen können. Ich habe etwas gegen “Verbote“ in der Natur, denn Verbote sind eine menschliche Erfindung.
salve und sorry,

an genügend Stellen ist es sehr schwer, nur gespannt zuzuhören. Das mit den Valenzelektronen für die Glasplatte hätte jetzt aber wirklich nicht kommen dürfen. @uwebus, wer noch derart krasse Fehler bei im Grunde schülerbekanntem einbaut, sollte wirklich nicht mal über unseren lokalen Fensterputzer Spottelegien verkünden geschweige denn über den Glasmonteur, der nicht mal bei größten Schaufensterscheiben auf die Idee käme, sie zu erden. ;) Angemessen wäre, seinen Stoff zu lernen. Man sollte beurteilen können, worüber man spricht.

Gruß Uranor

orca
24.04.08, 15:01
Ich ...., der ...... nun seinen messerscharfen Struktur- und Systemverstand auf die Naturwissenschaft loslässt....



Ist das ironisch gemeint?:rolleyes:

Ich vermute nicht!:D

Querkopf
24.04.08, 15:37
Und nun langsam -- ohne Emotion : zentralsymmetrisch ist für mich kreisförmig oder denke ich chinesisch?
Ein Quadrat ist ein Rechteck, aber nicht jedes Rechteck ein Quadrat.
Ein Kreis ist Zentralsymmetrisch. Aber nicht jedes zentralsymmetrische Problem hat eine Kreisförmige Lösung. Ich habe auch gewisse Probleme mir bei einem Zentralsymmetrischen Problem sagen wir in 10 räumlichen Dimensionen Kreise vorzustellen.
Und chinesisches Denken ist kein Problem, weil für mich Alltag.

Es ist schon klar dass die Planetenbewegung seit Kepler genau berechnet ist und die Bahnen keine Kreise sind, aber dafür kann man ja auch die Summe der universellen Massekräfte als verantwortlich sehen, oder?
Für mich sind das Einfach mögliche Lösungen der klassischen Bewegungsgleichungen für ein solches Zentralsymmetrisches Problem.


Es geht mir nur um die naivistische (= nicht naiv) Betrachtungsweise der analogen Bewegung der Planeten um die Sonne oder des Mondes um die Erde und diejenige der Elektronen um den Atomkern (nach Nils Bohr).
Nun ist aber das Bohrsche Modell schon seit mindestens 80 – 90 Jahren theoretisch überholt und bereits zu der Zeit als es postuliert wurde im Widerspruch zum Experiment.
Es war ein Übergangsmodell mit extrem kurzer Lebenszeit, das eigentlich zu keinem Zeitpunkt wirklich wissenschaftlich Etabliert war. Es war ein Aufbruchszeichen. Bohr hat gezeigt, dass im Mikroskopischen Bereich eine andere Physik gilt und dass diese beschreibbar ist. Daraufhin sind alle jungen talentierten Physiker zu Bohr geströmt und innerhalb weniger Jahre (1925) war die Quantenmechanik aus dem Boden gestampft. Damit war das alte Modell natürlich passe.
Es hat sich aber in der Schule erstaunlich lange gehalten und es soll immer noch Lehrer geben die diesen Dinosaurier unterrichten.

Physik ist eine beobachtende Wissenschaft, die Mathematik macht sie zu kleinlich!
Das Planksche Wirkungsquantum wurde nicht errechnet, sondern physikalisch festgestellt!
Streng genommen wurde es von einem der Mathematischen Physik sehr zugewandten Max Planck einfach postuliert um die Experimente zu erklären, nach dem die Experimentelle Zunft dem Problem jahrelang ziemlich Hilflos gegenüber stand.


OK, dann habt Ihr damals aber eine Oberfläche betrachtet, ich meine die Struktur innerhalb eines Körpers (Metalles).
Uwebus hatte einige sehr unrealistische Annahmen zu Elektronen an Oberflächen von Metallen gemacht.
Die Oberflächen bzw. Grenzflächenphysik ist ein Eigenständiges Gebiet der Festkörperphysik und das Verhalten von Oberflächen unterscheidet sich z.T. sehr deutlich vom inneren des Festkörpers.

Irgendwo habe ich gelesen, dass das BOHRsche Atommodell (1913) nicht ganz so einfach ist, wie wir es noch Ende 60er Jahre gelernt haben. Die wesentlichen Bewegungen innerhalb eines einigermassen homogenen Körpers sind aber schon so, oder gilt das nicht mehr?
Das Bohrsche Modell war auch in den 60er Jahren sehr lange als falsch bekannt.
Ich habe in einem Festkörper eine sehr hohe Anzahl von Freiheitsgraden (weil ich in der Größenordnung immer bei 10^23 Teilchen bin)
Ich habe also massive Wechselwirkungen zwischen meinen Teilchen, weshalb ich keine Einzelnen Teilchen mehr beschreibe, sondern ein kollektiv.
Das ist eine typische Aufgabe der Quantenfeldtheorie

Ein Einfaches Modell für ein Quasifreies Elektron in einem Festkörper, währe es ein periodisches Potential anzunehmen. Wenn ich mir die allgemeine Struktur von Lösungen der Schrödingergleichung (also nichtrelativistisch ohne Spin) in Periodischen Potentialen anschaue, dann bekomme ich die z.B. aus der Halbleiterphysik bekannte Bandstruktur für die Energieeigenwerte, also Bereiche mit Quasikontinuierlichen Lösungen, unterbrochen durch Energiebereiche ohne Lösung. Die genaue Struktur hängt dann vom Potential ab.

Das ist aber eine starke Vereinfachung. Ich habe nur ein Elektron (bzw. viel nicht Wechselwirkende) und mein Gitter ist bloß ein starrer Potentialhintergrund.

Ich kann also anfangen, Gitterschwingungen zu betrachten. Die Einfachste Möglichkeit das zu tun, ist die Annahme eines Masse Federmodels, also einer Matraze. Ich habe also ein Feld von Massen die in guter Näherung durch harmonische Oszillatoren Wechselwirken. Jede Menge harmonische Oszillatoren? Das ist bekannt, ihre Quantisierung führt auf eine Quantenfeldtheorie für Gitterschwingungen. Unser Feld erzeugt und vernichtet Schallteilchen. Unser Vakuum ist kein leerer Raum, sondern der Grundzustand unserer Matratze. Meine Teilchen, eine Art massive Photonen sind Schallquanten, Phononen. Ich habe nur nach Schema F mein Programm durchgezogen und kann nun den ganzen Apparat der QFT anwenden.
Nun kann man einwenden, dass das mit den harmonischen Oszillatoren ja gemogelt war. Vielleicht ist die Wechselwirkung zwischen unseren Gitteratomen ja eine andere.
Da kommt uns wieder unsere Quantenfeldtheorie zur Hilfe. Wir können unsere Abweichung in der Wechselwirkung nämlich als Wechselwirkungsterm in die Störungsrechnung mit einbringen. Im Gegensatz zu Photonen sind unsere Phononen dann selbstwechselwirkend.

Ich habe also in meinem Diagram ein Dreibein, das die Wechselwirkung zwischen drei Phononen beschreibt. Z.B. kann ich zwei Niederenergetische Phononen erzeugen und dafür ein hochenergetisches vernichten.

Zurück zu unseren Elektronen. Die Elektronen die uns für gewöhnlich (z.B. in der Halbleiterphysik interessieren) sind die Elektronen im Leitungsband. Um ein Elektron ins Leitungsband zu heben muss ich ihm die nötige Energie zur Verfügung stellen um die Bandlücke zwischen seinem gefüllten Band (in dem es sich nicht bewegen und daher auch nicht Strom leiten kann) und dem Leitungsband zu überwinden. In dem ursprünglichen Band bleibt eine Lücke zurück, ein Loch, das sich wie eine Positive Ladung verhält.
Ich erzeuge also im Grunde genommen ein Teilchen – Antiteilchen Paar, ein Elektron Loch Paar.
Mein Elektron – Loch Feld wechselwirkt mit dem Maxwellfeld (also mit Photonen; Solarzelle, Leuchtdiode, ...) aber auch mit dem Phononenfeld.
Ich habe also eine Art QED mit einem zusätzlichen Phononenfeld, das nicht nur mit den anderen Feldern, sondern auch mit sich selbst wechselwirkt.
Wenn meine Elektronen stark an die rümpfe gebunden sind, dann können sie nur noch über ihren Spin Wechselwirken und ich betrachte ein Spinproblem, das z.B. Magnetisierung erklärt (das kann ich mit statistischer Physik beschreiben, aber auch als Quantenfeldtheorie).
...

Beliebig kompliziertere Modelle sind denkbar, z.B. Selbstwechselwirkende Elektronen, kompliziertere Gitterstrukturen, neue Quasiteilchen wie Cooperpaare, ...
Etwa 50% aller Physiker machen Festkörperphysik. Entsprechend viel Material gibt es sowohl theoretisch als auch experimentell (es stecken schließlich auch massive wirtschaftliche Interessen dahinter, was in der Teilchenphysik weniger der Fall ist).

Das Bohr’sche Modell scheitert schon am Atomaren Wasserstoff. Einfache Emissions - und Absorptionsspektren liefert es noch, aber schon bei einer lange vor Erstellung des Modells bekannten Niveauaufsplittung durch äußere Felder (z.B. Peter Zeeman 1896 in Leiden) versagt es. Wasserstoff kommt üblicherweise in Molekularer Form vor. Das Bohr’sche Modell macht keine Aussagen zur Molekülbildung, von höheren Ordnungszahlen und der Bildung von Festkörpern einmal ganz zu schweigen.

Es bleibt beim Objekt Vakuumzustand: Es ist ein Zustand eines Quantenfeldes und elektromagnetischen Feldes, das quantisiert ist. Das wäre dann ganz grob also gleichzusetzen.
Das war vielleicht etwas ungenau ausgedrückt. Der Vakuumzustand ist der Grundzustand eines Quantisierten Feldes.
In der QED ist das Elektromagnetische Feld ein Quantisiertes Feld.Es hat also einen Grundzustand, einen Vakuumzustand.
Andere Quantisierte Felder z.B. ein Photonenfeld haben auch einen Grundzustand.
Die Grundzustände sind andere, da ich andere Grundmoden habe.
Zur Unterscheidung kann ich z.B. von einem Photonischen Vakuum (also den Grundzustand meines Elektromagnetischen Feldes) reden.

Es hängt also davon ab, welches Modell ich betrachte. Gitterschwingungen in einem Festkörper werden nicht merklich durch sie starke Wechselwirkung oder die Gravitation beeinflusst. In meiner Lagrangedichte oder meiner Hamiltondichte brauche ich diese Felder also nicht zu berücksichtigen.

In meinem Spielzeugmodell für den Casimireffekt war die Elektromagnetische Kraft die bedeutende (wie mehr oder weniger für alle Prozesse auf diesem Planeten, also z.B. alle chemischen Bindungen und Reaktionen und damit alles Leben).
Die Gravitation ist sehr schwach, die schwache Wechselwirkung ebenfalls und nur an wenigen lokalen Prozessen beteiligt (denn ihre Austauschteilchen sind sehr massiv und haben eine kurze Lebensdauer). Sonst hätte man diese Wechselwirkung nicht erst so spät entdeckt.
Die Starke Wechselwirkung ist hat Selbstwechselwirkende Ausstauschteilchen und ist daher sehr kurzreichweitig (es hat auch noch niemand ein Gluon gesehen, im Gegensatz zu Photonen oder W und Z – Bosonen) und freie Quarks gibt es nur kurzfristig bei extrem hohen Energien.

Das ist der Grund warum unsere Physik vor Endeckung der starken und schwachen kraft schon sehr genau war. Wir haben eine sehr starke Separation von Längen - , Zeit – und Energieskalen.
Ich brauche für einen Billiardstoss kein Standardmodell. Ich brauche die Mechanik als universellen Apparat zur Beschreibung von Bewegungen aufgrund von Kräften. Und meine Kraft ist die Elektromagnetische, die ich aber nur global z.B. als Reibungsterm betrachte.

Querkopf
24.04.08, 15:37
Das mag ja ebenfalls zutreffen, aber das heißt doch nicht, daß nicht auch hier freie Elektronen eine Rolle spielen können. Ich habe etwas gegen “Verbote“ in der Natur, denn Verbote sind eine menschliche Erfindung.
http://de.wikipedia.org/wiki/Casimir-Effekt
“Damit sind zwischen den Platten bestimmte Zustände virtueller Teilchen verboten, die außerhalb angenommen werden können. Alle erlaubten virtuellen Teilchen werden aber an den Platten reflektiert.“
Da stellt sich keiner hin und sagt den Teilchen das sie nicht erlaubt sind, sondern aufgrund der Randbedingungen ergeben sich innerhalb des Kondensators (wobei man in der Praxis Kugeln und keine Plattenkondensatoren hat, ist allerdings schwieriger zu rechen) andere Lösungen als außerhalb. Der deutschsprachige Wiki Artikel ist halt einfach nicht besonders.


Ich nehme an, daß irgendwann mal ein Physiker, der auch ein wenig philosophisch denkt, sich zu fragen beginnt, was eigentlich der Raum zwischen den Gestirnen ist, welche Funktion er ausübt, und dann wird in der Physik ein Umdenken einsetzen. Solange jedoch das Vakuum lediglich als Aufbewahrungsbehälter für Galaxien betrachtet wird, solange werden die Urknallideologen die Weltsicht bestimmen.
Du überschätzt die Bedeutung und das Interesse der Physikergemeinde für Kosmologie und Astrophysik. Wenn ich mich für den Phasenübergang eines Suprafluids interessiere, dann ist mir der Raum zwischen den Sternen herzlich egal (trotzdem habe ich einen Vakuumzustand).

Was mich eben wundert sind die Werte, die sich da ergeben zwischen Modell und Empirie; querbeet durch diverse Gebiete der Physik komme ich zu zahlenmäßigen Übereinstimmungen mit deren empirischen Werten, von der Gravitation über die Coulombkraft zu den Atom- und Molekülgrößen einschließlich Bindungsabständen, Planck-Konstante, c=(fast)konstant, Zeitdilatation etc., und das alles mit einer einzigen Annahme, wobei ich sogar noch Erklärungen liefere, die in der Physik noch ausstehen.
Dann schau dir doch mal deine Zahlen an. Nehmen wir einmal an, die von dir angegebenen Zahlen für den Casimireffekt wären richtig.
Experimentell ist das schwierig zu messen, man hat etwa 5%, im klassischen Fall (also zwei Kondensatorplatten) etwa 15% Abweichung zwischen Theorie und Experiment.
Dein Bestes Ergebnis (Titan) weicht 15 Prozent von der Theorie ab. Ok, dass klingt nicht schlecht. Dein zweitbestes Ergebnis ist Vanadium mit 29% Abweichung.

Schauen wir uns aber einmal klassische Materialien im Mikroskopische Bereich an (z.B. in der Halbleiterindustrie) Al (früher die Leiterbahnen auf Prozessoren), Cu (heute das Material der Wahl) und Gold (für Spezialanwendungen, Raumfahrt und Forschung).
Cu: 69% Abweichung
Al: fast 500% Abweichung
Au: 760% Abweichung

Gold und Aluminium wurden übrigens auch in den bisher besten Experimenten genutzt: http://www.casimir.rl.ac.uk/measurements.htm

Nun, ich bin nicht wirklich überzeugt. Dazu kommen haarsträubende annahmen zur Oberflächenstruktur von Festkörpern und ich verstehe immer noch nicht, wie du in einem symmetrischen System eine anziehende Kraft durch die Wechselwirkung gleichgeladener Ladungsträger erhältst. Vielleicht solltest du deine Rechnungen einmal offenlegen (und dabei eine Formeleditor verwenden, denn deine Gleichungen sind nahezu unlesbar).

Ich kann es nicht beweisen, aber ich bin sicher recht zu haben mit der Aussage:
Eine endliche Energiemenge kann nur endliche Werte in Ausdehnung und Dichte annehmen.
Ich bin mir sicher es gibt grüne Männchen auf dem Mars, ich kann’s bloß nicht beweisen.
Ich musste im ersten Semester fast jede Woche beweisen (und zwar wirklich mathematisch beweisen) das eine unendliche Reihe gegen einen endlichen wert konvergiert. Und entsprechendes macht man dann später auch für Integrale.
Nimm eine homogen geladene Kugel. Errechne die Energie des äußeren Feldes durch Integration bis ins unendliche und du wirst immer eine endliche Energiedichte und eine endliche Energie erhalten, allerdings bei unendlicher Ausdehnung des Feldes.

rafiti
24.04.08, 16:36
..Vielleicht erlebe ich es ja auch noch, daß auch ein endliches Feldmodell in Physikerhirne Einzug hält, dann hören diese absurden Unendlichkeiten endlicher Größen in der Physik auf.

Ich kann es nicht beweisen, aber ich bin sicher recht zu haben mit der Aussage:
Eine endliche Energiemenge kann nur endliche Werte in Ausdehnung und Dichte annehmen.
Daraus folgt ein Energiequantenfeldmodell und daß ein solches prinzipiell funktioniert führe ich mit meiner HP vor. Nicht mehr und nicht weniger.

Gruß

Vielleicht erlebe ich ja dann auch, dass Melonen von ganz alleine aufs LKW steigen.
Dein Gesülze da immer...


gruss
rafti

uwebus
24.04.08, 19:00
Uranor,

Du brauchst doch gar nicht gespannt zuzuhören, laß einfach meine Beiträge links liegen. Wenn zwischen BEC und Glasplatte der Casimireffekt zu beobachten ist, dann müssen die Elektronen ja nicht auf der Glasplatte hocken, sondern können auch zum BEC gehören. Soviel ich weiß, ist ein BEC nicht frei von Elektronen.

http://www.kworkquark.de/lexikon/lexikon.materieteilchen/2/index.html

Ein Bose-Einstein-Kondensat ist eine Ansammlung von Teilchen, die sich alle in einem Zustand geringster Energie befinden.

Bose-Einstein-Kondensate sind nur mit Bosonen möglich, weil sich Fermionen aufgrund des Pauli-Verbots nicht im selben Zustand befinden dürfen.

Bosonen sind alle Teilchen mit ganzzahligem Spin (1, 2, ...).

Pauli-Verbot:
Für Fermionen gilt das Pauli-Verbot: Danach können zwei Fermionen niemals im selben Zustand verweilen.

http://de.wikipedia.org/wiki/Fermion
Unter den Elementarteilchen gehören zu den Fermionen die Leptonen (z. B. das Elektron und das Neutrino) und die Quarks.

Nun wird ein BEK (BEC=BEK) aus Atomen hergestellt und die bestehen aus Fermionen (Elektronen und Quarks), also müssen Fermionen in Bosonen umwandelbar sein und umgekehrt, oder meinst Du, ein BEK hat weder Elektronen noch Quarks? Dann bliebe nämlich nichts mehr übrig außer sog. Wechselwirkungsteilchen, die bis heute mit Ausnahme von Photonen noch niemand nachgewiesen hat (oder hat schon mal jemand Gluonen oder Gravitonen nachgewiesen?).

http://de.wikipedia.org/wiki/Gluon
Gluonen sind elektrisch neutral und werden innerhalb des Standardmodells als masselos angenommen, während experimentell eine Masse von einigen MeV nicht ausgeschlossen werden kann.

Ihr könnt machen was Ihr wollt, solange Ihr den Spin und die Gravitation nicht erklären könnt, solange habt Ihr keine Ahnung, was Bosonen oder Fermionen sind, Ihr meßt etwas, von dem Ihr nicht wißt, was es ist und stellt Theorien auf, die reine Annahmen sind.

Mein Modell beruht auch auf Annahmen, aber auf sehr wenigen, während die Physik einen ganzen Sack voll davon braucht, für jedes Fachgebiet andere. Daß auch bei mir Irrtümer eingeschlossen sind, ist selbstverständlich, aber das enthebt Euch nicht der Aufgabe darüber nachzudenken, wie und warum die Ergebnisse zustande kommen.

Teilchen, und zwar ALLE, sind die EM-Zentren endlicher Felder, sie unterscheiden sich nur durch ihren Spin und ihre Größe. In einem BEK ist es anscheinend möglich, alle beteiligten Teilchen zu einer Einheit zu verbinden derart, daß man ihre individuelle Dynamik so weit zurückfährt, daß sie zusammen in einen gleichen Grundzustand fallen. Nimm eine Geige und bring alle Obertöne auf dieselbe Frequenz, dann bekommst Du so etwas wie eine reine Sinusschwingung, ähnliches dürfte in einem BEK passieren.

Uranor, wenn Du das Universum zu verstehen suchst, mußt Du bei Dir selbst zu suchen anfangen, d.h. als erstes mußt Du Dir darüber klar werden, was Raum und Zeit sind und wie sie zustande kommen. Solange Du da nicht weiterkommst, brauchst Du Dir über die Physis keine weiteren Gedanken zu machen, Du kannst dann nur Beobachtungen beschreiben ohne Vorstellung dessen, was Du beschreibst. Damit kann man dann zwar Handys bauen oder zum Mond fliegen, aber was Du selbst bist, davon hast Du keine Ahnung. Und mich interessiert nun mal, was ich bin und nicht, ob ein Handy Internetzugang ermöglicht. Darin unterscheiden wir uns offensichtlich.


Querkopf,
----
Du überschätzt die Bedeutung und das Interesse der Physikergemeinde für Kosmologie und Astrophysik.
.....
und ich verstehe immer noch nicht, wie du in einem symmetrischen System eine anziehende Kraft durch die Wechselwirkung gleichgeladener Ladungsträger erhältst.
----
Und Du mißverstehst meine Überlegungen. Ich will wissen, was das Universum ist und warum ich existiere, da spielt es keine Rolle, ob ich mich im Kleinen oder im Großen bewege, das Ganze ist entscheidend.

Alle Wissenschaft beruht auf Bewußtsein, ohne letzteres keine Realität. Und nun versuche ich das Prinzip zu ermitteln, welches diese Realität ermöglicht, dazu muß ich als erstes die Phänomene Raum und Zeit zu verstehen suchen, ihr Zustandekommen und ihr Wechselwirken. Dies gelingt scheinbar mit dem Energiequantenfeldmodell, und zwar empirieverträglich in mehreren Bereichen.

Und zur Anziehung: Es gibt m.E. keine Anziehung, alle Kräfte entstehen aus dem Raum heraus, ein Kraftvektor ist lediglich eine Resultierende von Feldwirkungen und Felder wechselwirken “flächig“, also drückend, Felder können nicht ziehen. Wir hier auf der Erdoberfläche befinden uns im Gleichgewicht zwischen EM-Felddruck und Vakuumdruck, wir werden nicht angezogen, sondern zur Masse hin gedrückt von dem unserem Körper zuzuordnenden G-Feldanteil der Erde. Aber solange Physiker sich um die Frage nach Raum und Zeit drücken und Vakuum lediglich als Aufbewahrungsbehälter betrachten, solange bleibt Physik Stückwerk. Erst wenn Ihr in der Lage seid, die Entstehung der Dynamik im Universum zu erklären, werdet Ihr verstehen, was Teilchen sind, bis dahin werdet Ihr sie nur weiterhin unverstanden messen.

Ob ich nun richtig liege mit der Annahme nur einer einzigen metaphysischen, das Universum konstituierenden Substanz und eines einzigen Wirkprinzips sei dahingestellt, aber bisher scheint die Annahme ausreichend zu sein, um die Grundphänomene empirieverträglich erklären zu können.

----
Ich musste im ersten Semester fast jede Woche beweisen (und zwar wirklich mathematisch beweisen) das eine unendliche Reihe gegen einen endlichen wert konvergiert.
----
Ich auch. Aber das beweist nichts. Ein Feld mit einer vom Feldzentrum nach außen abnehmenden Energiedichte größer Null ist nun mal endlich, wenn der Energiegehalt endlich ist. E(Feld) = Integral [(EDr > 0)·r²·4·Pi·dr] ergibt nun mal bei E(endlich) ein endliches r. Ganz ohne komplizierte Mathematik. Allerdings beruht dieses Modell auf meinen Annahmen, welche das Universum als aus Energiefeldquanten aufgebaut ansehen. Ihr mit Eurem Riemannraum und feldfreiem Vakuum könnt ja etwas anderes annehmen, aber Eure Grundannahmen könnt Ihr halt empirisch nicht belegen, während ich mit Sicherheit davon ausgehen kann, daß es im beobachtbaren Universum kein feldfreies Vakuum gibt.

Gruß

rafiti
24.04.08, 20:15
@uwebus
Ich glaube dein "Fehler" ist, dass du manche Sachverhalte unterschätzst bzw. ignorierst. Das beweist nichts, jenes auch nicht, usw. Nimm mal die QT her, die scheint ja gar total daneben zu sein, was unsere Empirieerfahrungen betrifft, wieso sollte da nichts "widersprüchliches" bei rauskommen? Wie unendlich über endlich...

gruss
rafiti

Uranor
24.04.08, 21:21
Du brauchst doch gar nicht gespannt zuzuhören, laß einfach meine Beiträge links liegen. Wenn zwischen BEC und Glasplatte der Casimireffekt zu beobachten ist, dann müssen die Elektronen ja nicht auf der Glasplatte hocken, sondern können auch zum BEC gehören. Soviel ich weiß, ist ein BEC nicht frei von Elektronen.
salve uwebus,

gehst du jetzt schon dazu über, die Themeninteressierten da wegzukomplementieren, damit du deinen gebündelten Unsinn unentdeckt schreiben kannst? Wozu schreibst du das denn überhaupt? Was haben freie Elektronen mit dem Casimir-Efekt zu tun? Soll man sich nicht bissele auskennen, bevor man überhaupt was schreibt? Und dann die regelmäßigen Selbstbeweihräucherungen, wie toll ausgerechnet deine Märchenzusamenstellungen funktionieren. Es war noch in keiner Armee befohlen, dass sich Grenadire zu Cowboyclowns nachen sollen.

Lass dir's mal durch den Kopf gehen.
Uranor

ingeniosus
25.04.08, 17:55
Ist das ironisch gemeint?:rolleyes:

Ich vermute nicht!:D

ORKA, darf ich Ihnen eine Rüge erteilen! Dick- und Doof-Manieren sind hier nicht ganz am Platze!
Schade um den Speicherplatz !!!!

uwebus
25.04.08, 17:56
Uranor,
----
Was haben freie Elektronen mit dem Casimir-Efekt zu tun?
----
Das weiß ich nicht, genauso wenig wie ich weiß, ob die Theorie mit den virtuellen Teilchen richtig ist. Ich weiß nur, daß im Makrobereich der Adsorptionseffekt mit angelagerten Gasmolekülen funktioniert und daß ein ähnliches Phänomen auch im Vakuum mit freien angelagerten Elektronen denkbar ist. Und rechnerisch kommt ja merkwürdigerweise ein ähnlicher Wert heraus.

Dein Problem ist, daß Du Physikbücher als Bibel betrachtest und alles darin Niedergeschriebene einem Zeugen Jehovas ähnlich für bewiesen ansiehst. Gerade Dir empfehle ich, Dir mal Gedanken darüber zu machen, was Realität eigentlich ist, wie und wo sie erzeugt wird und was dazu an physischen Voraussetzungen erforderlich ist.

Und wenn Du schon so allwissend daherkommst, erzähl doch mal der hier versammelten Gemeinde, wie Gravitation und wie die Dynamik im Universum entstehen und wie und aus was Vakuum sich konstituiert, einfach mal so als Einführungsvortrag. Ich mache hierzu Vorschläge, die untermauert sind mit einem Energiequantenfeldmodell, welches bis heute in dieser Form nicht existiert, von Dir habe ich bisher noch nicht eine einzige konstruktive eigene Idee gehört. Du solltest in das Forum
http://www.astronews.com/forum/index.php
wechseln, da sind mehrere Allwissende Deines Genres am Wirken.

Viel Spaß dort.

ingeniosus
25.04.08, 18:07
Ein Kreis ist Zentralsymmetrisch. Aber nicht jedes zentralsymmetrische Problem hat eine Kreisförmige Lösung. Ich habe auch gewisse Probleme mir bei einem Zentralsymmetrischen Problem sagen wir in 10 räumlichen Dimensionen Kreise vorzustellen.


Querkopf, ich wäre Ihnen dankbar, wenn Sie mir dann eine brauchbare Definition Ihres Attributes "zentralsymmetrisch" festlegen könnten.

Es zeigt sich, dass heutzutage das Zusammentreffen von Fachleuten nicht mehr durch langwierige "Einheitendiskussionen" eingeleitet werden, sondern es heutzutage eines Modell- und Begriffsabgleiches bedarf. Tempera mutantur!

Informatiker haben bei Physikern also doch eine echte Existenzberechtigung!

orca
25.04.08, 19:02
Ich ...., der ...... nun seinen messerscharfen Struktur- und Systemverstand auf die Naturwissenschaft loslässt....



Hab ich doch richtig vermutet, war tatsächlich ernst gemeint!:rolleyes:

Nun haben wir schon zwei richtige "Intelligenzbestien":rolleyes:,
Urinator und ingeniosus hier im Forum, das kann ja heiter werden.

MfG
Orca:D

Uranor
25.04.08, 19:03
Dein Problem ist, daß Du Physikbücher als Bibel betrachtest
Gewöhn dir endlich das seichte Gewäsch ab. Es wäre angemessen. Wie willst du deine Melonensuppe als schmackhaft verkaufen, wenn du für dich ständig und immer nur das gleiche Armutszeignis lallst? Merxt du denn nicht dass du ausgerechnet damit so :p (also so ulkig) wirkst?

Also, dein Problem ist es offenbar, dass andere weder die Bibel noch irgendwelchen Ersatz als Esoterikbasis anerkennen. Das hätt'ste halt gern, oder? Denn dann träfen deine ewig lauen Elegien sicher nur noch auf taube Melonen. :p

Gruß Uranor


NachPS: Oh ja, ich mag Physik. Da du sie erklärtermaßen ablehnst, kannst du es ja mal in einer Esoterik-Runder versuchen. Dort mag man meist noch unvergorenes, da es halt noch ohne Allehol ist. :p

Uranor
25.04.08, 19:12
Hab ich doch richtig vermutet, war tatsächlich ernst gemeint!:rolleyes:

Nun haben wir schon zwei richtige "Intelligenzbestien":rolleyes: ,
Urinator und ingeniosus hier im Forum!

MfG
Orca:D
Horch mal zu, du Orcpi.sser. Millionen Jahre habe ich solche absichtlichen Namensverunglimpfungen mit Humor weggesteckt, da ich nur spatzig kleine Hysteriker sah. Dich als gezielten Verunglimpfer fordere ich zum Düll!

criptically
25.04.08, 22:55
Casimir Effekt:
Der Quantenmechanische Harmonische Oszillator hat im Grundzustand eine nichtverschwindende Energie von ½ h-quer Omega. Also auch die Nullmode schwingt noch mit einer gewissen Energie.
...

Wie hoch ist diese Energie wenn Omega = Null ist?

ingeniosus
26.04.08, 21:16
Und zur Anziehung: Es gibt m.E. keine Anziehung, alle Kräfte entstehen aus dem Raum heraus, ein Kraftvektor ist lediglich eine Resultierende von Feldwirkungen und Felder wechselwirken “flächig“, also drückend, Felder können nicht ziehen. Wir hier auf der Erdoberfläche befinden uns im Gleichgewicht zwischen EM-Felddruck und Vakuumdruck, wir werden nicht angezogen, sondern zur Masse hin gedrückt von dem unserem Körper zuzuordnenden G-Feldanteil der Erde.

Uwebus,
es scheint mir ein wenig überspitzt zu sein, wenn Sie sich Gedanken machen, ob eine Feld (Kraft) zieht oder drückt.
In meiner Vorstellung setzt die Kraft im Schwerpunkt an. Das ist natürlich maskoskopisch-mechanisch gedacht, aber ein Feld zieht, wenn die Kraft von vorne (in der Bewegungsrichtung) kommt und es drückt, wenn ein Feld von hinten kommt. Ich würde sagen es wirkt ein, ob drückend oder ziehend ist nur eine Frage der Wirk-Oberfläche, -Richtung und Versuchsanodnung.

Die Feldtheorie hat ja den Nachteil, dass sie ihre Vorstellbarkeit verliert. Jedes Feld hat empirisch-physikalische Ursachen (Gravitation und Elektromagnetismus). Bei der elektroschwachen und starken Wechselwirkung habe ich derzeit noch echte Wissenslücken, daher wage ich noch keine Aussagen). Unter Masse oder Ladung kann ich mir einigermassen was vorstellen, aber ein Feld ist eben ein Abstraktum, sicherlich gedanklich hilfreich - aber doch.

ingeniosus
26.04.08, 22:22
Vielleicht erlebe ich es ja auch noch, daß auch ein endliches Feldmodell in Physikerhirne Einzug hält, dann hören diese absurden Unendlichkeiten endlicher Größen in der Physik auf.

Ich kann es nicht beweisen, aber ich bin sicher recht zu haben mit der Aussage:
Eine endliche Energiemenge kann nur endliche Werte in Ausdehnung und Dichte annehmen.
Daraus folgt ein Energiequantenfeldmodell und daß ein solches prinzipiell funktioniert führe ich mit meiner HP vor. Nicht mehr und nicht weniger.



uwebus,
Ihre Festlegung über die endliche Energiemenge ist völlig OK, nur die "Virtuellen Teilchen" im Casimir-Effect weisen auf eine Energiemenge hin, die nicht endlich ist. Es ist nicht mehr definitiv haltbar, dass nach den Fermionen nicht noch kleinere "Quantisierungen" existieren. Es entsteht also eine unendliche Energiequelle durch "logische Öffnung" wie zur vorletzten Jahrhundertwende eben die Atomenergie gefunden wurde.

Die Heisenbergsche Unschärferelation sagt ja nicht : bis zum Plankschen Wirkungsquantum und dann Schluss, sondern besagt nur : da ist eine Schwelle und keine Grenze!

ingeniosus
26.04.08, 22:41
Daraus folgt ein Energiequantenfeldmodell und daß ein solches prinzipiell funktioniert führe ich mit meiner HP vor. Nicht mehr und nicht weniger.

Ihr Energiequantenfeldmodell deckt sich mit meinem "Energonmodell". Nur habe ich mein Energon in seinem Energium noch nirgendwo berechnet. Es ist eben ein "Paradigma", wie Informatiker es nennen. Wir haben keine Scheu vor Immateriellem, wie die tradierten Philosophen, nur nicht ohne praktische Wirkung.

Aber offensichtlich kann man dann ein Energiequantum aus der QFT mit meinem "Energon" gleichsetzen. Das wäre für mich ein kleiner Erfolg!

Der logische Pfad ist aber schon insofern konsequent, als ich ein Energon herauslösen will, wie ein Molekül, Atom, Fermion usw. und versuche - gedanklich zumindest - sein "Energium= Summe aller einwirkenden Felder und Kräfte" zu isolieren.

An der Front der Physikalischen Forschung zeigt sich doch derzeit bereits eine neue Schwelle : das Verschwinden der Masse. Was bleibt dann? Passen würde sicherlich so über den Daumen: Energie.....

criptically
26.04.08, 23:00
...
An der Front der Physikalischen Forschung zeigt sich doch derzeit bereits eine neue Schwelle : das Verschwinden der Masse. Was bleibt dann? Passen würde sicherlich so über den Daumen: Energie.....

Heißt es nicht E=mc² bzw. m=E/c² ? :D :D :D

möbius
27.04.08, 04:05
Heißt es nicht E=mc² bzw. m=E/c² ? :D :D :D

Und heißt es nicht E = h f (Max Planck) - und
mc² = hf (Louis de Broglie) ???

Und m = hf/c² ???
:D :D :D
möbius

criptically
27.04.08, 09:19
Und heißt es nicht E = h f (Max Planck) - und
mc² = hf (Louis de Broglie) ???

Und m = hf/c² ???
:D :D :D
möbius

Und noch weiter, Gleichverteilungssatz: Gesamtenergie = Schwingungsenergie + Kinetische Energie + Potentielle Energie + Rotationsenergie +...

Und für Photonen Ekin = Eschwing = mc²/2 = hf/2 ,

und Egesamt = mc² = hf ? :D :D :D

uwebus
27.04.08, 12:41
Ich geb Euch mal wieder was zum Nachdenken oder auch einfach nur zum besserwisserischen Kritisieren:

xyz schrieb u.a.:
----
Die Wellenkinematik ist primär eine vorphysikalische Theorie,......

Wir bewegen uns hier auf dem Gebiet der vorphysikalischen Wellenkinematik.....

Eine Welle läuft sich nicht tot !!!
----
Das bedeutet, bei der “vorphysikalischen Wellenkinematik“ handelt es sich um “physisfreie Mathematik des leeren Raumes“!

----
Eine wirkliche Welle ist unumkehrbar und aufgrund der Phasenexistenz und Phasenwanderung als Folge der Ausbreitung einer Störung unzerlegbar. Alle störungsfreien Felder sind ausnahmslos Endprodukte ermattender Wellenquellen, also störungsfrei gewordener Felder.
-----

Ich bin der Auffassung, daß xyz hier einen GRUNDSÄTZLICHEN Fehler begeht, sofern wir vom Licht sprechen. Man kann die “vorphysikalische Wellenkinematik“, wie hier die Kinematik des leeren Raumes bezeichnet wird, nicht auf einen physischen Raum übertragen, und zwar aus folgendem Grund:

Von der Oberfläche einer Licht aussenden Sphäre in radialer Richtung ausgehend ändert sich die Feldgeometrie. Nun kann man den Faktor 1/r² als Raumkrümmung (Physik) oder als Raumspannung (mein Modell) bezeichnen, das Resultat bleibt das gleiche (ich ziehe Raumspannung im Sinne von Energiedichte vor, da Krümmung keine Begründung für Kraftentstehung ist, Spannung hingegen ja).

Eine Lichtpartikel erzeugt eine sie begleitende Welle, wenn man davon ausgeht, daß das Vakuum aus etwas besteht. Ab Austrittsfläche ins Vakuum entsteht also vor der Partikel Verdrängung, hinter ihr Rückfluß und damit Antrieb, die Partikel wird von ihrer Begleitwelle vorwärts getrieben. Da nun aufgrund der abnehmenden Energiedichte/Raumkrümmung der Widerstand abnimmt (das Vakuum “entspannt“ sich mit zunehmendem r), wird die Partikel beschleunigt so wie ein Gasbläschen, welches in einer Flüssigkeit aufsteigt.

Beschleunigung ist gleichbedeutend mit einem Kräftepaar actio=reactio, zwischen Lichtteilchen und Quelle erfolgt ein Abstoßungseffekt. Dies geschieht bis zu dem nächsten Lagrange-Punkt, dort tritt das Lichtteilchen wieder in ein dichter werdendes Feld ein, es erfolgt Verzögerung mit dem gleichen Effekt einer Kräftepaarbildung actio=reactio, welches zwischen Lichtteilchen und dem jetzigen Feldzentrum einen Druck aufbaut.

G-Felder sind damit Bestanteil der Quellen (Massen)!!!!, die Impulserzeugung eines Lichtteilchens verläuft über den gesamten Bereich von Eintritt ins Vakuum bis zum Lagrange-Punkt und die Impulsübertragung vom Lagrange-Punkt bis zur absorbierenden Fläche.

Da nun bei Durchquerung eines G-Feldes ein am Feldzentrum vorbeifliegendes Lichtteilchen sowohl beim Eindringen als auch beim Verlassen des Feldes eine Kraftwirkung actio=reactio zwischen sich und dem Feldzentrum erzeugt und die Resultierende dieser Kräfte größer Null ist, gibt das Lichtteilchen Energie an das durchquerte Feld ab. Folglich ermüdet Licht.

Ein Lichtteilchen entstammt einem Elektron, letzteres hat im Ruhzustand Spin ½. Das Lichtteilchen tritt aus mit Spin 1, d. h. es trägt “geborgte Energie“ in Form eines Impulses. Energieverlust ist Impulsverlust und Impulsverlust ist Verlust an geborgter Energie, wobei sich dieser Verlust durch eine Spin“abwicklung“ bemerkbar macht, welche zu einer Verlängerung der Welle führt, Lambda wächst. Und da die Fortpflanzungsgeschwindigkeit vakuumabhängig ist, macht sich ein größeres Lambda als Frequenzrückgang bemerkbar. Das ursächliche Kriterium für die Energie eines Lichtteilchens ist also nicht dessen Frequenz, sondern dessen Wellenlänge

Auch hier kann man wieder ein mechanisches Bild anführen: Eine konstant drehende Gewindestange wird gelängt, damit erhöht sich die Gewindesteigung, so daß ein sich parallel zur Stange mit v=c bewegender Beobachter immer länger braucht, um eine volle Umdrehung zu messen.

Und wenn Licht ermüdet, dann wird irgendwann der geborgte Spin “abgewickelt“ sein, so daß die kinetische Energie verbraucht ist und das Lichtteilchen in die nächstgelegene Ruhmasse stürzt und diese um den eigenen Ruhmassewert erhöht.

Ich bin mir bewußt, daß ich mit diesem Modell gegenwärtig nicht punkten kann, trotzdem bin ich mir sicher, richtig zu liegen. Der Urknall mit beschleunigter Universumsexpansion sowie die postulierte Masselosigkeit des Lichtes und die unsinnige Anwendung der Riemanngeometrie auf physische Felder dürften den größten Flop des 20sten Jahrhunderts darstellen.

möbius
27.04.08, 15:44
Und noch weiter, Gleichverteilungssatz: Gesamtenergie = Schwingungsenergie + Kinetische Energie + Potentielle Energie + Rotationsenergie +...

Und für Photonen Ekin = Eschwing = mc²/2 = hf/2 ,

und Egesamt = mc² = hf ? ...

S e h r richtig, criptically!!!

Und was bitteschön - ist jetzt noch das Problem der PHYSIKER:confused:
:D :D :D
möbius

Querkopf
27.04.08, 16:03
Nun wird ein BEK (BEC=BEK) aus Atomen hergestellt und die bestehen aus Fermionen (Elektronen und Quarks), also müssen Fermionen in Bosonen umwandelbar sein und umgekehrt, oder meinst Du, ein BEK hat weder Elektronen noch Quarks? Dann bliebe nämlich nichts mehr übrig außer sog. Wechselwirkungsteilchen, die bis heute mit Ausnahme von Photonen noch niemand nachgewiesen hat (oder hat schon mal jemand Gluonen oder Gravitonen nachgewiesen?).
Fermionen können aber gebundene Zustände eingehen, die sich wie Bosonen verhalten (Prominentes Beispiel sind z.B. Cooper Paare).
Ein Atom ist ein gebundener Zustand. Ein Nukleon ist ein gebundener Zustand. Ich wandele keine Teilchen um. Auch in einem Cooper Paar sind meine Elektronen immer noch Fermionen (ich kann sie also z.B. nicht aufeinandersetzen) aber als Kollektiv verhalten sie sich nach außen hin wie ein Boson.
Im übrigen kann ich auch die Eichbosonen der Schwachen Wechselwirkung messen.
Deine Vermutung, dass Elektronen aus dem Bose- Einstein -Condensate kommen könnten, steht im Widerspruch zu deiner Feststellung, dass sich das Kondensat im niedrigsten energetischen Zustand befindet. Es ist einfach nicht genügend Energie da um freie Elektronen aus dem Gebundenen Zustand herauszulösen (Für eine Ionisierung brauche ich jede Menge Energie).

Ihr könnt machen was Ihr wollt, solange Ihr den Spin und die Gravitation nicht erklären könnt, solange habt Ihr keine Ahnung, was Bosonen oder Fermionen sind, Ihr meßt etwas, von dem Ihr nicht wißt, was es ist und stellt Theorien auf, die reine Annahmen sind.
Der Spin folgt zwangsläufig aus dem Versuch eine relativistisch kovariante Quantenmechanik zu konstruieren (eine der großen Leistungen der Dirac Theorie).
Man kann mathematisch beweisen (über die Topologie von Weltlinien im Pfadintegralformalismus), dass es in drei oder mehr Dimensionen nur zwei Sorten von Teilchen geben kann (Bosonen und Fermionen), die entweder Symmetrisch oder antisymmetrisch unter Teilchenvertauschung sind, woraus auch das Pauliverbot folgt.
Man kann beweisen (Spin Statistik Theorem), das diese Teilchen einer unterschiedlichen Verteilungsfunktion gehorchen.

Eine Quantentheorie der Gravitation ist noch Forschungsthema, aber alle anderen Fragen sind seit Jahrzehnten beantwortet. Möglicherweise bist du nicht mehr ganz auf dem neusten Stand.
Und der gute Albert Einstein hat die Theorie sehr gut verstanden. Alle Welt schießt sich ja heute auf seine Relativitätstheorie ein, aber er war eigentlich ein Meister der Thermodynamik und Statistischen Physik (was sicherlich auch mit seiner praktischen Arbeit beim Patentamt zusammenhing). Er hat das Kondensat im Glauben an die Richtigkeit der Statistischen Physik (in Tradition von Boltzmanm und Planck und auf Anregung von Bose) während seiner Zeit in Leiden 1924 vorhergesagt. 1995 wurde die Vorhersage bestätigt. 71 Jahre später. Das nenne ich Klasse.

Mein Modell beruht auch auf Annahmen, aber auf sehr wenigen, während die Physik einen ganzen Sack voll davon braucht, für jedes Fachgebiet andere. Daß auch bei mir Irrtümer eingeschlossen sind, ist selbstverständlich, aber das enthebt Euch nicht der Aufgabe darüber nachzudenken, wie und warum die Ergebnisse zustande kommen.
Auch da scheinst du nicht ganz auf dem laufenden zu sein. Die Physik beruht auf sehr wenigen Annahmen.

Moderne Physik ist auf fundamentaler ebene Quantenfeldtheorie und Statistische Physik, wobei letztere heute im wesentlichen als Quantenfeldtheorie verstanden wird.
Die beiden Säulen der Physik sind Symmetriegruppen (im wesentlichen Lie – Gruppen) und die Renormierungsgruppe.

Wenn ich eine neue Theorie konstruiere dann wird diese vollständig durch die Forderung nach bestimmten Symmetrien und nach renormierbarkeit festgelegt.
Die Struktur der Elektrodynamik und ihrer möglichen Wechselwirkungen wird festgelegt durch die Forderung nach Invarianz unter Lorentztransformationen, Invarianz unter U(1) Eichtransformationen und was zusätzliche Wechselwirkungsterme angeht durch die Forderung nach Renormierbarkeit.
Das ist eine ziemlich einfache Struktur, setzt aber eine gewisse Kenntnis der modernen Physik und der Mathematik voraus.


Ein Feld mit einer vom Feldzentrum nach außen abnehmenden Energiedichte größer Null ist nun mal endlich, wenn der Energiegehalt endlich ist. E(Feld) = Integral [(EDr > 0)·r²·4·Pi·dr] ergibt nun mal bei E(endlich) ein endliches r. Ganz ohne komplizierte Mathematik. Allerdings beruht dieses Modell auf meinen Annahmen, welche das Universum als aus Energiefeldquanten aufgebaut ansehen.
Das ist zwar mathematisch richtig, aber physikalisch völlig unsinnig.
Beweis: Durch Haare kämmen
Beim Haare kämmen werden Ladungen getrennt. Eine Elektrische Punktladung hat ein potential von 1/r, also eine Kraft von 1/r^2 und eine energiedichte von 1/r^4 (Coulomb 1784, also noch vor der Französischen Revolution). Die Energie eines solchen Feldes ist gegeben durch ein Integral über 1/r^2. Das entspricht genau dem von mir beschrieben Fall.
Und wir reden nicht von einem exotischen Experiment, sondern vom alltäglichen erleben von Millionen Menschen.
Sage mir, welche Struktur muss ein Universum haben um eine Energiedichte größer oder gleich r^2 zu erlauben. Ich behaupte es gibt keine realistische Ladungsverteilung für das Universum die das erlaubt.
Mit realistisch meine ich, die Theorie muss im Einlang mit Beobachtungen wie denen beim Haarekämmen stehen und sie muss erklären, warum mein Computer sie auf dem Bildschirm anzeigen kann.

Ich behaupte jetzt mal ganz mutig, dass du nicht in der Lage sein wirst mir eine derartige Ladungsverteilung zu nennen.

Ich warte ohnehin noch auf ein Statement in Sachen Übereinstimmung deiner Theorie mit dem Experiment. Beim Casimir – Effekt stimmt deine Theorie ja so einigermaßen bei Materialien überein für die man keine Experimente macht. Viel interessanter sind aber eigentlich Materialien wie Gold und Aluminium, mit denen man aufgrund der Industriell verfügbaren Nanotechnik (sprich Halbleiterindustrie) üblicherweise solche Experimente durchführt. Und da liegst du einfach nur um ein paar hundert Prozent falsch.

Das ist ja eine tolle Theorie. Sie ist wunderschön und erklärt alles, aber dort wo man sie tatsächlich in Zahlenwerten überprüfen kann, ist sie bis zu 760% vom Experiment entfernt.
Das ist nicht bloß Spielerei, sondern hat handfeste Konsequenzen. Casimir war nach seinem Studium in Leiden ja auch bei Phillips beschäftigt. Phillips stellt zwar kaum noch Produkte her, ist aber in Sachen Forschung (und Patente) immer noch eine der Führenden Halbleiterfirmen. Wenn ich zu kleineren Strukturen übergehe, spielt der Casimireffekt eine entscheidende Rolle, z.B. für die Reibung von Mikromechanischen Bauteilen oder die Wärmeentwicklung von Chips. Du behauptest nun (im Widerspruch zum Experiment), dass der Effekt z.B. für Gold um den Faktor 7,6 mal so groß ist. Überlege welche Konsequenzen das hätte.

Ins blaue philosophieren mag ja unterhaltsamer sein, aber von einem Handy weiß ich das es funktioniert. Jeden Tag, abermillionenfach auf der ganzen Welt.
Und jede noch so tolle Weltformel, bei der am Ende rauskommt, dass mein Handy eigentlich nicht funktionieren dürfte, wird direkt in den Papierkorb befördert.

Wie hoch ist diese Energie wenn Omega = Null ist?
Null wenn ich richtig rechne! Warum muss das richtig sein? Weil (ω^2) * (x^2) mein harmonisches Oszillatorpotential ist. Wenn ich Omega gegen Null gehen lasse, wird mein Potential immer flacher, bis es schließlich verschwindet. Kein Potential bedeutet keine Schwingung bedeutet keine Energie für einen gebundenen Zustand (da ein solcher nicht existiert).


Querkopf, ich wäre Ihnen dankbar, wenn Sie mir dann eine brauchbare Definition Ihres Attributes "zentralsymmetrisch" festlegen könnten.
Meine Funktion (also mein Potential) ist Punktsymmetrisch, bei geeigneter Wahl des Koordinatensystems Punktsymmetrisch zum Ursprung, d.h.:
V(x)=-V(-x)
In höheren Dimensionen habe ich typischerweise Rotationssymmtrien um alle Achsen, also in 2D Kreissymmetrie, in 3D Kugelsymmtrie (z.B. Coulombpotential), in 4D ...
Entsprechend der Kugelsymmtrie des Coulombpotentials tauchen z.B. in der Elektrostatik (Multipolentwicklung) oder der Atomphysik (Winkelanteil der Wellenfunktion) Kugelflächenfunktionen auf.

criptically
27.04.08, 17:38
S e h r richtig, criptically!!!

Und was bitteschön - ist jetzt noch das Problem der PHYSIKER:confused:
:D :D :D
möbius

Die Physiker behaupten mc² ist die kinetische Energie des Photons, dabei vergessen sie, dass es auch eine Schwingungsenergie gibt (Grundzustandsenergei E=hf/2.

Daraus ergibt sich ein Problem für RT, weil die Ruhemasse des Photons nicht mehr = 0, sondern gleich m ist, wodurch auch relativistische Massenzunahme ganz andere Erklärung benötigt! :D :D

mfg

criptically
27.04.08, 17:49
...Der Spin folgt zwangsläufig aus dem Versuch eine relativistisch kovariante Quantenmechanik zu konstruieren (eine der großen Leistungen der Dirac Theorie)

Der Spin wurde experimentell gefunden!


...Null wenn ich richtig rechne! Warum muss das richtig sein? Weil (ω^2) * (x^2) mein harmonisches Oszillatorpotential ist. Wenn ich Omega gegen Null gehen lasse, wird mein Potential immer flacher, bis es schließlich verschwindet. Kein Potential bedeutet keine Schwingung bedeutet keine Energie für einen gebundenen Zustand (da ein solcher nicht existiert).

...
Das bedeutet, alle Energien sind möglich von 0 bis ?? und die Unschärferelation müsste neu überdacht werden :D . Und die Quantisierung gilt nur für gebundene Systeme was übrigens auch makroskopisch gilt. Beispiel: Schwingungen einer gespannten Saite! :D :D

mfg

uwebus
27.04.08, 18:58
Querkopf,
----
Fermionen können aber gebundene Zustände eingehen, die sich wie Bosonen verhalten (Prominentes Beispiel sind z.B. Cooper Paare).
----
Das meinte ich, wenn ich sagte, daß Fermionen in ortsfeste “Bosonen“ umwandelbar sein müssen und umgekehrt. Ortsfest bedeutet für EIN individuelles Teilchen immer Spin ½ , aber wenn ZWEI individuelle Teilchen mit +/- Spin ½ aufeinander wirken, ergibt sich Spin 1, wie das beim Zusammentreffen von Positron und Elektron passiert, wenn sich daraus zwei Photonen bilden. Treffen zwei Photonen mit +/- Spin 1 aufeinander, kann der Gesamtspin auch nur 1 betragen, also bilden sich zwei Teilchen mit Spin ½, Positron und Elektron. “Ortsfeste“ Ruhmasse bedeutet Spin ½. Ein einteiliges Boson bewegt sich mit c, d.h. hier wird Ruhmasse in kinetische Energie umgewandelt, indem der Spin auf 1 erhöht wird und dadurch der Vakuumdruck (die Raumkrümmung der Relativisten) nicht mehr ausreicht, die Energie im Atom zu binden. Ich hatte das mechanisch verdeutlicht mit einem Elektronen“kreisel“, der in einem konischen Trichter läuft. Bei Spin ½ hält der Vakuumdruck das Elektron im Protonentrichter, bei erreichen von Spin 1 ist der Trichterdruck aufgrund der Kegelwirkung größer als der Vakuumdruck, die auf Spin 1 erhöhte Elektronenenergie wird vom Proton ins Vakuum ausgestoßen.

Ich habe dafür zwar schon genügend Hohn geerntet, aber stell Dir einen Eiskunstläufer vor, der in einem Spintrichter drehend kreist derart, daß sein Gewicht durch eine auf seine Schlittschuhe wirkende oszillierende Gegenkraft gehalten wird. Im Spintrichter steigt die Gegenkraft mit zunehmender Tiefe. Erhöht jetzt der Läufer seine Drehzahl durch Pirouetteneffekt, sinkt er im Trichter tiefer, sein Gewicht bleibt konstant, die Gegenkraft aber steigt, dem Läufer werden die Schlittschuhe von den Füßen gerissen und fliegen aus dem Trichter. Ist der gedachte Läufer ein Elektron, wird der tiefstliegende Elektronenbereich als Photon aus dem Trichter geschleudert. Um den Trichter zu verlassen ist eine bestimmte Fluchtgeschwindigkeit notwendig, die von dem Protonentrichter vorgegeben wird. Daher ist die Lichtaustrittsgeschwindigkeit aus einem Atom konstant. Aus einem Neutron wird bei dessen Zerfall ebenfalls Energie abgegeben, ein Elektron und ein Neutrino. Da die Masse eines Neutrons in etwa der Protonenmasse entspricht, dürfte die notwendige Fluchtgeschwindigkeit für ein Neutrino bei der eines Lichtteilchens liegen, damit müßten Licht und Neutrino die gleiche Geschwindigkeit aufweisen.

----
Es ist einfach nicht genügend Energie da um freie Elektronen aus dem Gebundenen Zustand herauszulösen (Für eine Ionisierung brauche ich jede Menge Energie).
----
Das ist ja richtig. Aber der energieärmste Zustand ist ein Zustand gleicher Schwingung aller beteiligten BEK-Teilnehmer, so daß sich hier grundsätzlich die Wirkungen akkumulieren können so wie auch die Wirkungen freier Elektronen auf einer Metalloberfläche. Ich will Euch ja nicht Eure virtuelle-Teilchen-Theorie verderben, aber vielleicht sollte man einfach mal andere Alternativen in Erwägung ziehen, zumal, wenn ein Gedankenmodell zu Werten führt, die größenordnungsmäßig ziemlich passen.

----
Der Spin folgt zwangsläufig aus dem Versuch eine relativistisch kovariante Quantenmechanik zu konstruieren (eine der großen Leistungen der Dirac Theorie).
----
Eure Theorien in Ehren, aber die Ursache des Spins ist zu erklären aufgrund eines Modells, welches die Notwendigkeit der Dynamik im Universum begründet. Ihr müsst erklären können, warum sich das Universum notwendigerweise bewegt, unabhängig von den einzelnen Bewegungsarten. Der Spin ist ja nur eine von mehreren dynamischen Verhaltensweisen von Energie.

-----
Das ist zwar mathematisch richtig, aber physikalisch völlig unsinnig.
----
Physikalisch unsinnig mag sein, aber physisch zutreffend. Physik ist Beschreibung der Physis, letztere steht vor der Physik. Beschreibungen können falsch sein. Euer Problem ist und bleibt das Verständnis des Raumes als Summe endlicher Felder. Und solange Ihr da nicht weiterdenkt, werdet Ihr die Gravitation nicht verstehen und ohne Verständnis der letzteren ist es unmöglich, die Dynamik des Universums zu begründen. Ihr nehmt das Universum als gegebenes dynamisches Objekt zur Kenntnis, Verständnis aber erfordert Begründungen, Begründungen, Begründungen!!!!

----
Beim Haare kämmen werden Ladungen getrennt.
----
Kein Physiker weiß, was eine Ladung ist. Ihr beschreibt Beobachtungen mithilfe von Ladungen, d.h. Ihr definiert Gemessenes, ohne zu wissen, was ihr gemessen habt.

----
Sage mir, welche Struktur muss ein Universum haben um eine Energiedichte größer oder gleich r^2 zu erlauben.
----
Bei mit ist die Energiedichte eines Feldes im Vakuumbereich EDmin·ra²/r² mit ra = äußerer Feldradius. Berechnung siehe meine HP. D.h. es ergibt sich im Universum eine geringstmögliche Energiedichte EDmin. Und dieser Wert folgt aus den empirischen Werten G, h und c und führt zu den in der HP aufgeführten Übereinstimmungen mit den unterschiedlichsten Messwerten der Physik. Warum wohl? Weil Gravitationsfelder Bestandteile der zentralen Massen sind, es gibt kein feldfreies Vakuum, wie Ihr dies annehmt. Eure Feldtheorien sind falsch.

----
....ist sie bis zu 760% vom Experiment entfernt.
----
Also wenn eine grobe Schätzung bei einem Grundwert von 10^10 maximal um den Faktor <10^1 abweicht, dann ist das m.E. keine Katastrophe. Überleg mal, wie weit bei der sog. Vakuumenergie/Dunkle Energie Empirie und physikalische Annahme auseinander gehen, um den Faktor 10^120!. Da müßte Dir doch richtig schlecht werden.

Gruß

Querkopf
27.04.08, 19:34
Der Spin wurde experimentell gefunden!
Das ist schon richtig und in der Pauli Theorie baue ich ihn dann von Hand ein. Aber in der Dirac Theorie ergibt er sich auf ganz natürliche Weise Aufgrund von ganz anderen Annahmen unabhängig vom Experiment.

Das bedeutet, alle Energien sind möglich von 0 bis ?? und die Unschärferelation müsste neu überdacht werden . Und die Quantisierung gilt nur für gebundene Systeme was übrigens auch makroskopisch gilt. Beispiel: Schwingungen einer gespannten Saite!
Ganz so kann man das vielleicht nicht sagen, aber es stimmt schon: Quantisierung bedeutet nicht, das man nur noch diskrete Energien hat.

Das wird in der Schule, aber auch in der Populärwissenschaft gerne so verkauft. Es deckt sich auch gut mit den üblicherweise gezeigten Beispielsystemen. Diese Beispielsysteme z.B. der unendlich tiefe Potentialtopf haben aber auch nur gebundene Zustände. Betrachte ich einen endliches Potential und gehe zu hohen Energien, habe ich quasifreie Teilchen und ein annähernd kontinuierliches Energiespektrum (sonst könnte ich auch nie so etwas wie klassisches Verhalten bekommen).

Was passiert aber wenn mein potential null ist, ich also eich echtes freies Teilchen habe? Der Anschaulichkeit wegen, gehen wir einmal zur Ortsdarstellung über. Die Wellenfunktion eines freien Teilchens ist eine Ebene Welle. Das Teilchen muss also gleichmäßig über ein Universum unendlicher Ausdehnung verteilt sein. Für eine Ebene Welle ist unendlich viel Energie notwendig. Sollte das Universum endlich sein, kann es unabhängig von der Energie eine solche Lösung ohnehin nicht geben. Ich bin mir ziemlich sicher, das ich in meinem Leben nie einem Freien Teilchen über den Weg laufen kann (allerdings vielen quasifreien Teilchen).
Wenn du also die Energie eines harmonischen Oszillators (der hat nur gebundene Zustände) untersuchst und die Energie für omega = null wissen willst, dann ist die Energie des gebundenen Zustandes Null. Du hasst nämlich keinen harmonischen Oszillator mehr. Was nicht existiert trägt auch nicht zur Energie bei. Stattdessen hast du ein freies Teilchen.

Quantenmechanik ist eben keine Magie, sondern das Wort Mechanik hat seine Bedeutung. Es ist ein bisschen Quanten und viel, viel Mechanik.

Soweit die Quantenmechanik, also die Vorstellung, das ich ein Teilchen mit einer gewissen kinetischen Energie in ein Potential setze.

In der QFT habe ich erst einmal überhaupt kein Teilchen. Meine Teilchen werden erzeugt bzw. vernichtet durch Operatoren meines Feldes. Diese Operatoren entsprechen denen eines QM harmonischen Oszillators. Die Operatoren kommen von den Moden einer Fouriertransformation. Diese Transformation war möglich, weil mein Feldgleichung linear war.

Meine Oszillatoren haben also nur noch mathematische Berechtigung. Es ist ein Ansatz für ein lineares System. Es macht keinen Sinn sich über ein zu Grunde liegendes Masse – Feder- Modell oder einen Schwingkreis zu streiten. Und ich kann über alle Omega summieren, also alle möglichen Oszillatoren, auch den mit einer Frequenz von Null, den die gibt mir keinen Energetischen Beitrag.
Wenn ich Teilchen Betrachte, muss ich mir um die Grundzustandsenergie keine sorgen machen, denn der Grundzustand enthält hat per Definition keine Teilchen. Ein Teilchen ist ein Zustand der durch einen Erzeugungsoperator aus dem Grundzustand erzeugt wird. Dieses Teilchen hat dann eine Frequenz von Omega und eine Energie von h – quer Omega.



Also wenn eine grobe Schätzung bei einem Grundwert von 10^10 maximal um den Faktor <10^1 abweicht, dann ist das m.E. keine Katastrophe. Überleg mal, wie weit bei der sog. Vakuumenergie/Dunkle Energie Empirie und physikalische Annahme auseinander gehen, um den Faktor 10^120!. Da müßte Dir doch richtig schlecht werden.
Da wird mir gar nicht schlecht. Da zucke ich mit den Schultern und sage: Wen Interessierts? Irgendwelche dunklen Energien irgendwo weit draußen im Universum. Wer weiß schon was da wirklich los ist.

Aber wenn man sich im Mikroskopischen bereich um einen Faktor im Bereich von 10 verschätzt ist das schon heftig. Teilweise haben wir in der Atom oder Festkörper Experimente, in denen es nicht um einen Faktor 10, sondern um die zehnte Nachkommastelle geht.
Wir reden nicht von irgendwelchen neuen, ungeklärten Phänomenen, sondern von irdischen, im Laufe der Jahrzehnte von verschiedensten Gruppen durchgeführten Experimenten, zu einem vor 60 Jahren auf Grundlage eines mittlerweile als best überprüfte menschliche Theorie geltendem Fundament postulierten Effekts, der darüber hinaus im Rahmen der weiteren Entwicklung der Halbleiterindustrie von Endscheidender Bedeutung ist.

Du machst also eine Vorhersage, die vielleicht schon in zehn Jahren im Widerspruch zur Funktionsfähigkeit deines Computers oder Handys steht.

Kein Physiker weiß, was eine Ladung ist. Ihr beschreibt Beobachtungen mithilfe von Ladungen, d.h. Ihr definiert Gemessenes, ohne zu wissen, was ihr gemessen habt.
Was eine Ladung ist sowie ihre Erhaltung folgt aus dem Eichprinzip und dem Noethertheorem. Diese theoretische Ladung kann dann mit einer experimentell gefundenen identifiziert werden

Querkopf
27.04.08, 19:38
Photonen mit +/- Spin 1 aufeinander, kann der Gesamtspin auch nur 1 betragen, also bilden sich zwei Teilchen mit Spin ½, Positron und Elektron.
Ich habs ausprobiert. Bei mir kam es einfach zu einer Überlagerung von Elektromagnetischen Wellen nach dem guten alten Superpositionsprinzip.

rafiti
27.04.08, 21:58
Physikalisch unsinnig mag sein, aber physisch zutreffend. Physik ist Beschreibung der Physis, letztere steht vor der Physik. Beschreibungen können falsch sein. Euer Problem ist und bleibt das Verständnis des Raumes als Summe endlicher Felder. Und solange Ihr da nicht weiterdenkt, werdet Ihr die Gravitation nicht verstehen und ohne Verständnis der letzteren ist es unmöglich, die Dynamik des Universums zu begründen. Ihr nehmt das Universum als gegebenes dynamisches Objekt zur Kenntnis, Verständnis aber erfordert Begründungen, Begründungen, Begründungen!!!!


Wen interessiert schon dein Rumgejammer hier? Gar nichts steht vor gar nichts, das überlasse mal lieber jedem und jeder Wissenschaft für sich. Und tue nicht immer so als wenn du ein unverstandenes Opfer wärst und andere mit Melonenöl kochen sollen, während du selber nur mit Wasser kochst... Ich denke, was @Querkopf sagt, reicht aus, um deine Theorie ad absurdum zu führen.


gruss
rafiti

Uranor
27.04.08, 23:25
Physikalisch unsinnig mag sein,
Und? Er weiß es und posaunt dennoch permanent unsinnig? Prangert, kekst und doctriniert pauschal alle anderen gegen sein eigenes besseres Wissen? Überspannten wird es zu einfach gemacht. Eine wichtige Bitte wäre, die totale Gesichtskontrolle einzufüren. Lieber fall ich selbst ab und zu durch, als dass ich mir ständig schräge Kekse anhören mag.

Lass das Dauergesäusel da weg. Es wäre mehr als angemessen. Schließlich gibst du selbst zu, dass du gar nix drauf hast. Wozu also das Gepiense?

Uranor

uwebus
28.04.08, 14:36
ingeniosus,
----
Unter Masse oder Ladung kann ich mir einigermassen was vorstellen, aber ein Feld ist eben ein Abstraktum, sicherlich gedanklich hilfreich - aber doch.
----
Nein, ein Feld ist eben KEIN Abstraktum, sondern ein das Universum konstituierendes Element. Ein Abstraktum wird ein Feld erst, wenn man wie die Physik Raum/Vakuum als Aufbewahrungsbehälter für Felder postuliert sowie Felder von den Massen trennt, denn dann wird ein Feld zu einer Eigenschaft eines feldlos gedachten Raumes/Vakuums.

Das Problem der Physik liegt m.E. in der fehlenden philosophischen Grundlage. Ich muß das Universum und damit auch das Vakuum als OBJEKT betrachten, als physische Manifestation des metaphysischen Seins, wobei metaphysisch jetzt kein spinnerter Begriff ist, sondern lediglich aussagt, daß das Universum auf ein nicht mehr deduzierbares und damit unbegründbares Etwas zurückgeführt werden muß. Den Ursprung kann ich nur postulieren, als Materialist als ein metaphysisch-physisches Etwas (ich habe mich hier für die aristotelische SUBSTANZ entschieden), als Schöpfungsanhänger als GOTT. Mein Modell beruht auf einer materialistischen Weltsicht, d.h. ich gehe von der Welt als einem Perpetuum mobile aus.

Ich bin mir bis heute noch nicht klar, von was eigentlich Urknallanhänger ausgehen, denn Urknall bedeutet auch Universumstod (Urknaller sprechen ja davon, daß sich das Universum in schwarze Löcher zurückzieht und dort für alle Zeiten begraben bleibt). Der Urknall ist einfach philosophisch nicht zu Ende gedacht und beruht auf der irrigen Annahme verlustfreier Energieübertragung mittels EM-Wellen durchs Vakuum. Das wäre aber nur möglich, wenn Vakuum leerer Raum wäre, also existierendes Nichts, das aber ist wiederum nicht möglich, da mathematische Ausdehnung eine Qualität und keine Entität ist.

----
Ihre Festlegung über die endliche Energiemenge ist völlig OK,...
----
Auch hier gehe ich philosophisch vor: Nichts ist nicht denkbar, alles was ist, ist Universum. Also kann ich das Universum in beliebig kleine Teilchen teilen, aber immer bleibt Ausdehnung erhalten. Damit kann ich eine beliebig kleine Arche postulieren, wobei der Wert E0 =h·1/s willkürlich gewählt ist, es ist jede beliebige Größe denkbar wegen R0/E0~G. Ich habe einfach nur einen möglichst kleinen Wert der Physik gewählt.


möbius
----
Und m = hf/c² ???
----
Nein, der Impuls des Lichtes f·h/c sitzt vorwiegend in der das Lichtteilchen begleitenden Vakuumwelle. Nehmen wir mal an, Jan Ulrich radelt bergab und schiebt eine große Luftmasse vor sich her, die an ihm vorbei gleitet und hinter ihm wieder zusammenströmt. Wenn er jetzt gegen eine Wand fährt, dann bekommt die zuerst die Frontwelle, dann den Körper und danach die nachströmende Welle als Impuls zu spüren, wobei im Falle des Lichtes der “Körper“, also das vom Elektron emittierte Teilchen die geringste kinetische Energie ausmacht. Die Teilchenmasse des sichtbaren Lichtes liegt bei m < 10^-45kg, ist damit praktisch nicht nachweisbar.

Querkopf,
----
Da zucke ich mit den Schultern und sage: Wen Interessierts?
----
Na ja, Physiker interessiert es offensichtlich nicht, aber ich bin “Physissophist“ und unternehme den Versuch, das beobachtete Universum philosophisch zu deuten. Meine auf Physik beruhende Ingenieurtätigkeit liegt in der Vergangenheit, vor mir liegt die Jordanüberschreitung, und da möchte ich doch schon eine Vorstellung davon haben, was ich eigentlich auf dieser Seite zu tun hatte.

----
Ich habs ausprobiert.
----
Manchmal klappt es wohl doch, z.B. wenn große Photonen mit Elektronen zusammenstoßen, denn es werden doch Positron und Elektron messtechnisch nachgewiesen oder nicht? Elektronen sind ja nicht ortsfest, sondern bewegen sich auch mit v gegen c, wenn ich Gammastrahlung in einen Elektronenstrahl schieße. Inwieweit energiereiche Strahlung zu Masse wird, weiß ich nicht, aber anscheinend passiert das ja ständig in der Stratosphäre, wo sich aus der Höhenstrahlung Myonen und Antimyonen bilden, die dann wieder zerfallen.


http://de.wikipedia.org/wiki/Paarbildung_%28Physik%29
„Bei der Wechselwirkung von Gammastrahlung genügender Energie (mindestens 1,022 MeV, das Zweifache der Ruheenergie von Elektron bzw. Positron) mit Materie kann je nach Ordnungszahl die Paarbildung der vorherrschende Prozess sein.“

Mir geht es lediglich darum aufzuzeigen, daß Licht, Masse und Vakuum einfach unterschiedliche Zustände ein und desselben Grundmaterials “Substanz“ sind, daß also die Energieumwandlung in beide Richtungen möglich ist und damit Licht nach Verlust seiner kinetischen Energie zu Ruhmasse wird. Denn etwas anderes passiert ja bei der Paarbildung auch nicht, dem Photon wird beim Zusammenstoß geborgte kinetische Energie entzogen und es teilt seinen Spin 1 in zwei gegenläufige (actio=reactio) Teilchen Spin ½ (wobei sicherlich noch ein Teil der Energie als Verlustwärme das Weite sucht).
Die Paarbildung beweist m.E., daß das Universum ein Perpetuum mobile sein kann, daß also Wärme wieder zurückverwandelt werden kann in Ruhmasse. Denn es dürfte unwahrscheinlich sein, daß große Photonen Teilchen bilden können, kleine jedoch nicht.

Mein Modell, welches das Vakuum einer Masse zuordnet, führt ja kinetische Energie und G-Feldenergie zusammen, hohe kinetische Energie entspricht kleinem Feldvolumen und viceversa. Berechnet man dies in Stufen, kommt die schon gezeigte Kurve http://uwebus.de/F/20V-A-6.gif dabei heraus, die eine merkwürdige Übereinstimung mit der SRT aufweist.

Gruß

Querkopf
28.04.08, 14:54
Manchmal klappt es wohl doch, z.B. wenn große Photonen mit Elektronen zusammenstoßen, denn es werden doch Positron und Elektron messtechnisch nachgewiesen oder nicht?
Das ist mir schon klar (ich habe solche Prozesse nämlich schon durchrechnen dürfen).

Deine Aussage war aber, das beim Zusammenstoß von zwei Photonen (wie es z.B. bei der Überlagerung von zwei Lichtquellen ständig der Fall sein sollte) ein Elektron - Positron – Paar frei wird.
Da habe ich auch für höhere Energien meine Zweifel. Du kannst mich aber gerne durch ein Feynmandiagram überzeugen.

Es gibt natürlich Prozesse mit zwei Photonen, du musst aber mal schauen, wo die Wechselwirkungspunkte sind.

pauli
28.04.08, 16:02
@uwebus
vor mir liegt die Jordanüberschreitung, und da möchte ich doch schon eine Vorstellung davon haben, was ich eigentlich auf dieser Seite zu tun hatte.

Die Jordanüberschreitung hat ja nun jeder vor sich, der eine früher der andere später, und wer will kann sich sicher Gedanken zu "seinem" spezifischen Lebenssinn und -Inhalt machen, aber was hat das mit Physik zu tun?

Natürlich kann man einen Großteil seiner "restlichen" Lebensjahre mit der Formulierung eigener Theorien verbringen, wenn es denn eine Erfüllung bedeutet, aber nur die wenigsten werden es schaffen, sich auf diesem Wege ein Denkmal zu errichten.

uwebus
28.04.08, 19:04
Querkopf,
----
Deine Aussage war aber, das beim Zusammenstoß von zwei Photonen...
----
Wo hört ein Photon auf Photon zu sein und beginnt ein Teilchen zu werden? Höhenstrahlung ist ja wohl überwiegend Teilchen“strahlung“ von Teilchen, die sich aber mit v nahe c bewegen. Ich glaube, daß ab UNTERschreitung einer bestimmten Energiemenge sich Spin 1 einstellt, wenn ich ein Teilchen auf c(Vakuum) beschleunige. Je größer das Teilchen wird, desto schwerer wird es, es zu beschleunigen. Nimm eine Kinder-Windmühle und fang an zu laufen, das Ding dreht immer schneller. Je größer die Mühle, desto größer der Widerstand und desto höher die Drehzahl, irgendwann stellt sich Gleichgewicht ein zwischen Widerstand und Vortrieb, das ist der Fall, wenn c vorwärts = c tangential erreicht ist. Wird die Mühle noch größer, erreichst Du c tangential schon bei v vorwärts < c. Ich werde mal zusehen, dies anhand eines Sinus-Modells zu modellieren.

http://de.wikipedia.org/wiki/Kosmische_Strahlung
“Beim Eintreten in die Erdatmosphäre in einer Höhe um 20 km über der Oberfläche erzeugt die kosmische Strahlung Teilchenschauer. Aus einem Proton der Energie von 1015 eV entstehen mehr als eine Million Sekundärteilchen. Nur ein kleiner Teil von ihnen erreicht auch die Erdoberfläche.“

Wenn ich meine Kalkulationen zugrundelege, dann hätte sichtbares Licht als Teilchen eine Masse von < 10^-45 kg, also könnte ich aus einem Elektron rd. 10^15 Lichtphotonen herausholen. Ein Proton hat rd. 10^3 Elektronenmassen, wenn dies nach obigem Text in 10^6 Sekundärteilchen zerfällt, dann sind das immer noch Riesenbrocken von rd. 10^-33 kg gegenüber einem Photon des sichtbaren Spektrums.

Und überzeugen will ich Dich weder mit Feynman noch mit anderem, ich will einfach versuchen, Euch dazu zu bewegen, das Vakuum als Energieform zu betrachten und in einen qualitativen UND quantitativen Zusammenhang mit den Massen im Universum zu stellen, weil nur so aus meiner Sicht das Phänomen Gravitation verständlich wird. Und erst das Verständnis der Gravitation führt zum Verständnis der Entstehung von Dynamik.

pauli,
----
...aber was hat das mit Physik zu tun?
----
Nur indirekt, da Physik die Physis beschreibt und die philosophische Deutung der Physis mit der physikalischen Empirie in Einklang stehen muß. Wenn ich also philosophisch ein Perpetuum mobile postuliere, welches aus einer einzigen metaphysischen Substanz hervorgeht, dann muß ich ein technisches Perpetuum-mobile-Modell entwickeln, von mir Arche getauft, welches mir erlaubt, die physikalisch-empirischen Beobachtungen damit darzustellen. Und das scheint bisher ganz gut zu gelingen, so daß ich Argumente für meine materialistische Weltsicht mit der physikalischen Empirie untermauern kann. Daß ich die Entstehung von Bewußtsein im Universum nun als teleologische Selbstorganisation der Physis ansehe, andere dies aber als bloßen Zufall abtun, ist dann reine Spekulation und kann nicht mehr empirisch gestützt werden außer vielleicht dadurch, daß wir nun mal existieren und dies bei einer endlichen Zahl unterschiedlicher Bausteine eben unumgänglich ist wie ein Sechser im Lotto bei nur 49 Zahlen.

Gruß

criptically
28.04.08, 21:28
...
Manchmal klappt es wohl doch, z.B. wenn große Photonen mit Elektronen zusammenstoßen, denn es werden doch Positron und Elektron messtechnisch nachgewiesen oder nicht? Elektronen sind ja nicht ortsfest, sondern bewegen sich auch mit v gegen c, wenn ich Gammastrahlung in einen Elektronenstrahl schieße. Inwieweit energiereiche Strahlung zu Masse wird, weiß ich nicht, aber anscheinend passiert das ja ständig in der Stratosphäre, wo sich aus der Höhenstrahlung Myonen und Antimyonen bilden, die dann wieder zerfallen.


http://de.wikipedia.org/wiki/Paarbildung_%28Physik%29
„Bei der Wechselwirkung von Gammastrahlung genügender Energie (mindestens 1,022 MeV, das Zweifache der Ruheenergie von Elektron bzw. Positron) mit Materie kann je nach Ordnungszahl die Paarbildung der vorherrschende Prozess sein.“

Mir geht es lediglich darum aufzuzeigen, daß Licht, Masse und Vakuum einfach unterschiedliche Zustände ein und desselben Grundmaterials “Substanz“ sind, daß also die Energieumwandlung in beide Richtungen möglich ist und damit Licht nach Verlust seiner kinetischen Energie zu Ruhmasse wird....

Gruß

Das Photon ist aus Elektron und Positron aufgebaut! Je nachdem wie viel Energie sie besitzen, sind ihre Ladungen (und somit die Masse) teilweise kompensiert. Bei hochenergetischen Photonen ist die Schwingungsamplitude so groß, dass Elektron und Positron beinahe ganz getrennt sind, weshalb sie sehr leicht in einem starken E-Feld zerfallen.

Photon ==> Elektron + Positron.

Die kinetische Energie eines Teilchens ist definiert als E=p²/2m.
Mit m = hf/c² und p=f·h/c erhält man für Photonen

Ekin=(fh/c)²/(2hf/c²)=fh/2=mc²/2.

mfg

rafiti
28.04.08, 21:57
Nur indirekt, da Physik die Physis beschreibt und die philosophische Deutung der Physis mit der physikalischen Empirie in Einklang stehen muß. Wenn ich also philosophisch ein Perpetuum mobile postuliere, welches aus einer einzigen metaphysischen Substanz hervorgeht, ...

Nein, die Philosophie muss nicht mit der Empirie im Einklang stehen, da sie darüber hinausgeht und was ein Philosoph nicht weiß, das kann er auch gar nicht deuten oder kennst du einen, der vor Einstein & Co. sagte: "Ach wie gut, dass niemand weiß, dass die Streke bei c mir kürzer (er)scheint..." Und ob die Seele jener bei Sonnenschein anders wird, interessiert die Sonne nicht. Unphilosophisch und hart, aber so ist das nun mal und es ist nicht abwertend gemeint.
Ich widerum verstehe nicht, wozu du diese Substanz überhaupt brauchst.


gruss
rafiti

möbius
29.04.08, 08:47
Die Physiker behaupten mc² ist die kinetische Energie des Photons, dabei vergessen sie, dass es auch eine Schwingungsenergie gibt (Grundzustandsenergei E=hf/2.

Daraus ergibt sich ein Problem für RT, weil die Ruhemasse des Photons nicht mehr = 0, sondern gleich m ist, wodurch auch relativistische Massenzunahme ganz andere Erklärung benötigt! :D :D

mfg

Aber hatte das nicht Louis de Broglie durch seine Gleichung "irgendwie"
gelöst???
mc² = hf
Was ist denn vor dem Hintergrund dieser Gleichung noch "un-gelöst":confused:
möbius

pauli
29.04.08, 13:36
Das Photon ist aus Elektron und Positron aufgebaut!
hä, ist es nicht aus Yin & Yang aufgebaut?

möbius
29.04.08, 14:04
.... oder aus Dick und Doof:confused: :confused: :confused:
:D :D :D
möbius

uwebus
29.04.08, 14:37
criptically,
----
Das Photon ist aus Elektron und Positron aufgebaut! Je nachdem wie viel Energie sie besitzen, sind ihre Ladungen (und somit die Masse) teilweise kompensiert.
----
Aus meiner Sicht wird die Ruhmasse eines Teilchens aus dessen G-Feld erzeugt, im Ruhzustand ist die innere Dynamik (= Ladung) genauso groß wie die G-Feldenergie. Betrachte ich ein Teilchen an der Erdoberfläche als Sphäre, dann ist das zugehörige G-Feld ein extrem langer schlanker Kegel im Erd-G-Feld. Dieser Kegel drückt das Teilchen in Richtung Gravitationszentrum, die Gegenkraft wird durch die unter und seitlich des Teilchens liegenden Teilchen erzeugt. Das Teilchen oszilliert folglich, da Kompression durch Entspannung ausgeglichen wird. Da aufgrund der Einspannung des Teilchens im Gesamtfeld keine sphärische Oszillation möglich ist, wird aus einer kugelsymmetrischen Oszillation eine Art Drehschwingung zwischen einem Brummkreisel und einer Unruh einer Uhr, wobei die Außenfläche des Teilchens eingespannt ist, sich damit die Schwingung als innere Oszillation bemerkbar macht ähnlich einem oszillierenden Strudel.

Jedes ruhende Teilchen hat solch einen Strudel, in den hinein ein weiteres kleineres Teilchen fallen kann, wobei sich dann an der Berührungsfläche Strudel-Teilchen Gleichlauf einstellt, d.h. das größere Teilchen schwingt innen wie eine Unruh, das kleinere Teilchen schwingt außen in der gleichen Frequenz mit und erhöht nur noch anteilig seine Frequenz nach innen in einem eigenen Strudel. Es stellt sich dynamisches Gleichgewicht zwischen Aufnahmestrudel und mitschwingendem inneren Teilchen ein, sobald die Dynamik des inneren Teilchen durch Energieabgabe soweit abgeschwächt ist, daß das Teilchen in dem Strudel stabil mitläuft. Das Resultat ist z.B. ein H-Atom.

Wird ein Teilchen vereinzelt, dann ist es nicht mehr eingespannt, die Oszillation kehrt sich um, das Teilchen fängt an um seine Strudelachse zu schwingen. Diese Schwingung wird durch den Vakuumwiderstand begrenzt, die maximale Geschwindigkeit wäre hier c. Da infolge des Strudels das Teilchen im Ungleichgewicht mit dem es umgebenden Vakuumdruck ist, wird es in Richtung der Strudelachse beschleunigt, auch hier wird die Geschwindigkeit durch den Vakuumwiderstand auf c begrenzt. Ein freies Teilchen bewegt sich also um seine Strudelachse schwingend mit c durchs Vakuum. Wird das Teilchen größer, steigt die Verdrängungsfläche und damit der Widerstand, das Teilchen gerät unter Spannung und schwingt schneller. Ab einer gewissen Teilchengröße wird die innere Dichte so hoch, daß die Schwingung nicht mehr das gesamte Teilchen erfasst, der Strudel wächst nicht mehr linear mit der Teilchengröße, der Antrieb nimmt damit relativ ab, das Teilchen erreicht nicht mehr c.

Wird jetzt ein Teilchen mit einem anderen Teilchen zum Zusammenstoß gebracht, dann wird durch den Stoß der vordere Teilchenbereich mit dem ihm inhärenten Drehimpuls gestaucht und seitlich verdrängt, und da die Verdrängungsgeschwindigkeit auf c begrenzt ist, erhält der hintere Teilbereich einen gleichgroßen Gegenimpuls in Form eines entgegengesetzten Spins. Die vernichtete kinetische Energie hat sich in Drehimpuls umgewandelt, wobei die beiden Teilchenhälften nun mit gegenläufigem Spin aufeinander liegen und sich damit abstoßen, es sind zwei Teilchen entstanden, ein Positron und ein Elektron oder - im großen Maßstab - ein Proton und ein Antiproton.

Ich glaube, man kann Licht und Teilchen und deren gegenseitige Umwandlung sehr gut mechanisch darstellen, sofern man das Vakuum als Teilchenbestandteil in seine Überlegungen einbezieht.


rafiti,
----
Nein, die Philosophie muss nicht mit der Empirie im Einklang stehen, da sie darüber hinausgeht,...
----
Was hat Philosophie für einen Sinn, wenn sie Aussagen trifft, die der empirischen Erfahrung widersprechen? Was hat Physik für einen Sinn, wenn sie Annahmen trifft, die der empirischen Erfahrung widersprechen?

Ich sag es immer wieder: nur die Evidenz einer Wahrnehmung und das zu einer evidenten Wahrnehmung führende Experiment stehen uns als Wahrheitskriterien zur Verfügung. Evidenz ist damit sowohl für Philosophen als auch für die Naturwissenschaften das ausschlaggebende Kriterium.

----
Ich widerum verstehe nicht, wozu du diese Substanz überhaupt brauchst.
----
Als Gottersatz. Irgendwas muß ja fürs Universum verantwortlich zeichnen, der Laden ist doch da!


möbius,
----
mc² = hf
----
Das bezieht sich auf die Photonenenergie. Jetzt bleibt es Aufgabe der Physik festzustellen, welcher Energieanteil als Teilchen und welcher als Vakuumwellenergie vorliegt. Ich hatte das mit Jan Ulrich auf dem Fahrrad zu verdeutlichen versucht; das Teilchen ist der Jan samt Fahrrad, die Welle die von ihm verursachte Luftbewegung. Ohne Teilchen keine Energie transportierende Welle, das gilt auch fürs Vakuum, auch wenn Physiker das nicht so sehen. Eine Lichtteilchenwelle ist etwas anderes als eine Magnetfeldwelle, ein Dauermagnet erzeugt ein Wellenfeld, aber er emittiert keine Energie dabei, sonst wäre er in kürzester Zeit kein Magnet mehr.

Gruß

möbius
29.04.08, 16:13
----



möbius,
----
mc² = hf
----
Das bezieht sich auf die Photonenenergie. Jetzt bleibt es Aufgabe der Physik festzustellen, welcher Energieanteil als Teilchen und welcher als Vakuumwellenergie vorliegt. Ich hatte das mit Jan Ulrich auf dem Fahrrad zu verdeutlichen versucht; das Teilchen ist der Jan samt Fahrrad, die Welle die von ihm verursachte Luftbewegung. Ohne Teilchen keine Energie transportierende Welle, das gilt auch fürs Vakuum, auch wenn Physiker das nicht so sehen. Eine Lichtteilchenwelle ist etwas anderes als eine Magnetfeldwelle, ein Dauermagnet erzeugt ein Wellenfeld, aber er emittiert keine Energie dabei, sonst wäre er in kürzester Zeit kein Magnet mehr.

Gruß
Na denn: Viel Glück bei der Feststellung, "welcher Energieanteil als Teilchen und welcher als Vakuumwellenenergie vorliegt"!!!

Ist denn innerhalb der Forschergemeinschaft der Physiker der Begriff der "Vakuumwellenernergie" schon akzeptiert und durch Experimente empirisch bestätigt worden:confused: :confused: :confused:

rafiti
29.04.08, 17:23
@uwebus
Was soll an deiner Substanz empirisch dran sein? Mal davon abgesehen war das Weltbild vor 2000 Jahren ganz anders als heute. Eine Messung sagt doch mehr aus als 100 Philosophen, so sind die Regeln nun mal hier...

gruss
rafiti

Uranor
29.04.08, 18:26
Ist denn innerhalb der Forschergemeinschaft der Physiker der Begriff der "Vakuumwellenernergie" schon akzeptiert und durch Experimente empirisch bestätigt worden:confused: :confused: :confused:
Nur beim Vakuum_Real_Cowboy ist das akzeptiert. Die Welt weiß noch rein gar nix von ihrem Schrecken. :rolleyes: :rolleyes: :D

ingeniosus
29.04.08, 21:11
Und noch weiter, Gleichverteilungssatz: Gesamtenergie = Schwingungsenergie + Kinetische Energie + Potentielle Energie + Rotationsenergie +...

Und für Photonen Ekin = Eschwing = mc²/2 = hf/2 ,

und Egesamt = mc² = hf ? :D :D :D

Das ganze hat nur einen Hacken : Photonen haben keine Masse!

criptically
29.04.08, 21:12
Aber hatte das nicht Louis de Broglie durch seine Gleichung "irgendwie"
gelöst???
mc² = hf
Was ist denn vor dem Hintergrund dieser Gleichung noch "un-gelöst":confused:
möbius

Die Gleichung gibt die Gesamtenergie an. Es fehlt noch die Aufteilung in den Schwingungsanteil und in kinetischen Anteil.

Das kann einfach verstanden werden, wenn man einen Oszillator betrachtet. Ruht der Oszillator, dann besitzt er nur Schwingungsenergie. Wird der Oszillator dagegen bewegt, hat er kinetische Energie und Schwingungsenergie.

Jeder Weiß, dass Photonen schwingen sonst hätten sie keine Frequenz. Und sie bewegen sich auch noch...

Aber bis Physiker so etwas begreifen... :D :D :D

mfg

criptically
29.04.08, 21:18
... Photonen haben keine Masse!

Das ist leider ein (absichtlicher) "Fehlschluss" um RT zu retten!

Hätten sie keine Masse gebe es keine Ablenkung im Gravitationsfeld (Gravitationskraft zieht nur Massen an).

mfg

criptically
29.04.08, 21:23
...
Die vernichtete kinetische Energie hat sich in Drehimpuls umgewandelt, wobei die beiden Teilchenhälften nun mit gegenläufigem Spin aufeinander liegen und sich damit abstoßen, es sind zwei Teilchen entstanden, ein Positron und ein Elektron oder - im großen Maßstab - ein Proton und ein Antiproton
...
Gruß

Ein Antiproton hat noch niemand gesehen!

mfg

orca
29.04.08, 21:37
Das ist leider ein (absichtlicher) "Fehlschluss" um RT zu retten!

Hätten sie keine Masse gebe es keine Ablenkung im Gravitationsfeld (Gravitationskraft zieht nur Massen an).




mfg

Die Ablenkung im Gravitionsfeld ist unabhängig von der Masse der Photonen, in der klassischen Formel taucht die Photonenmasse nicht auf.

Die Formel lautet n(r) = c / c' = 1 / [1 - 2Gm /(rc²)]

Dabei ist m die Masse des Gravitationszentrums.

Die doppelte Ablenkung im Gravitationsfeld beruht nicht auf der Gravitationskraft, sondern auf der Änderung der Frequenz und der Wellenlänge des Lichts durch das Gravitationsfeld

Das kann dir ingenius mit seinem auf die Naturwissenschaft losgelassenen messerscharfen Struktur- und Systemverstand sicher bestätigen!

MfG
Orca

criptically
29.04.08, 21:59
Die Ablenkung im Gravitionsfeld ist unabhängig von der Masse der Photonen, in der klassischen Formel taucht die Photonenmasse nicht auf.

Die Formel lautet n(r) = c / c' = 1 / [1 - 2Gm /(rc²)]

Dabei ist m die Masse des Gravitationszentrums.

Die doppelte Ablenkung im Gravitationsfeld beruht nicht auf der Gravitationskraft, sondern auf der Änderung der Frequenz und der Wellenlänge des Lichts durch das Gravitationsfeld

Das kann dir ingenius mit seinem auf die Naturwissenschaft losgelassenen messerscharfen Struktur- und Systemverstand sicher bestätigen!

MfG
Orca

Ich weiß noch nicht, ob 1/c² in der obigen Formel etwas mit Photonenmasse zu tun haben könnte (m=E/c²) ?

mfg

orca
29.04.08, 22:23
Wenn die Lichtgeschwindigkeit von der Photenmasse abhängig wäre, dann müßte in der Formel für die Lichtablenkung im Gravitationsfeld die Photenmasse erscheinen.

Wenn es vor Ort schnelle (leichte) und langsame (schwere) Photonen geben würde, dann könnte man das leicht feststellen, weil sie bei gleichem Lichtweg unterschiedliche Laufzeiten hätten.

Nach der klassischen Mechanik ist die Lichtgeschwindigkeit in Inertialsystemen konstant, aber nicht bewegungsinvariant und nicht an jedem Ort gleich, weil von der Gravitation abhängig.

MfG
Horst

JimWilson
29.04.08, 23:22
Moin Horst,
Nach der klassischen Mechanik ist die Lichtgeschwindigkeit in Inertialsystemen konstant, aber nicht bewegungsinvariant und nicht an jedem Ort gleich, weil von der Gravitation abhängig.
Eine der besten falschen Antworten!

In der 'klassischen Mechanik" (ohne RT) gibt es keine Konstanz der Lichtgeschwindigkeit, da diese in diesem Gebiet unendlich ist. So, wie jede andere Signalausbreitungsgeschwindigkeit. Deshalb hieß Einsteins ursprüngliche Lichtgeschwindigkeit auch V=∞.

Und wie, überhaupt, gelingt es einem Inertialsystem inertial zu sein, wenn es von der Gravitation abhängt?

Bei deiner Antwort bedarf es weder Inertialsystemen noch einer Konstanz von Irgendetwas, sondern es gilt Absolutes.

rafiti
29.04.08, 23:23
Hätten sie keine Masse gebe es keine Ablenkung im Gravitationsfeld (Gravitationskraft zieht nur Massen an).


Ja, auch ich seh das so ähnlich immer wieder aufs Neue, dazu muss ich dir erstmal die Vorgeschichte erzählen: ich dünge mein Feld gerne mit Kuhmist, weil ich selbst auch Melonen esse, und mit Kunstdüngern schmecken die später nicht so gut. Und dabei ist mir aufgefallen, dass Kuhmist nur Fliegen anzieht, ich wollt's dir nur sagen, dass du bescheid weißt. :)

gruss
rafiti

uwebus
30.04.08, 12:59
möbius,
----
Ist denn innerhalb der Forschergemeinschaft der Physiker der Begriff der "Vakuumwellenernergie" schon akzeptiert und durch Experimente empirisch bestätigt worden?
----
Für Physiker ist es meßbar, daß Licht in Wellenform das Vakuum durchquert und daß es einen Impuls trägt. Also ist Licht impulstragende Vakuumwellenenergie, oder?

Was für Physiker bisher nicht meßbar ist, ist das Vakuum konstituierende Etwas, viele glauben ja noch immer an das existierende volumenhaltige Nichts, an einen “Raum als solchen“ als leeren Aufbewahrungsbehälter fürs Universum. Das Problem der Physik besteht nicht in der Wellenerscheinung des Lichtes, sondern im Unverständnis des Vakuums, auch wenn der oberkluge Forumsguru Uranor hierzu wieder mal seine unproduktiven dummen Sprüche abläßt.

Solange Physiker nicht an die Frage des Vakuums herangehen, werden sie auch das Phänomen der Gravitation nicht verstehen, Licht und Gravitation werden erst verständlich, wenn man Vakuum als physisches OBJEKT betrachtet.


rafiti,
----
Was soll an deiner Substanz empirisch dran sein?
----
Gar nichts, sie ist mein Ursprungspostulat. Der Gläubige beruft sich auf Gott, worauf Physiker sich berufen, habe ich bis heute noch nicht herausbekommen, anscheinend gibt es da noch keine einheitliche Meinung, weil wohl Physiker generell keine Warum- und Woher-Fragen stellen. Deshalb können sie ja auch so herrlich unsinnige Modelle bauen wie ein endliches hüllenloses expandierendes Universum oder volumenlose kräfteerzeugende Strings und ähnlichen Unsinn.

Weil ich nicht an Götter glaube, baue ich ein Substanz-Modell als Perpetuum mobile, denn ohne Götter gibt es keinen Anfang und kein Ende des Seins. Physiker lassen mit dem Urknall die Zeit beginnen und das Universum irgendwann in schwarzen Löchern verkommen, das ist praktizierte Schizophrenie, wenn man gleichzeitig den Energieerhaltungssatz predigt.

Was ich mit meinem Substanz-PM Arche versuche, ist zu einer Übereinstimmung zwischen Modell und Beobachtung zu gelangen, um festzustellen, ob die Annahme eines einzigen das Universum konstituierenden Mediums gangbar ist. Und bisher scheint sie es zu sein, deshalb stelle diese Idee auch hier vor. Es könnte ja sein, daß auch der eine oder andere Physiker sich Gedanken macht über Dinge, die etwas über die praktische Verwertbarkeit physikalischer Erkenntnisse hinausgehen.

Gruß

pauli
30.04.08, 13:31
möbius,
----
Ist denn innerhalb der Forschergemeinschaft der Physiker der Begriff der "Vakuumwellenernergie" schon akzeptiert und durch Experimente empirisch bestätigt worden?
----
Für Physiker ist es meßbar, daß Licht in Wellenform das Vakuum durchquert und daß es einen Impuls trägt. Also ist Licht impulstragende Vakuumwellenenergie, oder?
lol
Und wie nennt man den Fall, wenn Licht in Wellenform Glas durchquert, impulstragende Glaswellenenergie?

Gutes Beispiel dafür, wie aus eigenen Begriffen eigene Realitäten entstehen

uwebus
30.04.08, 13:56
criptically,
----
Ein Antiproton hat noch niemand gesehen!
----
http://de.wikipedia.org/wiki/Antiproton
“Das Antiproton wurde erstmals 1955 im Lawrence Berkeley National Laboratory mit einem Protonenstrahl von 6,3 GeV, der auf ein Kupfertarget traf, künstlich erzeugt.“

orca,
----
Die Ablenkung im Gravitationsfeld ist unabhängig von der Masse der Photonen, in der klassischen Formel taucht die Photonenmasse nicht auf.

Die doppelte Ablenkung im Gravitationsfeld beruht nicht auf der Gravitationskraft, sondern auf der Änderung der Frequenz und der Wellenlänge des Lichts durch das Gravitationsfeld
----
Ist es nicht phantastisch, was Physiker so alles wissen? Wenn man sie fragt, was Gravitation ist und wie sie zustande kommt, müssen sie passen, aber sie wissen ganz genau, was in einem Gravitationsfeld mit Licht passiert, obwohl sie auch das Licht noch nicht richtig verstehen.

Nix genaues weiß man nicht, aber man tut so, als wisse man, um nicht blöd dazustehen.

----
Bezugssystem, die ruhen oder die sich geradlinig gleichförmig bewegen, weil sie frei von äußeren Kräften sind, nennt man Inertialsysteme.
----
Solche Systeme gibt es aber im Universum nicht, sondern nur in den Köpfen von Physikern. Geradlinige gleichförmige Bewegung ohne Krafteinwirkung ist nun mal in Gravitationsfeldern nicht möglich, es sei denn, man betrachtet einen mathematischen Punkt. Und Ruhe gibt es auch nirgendwo im Universum, alles bewegt sich gegeneinander. Inertialsysteme sind Näherungssysteme z.B. bei der Betrachtung eines geostationären Satelliten, bei dem das sich ändernde G-Potential bezogen auf dessen Ausdehnung vernachlässigbar ist.

----
Aus dem galileischen Relativitätsprinzip folgt, daß man auch mit Hilfe des Lichts nicht zwischen Ruhe und geradlinig gleichförmiger Bewegung unterscheiden kann.
----
Es gibt im Universum keine von äußeren G-Kräften freie physische Entitäten, nur Engel fliegen kräftefrei und nur Götter ruhen! Das hat Herr Galilei wohl noch nicht gewußt. Will ich geradlinig gleichförmig an der Erde vorbeifliegen, muß ich ständig beschleunigen, und zwar an jedem Ort unterschiedlich. Und auf einem radialen Kurs gilt das genauso. Selbst der freie Fall ist nicht kräftefrei aufgrund unterschiedlichen G-Potentials über die radiale Abmessung des fallenden Körpers gemessen. Ein frei fallender Körper wird langgezogen, je näher er einer Masse kommt.

pauli,
----
Und wie nennt man den Fall, wenn Licht in Wellenform Glas durchquert, impulstragende Glaswellenenergie?
----
Glaslichtwellenenergie! Das wäre nicht mal falsch, denn die Lichtwelle im Glas ist ja von der Lichtwelle im Vakuum verschieden. Physiker drehen es halt nur um, sie sprechen von einer Lichtwelle im Vakuum, von einer Lichtwelle im Glas, im Wasser etc.

Vakuumlichtwelle = Lichtwelle im Vakuum, ersteres spart 2 Buchstaben und 2 Leerzeichen!


Gruß

pauli
30.04.08, 14:17
orca,
----
Die Ablenkung im Gravitationsfeld ist unabhängig von der Masse der Photonen, in der klassischen Formel taucht die Photonenmasse nicht auf.

Die doppelte Ablenkung im Gravitationsfeld beruht nicht auf der Gravitationskraft, sondern auf der Änderung der Frequenz und der Wellenlänge des Lichts durch das Gravitationsfeld
----
Ist es nicht phantastisch, was Physiker so alles wissen? Wenn man sie fragt, was Gravitation ist und wie sie zustande kommt, müssen sie passen, aber sie wissen ganz genau, was in einem Gravitationsfeld mit Licht passiert, obwohl sie auch das Licht noch nicht richtig verstehen.
Orca versteht von Physik genausoviel wie du, ich, Verona Feldbusch, Tim Mälzer oder Karl Dall.

Uranor
30.04.08, 14:47
Ist es nicht phantastisch, was Physiker so alles wissen? Wenn man sie fragt, was Gravitation ist und wie sie zustande kommt, müssen sie passen, aber sie wissen ganz genau, was in einem Gravitationsfeld mit Licht passiert, obwohl sie auch das Licht noch nicht richtig verstehen.
Aber ausgerechnet Nichtforscher wissen ganz genau, wie die Natur aufgebaut ist. Merxwürdig nur, wen sie tatsächlich mal auf eine Frage antworden, erzählen sie lauter Müll, der mit der Natur gar nix zu tun hat.

;) :p


@pauli, du hast eine wahre Gabe, aus den Posts teffsicher das wesentlich rauszuziehen. Und es ist dann auch die Dosis, die man grad noch schmunzelnd ertragen kann. :)

Gruß Uranor

orca
30.04.08, 15:15
orca,

Inertialsysteme sind Näherungssysteme z.B. bei der Betrachtung eines geostationären Satelliten, bei dem das sich ändernde G-Potential bezogen auf dessen Ausdehnung vernachlässigbar ist.

Gruß

Ein geostationären Satellit ist kein Inertialsystem, weil er eine Winkelgeschwindigkeit von omegaF = 0,727 *10^-4 s^-1 hat und deshalb eine Führungsbeschleunigung im Gravitationsfeld der Erde von aF= r*omegaF aufweist, wenn r der senkrechte Abstand zur Drehachse der Erde ist.

Ob "das sich ändernde G-Potential bezogen auf dessen Ausdehnung vernachlässigbar ist" ist ganz unerheblich. Auf einer geostationären Umlaufbahn ändert sich übrigens das G-Potential überhaupt nicht.

Die Unterscheidung von Inertial- und Nichtinertialsystem ist wichtig, weil die klassischen Gesetze der Kinematik und Kinetik nur in idealisierten Inertialsystemen eine einfache Form annehmen. In manchen Fällen kann man es sich leicht machen und von diesen einfachen, idealisierten Fällen ausgehen, wenn der dadurch auftretende Fehler gering genug bleibt.

Siehe hierzu in einem Fachbuch der klassischen Mechanik die Kapitel "Kinematik der Relativbewegung" und "Kinetik der Relativbewegung", in Büchern der relativistischen Popphysik steht zu diesem Thema viel Unsinn.:D

MfG
Orca

pauli
30.04.08, 15:21
Und es ist dann auch die Dosis, die man grad noch schmunzelnd ertragen kann. :)
Hi Uranor,

ja, man muss sich schon anstrengen die Balance zu wahren, schliesslich weiß man nie, ob Verona Feldbusch sich mal in die Diskussion einmischen wird um zu klären, in welchem schwarzen Loch ihr Ehegatte die ganze Kohle versenkt hat :)

orca
30.04.08, 15:40
Die doppelte Ablenkung im Gravitationsfeld beruht nicht auf der Gravitationskraft, sondern auf der Änderung der Frequenz und der Wellenlänge des Lichts durch das Gravitationsfeld.

n(Psi) = 1 / (1 + 2 Psi /c²)

n(r) = 1 / [1 - 2 G m /(r c²)]



Mit der wissenschaftlichen Leistung und Qualifikation von uwebus, Urinator und Pauli kann F. Spieweck natürlich nicht mithalten.

Frequenzänderung der Cs-atome, Zwillingsparadoxon, Lichtablenkung im Gravitationsfeld, Periheldrehung der Planetenbahn, Frequenzverschiebung von Spektrallinien, Lense-Thirring-Effekt, Sagnac-Effekt lassen sich nach Dr. rer. nat. Dipl.-Phys. Frank Spieweck auf einem "klassischen metrologischen" bzw. "energetischen Weg" erklären, wobei nur Elementarmathematik erforderlich ist.

[von 1966 bis 1999 Wissenschaftlicher Laborleiter/Regierungsdirektor in der Physikalisch-Technischen Bundesanstalt in Braunschweig,
Träger des Helmhotzpreises von 1981]

www.helmholtz-fonds.de/preis.htm

Spieweck, Frank:
Relativistische Effekte - neu erklärt
Aachen: Shaker, 2000
ISBN 3-8265-7889-9


Literaturliste der Physikalisch-Technischen Bundesanstalt in Braunschweig

Der 54 Seiten umfassende PTB-Bericht von Frank Spieweck und Horst Bettin "Methoden zur Bestimmung der Dichte von Festkörpern und Flüssigkeiten" PTB-W-46, Braunschweig, Oktober 1991, 2. Nachdruck Februar 1998, mit Tabellen und Formeln zur Wasserdichte, Quecksilberdichte, Dichte von Ethanol-Wasser-Mischungen, Dichte von wässrigen Saccharoselösungen sowie der Luftdichte ist vergriffen. Eine aktualisierte Neuauflage wird vorbereitet.


Veröffentlichungen


W. Gorski, H.-G. Toth: Destilliertes Wasser als Dichtereferenzmaterial - Die elektrische Leitfähigkeit als Kriterium seiner Güte. PTB-Mitteilungen 98 (1988), S. 324-325

F. Spieweck, H. G. Toth: Dichtemessungen mit einer untergetauchten Waage. PTB-Jahresbericht 1988, S. 155


K. Sommer, H. Adametz, H. Fehlauer: Normalproben zur Darstellung und Weitergabe der Einheit der Dichte. Metrologische Abhandlungen, Berlin 9 (1989) 1, S. 65-76


F. Spieweck, A. Kozdon, H. Wagenbreth, H. Toth, D. Hoburg: A Computer-controlled Solid-density Measuring Apparatus. PTB-Mitteilungen 100 (1990), S. 169-173


H. Bettin, F. Spieweck: Die Dichte des Wassers als Funktion der Temperatur nach Einführung der Internationalen Temperaturskala von 1990. PTB-Mitteilungen 100 (1990), S. 195-196


H. Bettin, F. Spieweck: Die Dichte wäßriger Saccharoselösungen nach Einführung der Internationalen Temperaturskala von 1990 (ITS-90). PTB-Mitteilungen 100 (1990), S. 369-371


F. Spieweck, H. Bettin, H. Toth: Einfache Dichtebestimmung mit einer oberschaligen Waage. wägen + dosieren (6/1990), S. 20-25


H. Bettin, F. Spieweck: A Revised Formula for the Calculation of Alcoholometric Tables. PTB-Mitteilungen 100 (1990), S. 457-460


H. Bettin, F. Spieweck, H. Toth: A computer-operated fluid-density measuring device using a balance and two permanent magnets. Meas. Sci. Technol. 2 (1991), S. 1036-1038


P. Seyfried, P. Becker, A. Kozdon, F. Lüdicke, F. Spieweck, J. Stümpel, H. Wagenbreth, D. Windisch, P. De Bièvre, H. H. Ku, G. Lenaers, T. J. Murphy, H. S. Peiser, S. Valkiers: A determination of the Avogadro Constant. Z. Phys. B - Condensed Matter, 87 (1992), S. 289-298


A. Kozdon, F. Spieweck: Transferring the density unit to small silicon artifacts. PTB-Mitteilungen 102 (1992), S. 3-7


A. F. Kozdon, F. Spieweck: Determination of Differences in the Density of Silicon Single Crystals by Observing Their Flotation at Different Pressures. IEEE Trans. Instrum. Meas. 41 (1992), S. 420-426


F. Spieweck, H. Bettin: Review: Solid and liquid density determination. tm - Technisches Messen 59 (1992), S. 237-244; 285-292


R. Belac, H. J. Groß, H. Bettin, F. Spieweck: Volumenmeßgeräte für Laboratoriumszwecke. PTB-Prüfregeln, Band 21. Physikalisch-Technische Bundesanstalt 1992, S. 1-61


H. Bettin, F. Spieweck, H. Toth: Bestimmung der Oberflächenspannung mit Hilfe eines Glasstengels. PTB-Mitteilungen 103 (1993), S. 147-148


F. Spieweck: Sensors for Measuring Density and Viscosity. In: W. Göpel, J. Hesse, J. N. Zemel (eds): Sensors. A Comprehensive Survey; H. H. Bau, N. F. de Rooij, B. Kloeck (eds): Vol. 7 Mechanical Sensors, VCH Weinheim 1994, S. 359-372


H. Bettin, P. Seyfried+, F. Spieweck, H. Toth: A method for determining the density of small 28Si samples. PTB-Mitteilungen 104 (1994), S. 17-19


F. Spieweck: Das Internationale Einheitensystem - Entwicklung der Einheiten und heutiger Stand. Physikalisch-Technische Bundesanstalt, PTB-Bericht PTB-W-58, Braunschweig 1994, S. 1-21


......

F. Michel, K. Sommer, F. Spieweck: Untersuchungen zur Ermittlung der Meßunsicherheit von Kolbenhubpipetten mit Volumen von 1 µl bis 50 µl. PTB-Mitteilungen 105 (1995), S. 437-444


F. Spieweck: Viskositätsunabhängige Dichtemessung mit einem quer schwingenden Meßrohr. tm - Technisches Messen 63 (1996), S. 194-196


F. Spieweck: Festkörper, Dichtebestimmung. In: F. Kohlrausch, Praktische Physik, Bd. 1, 24. Aufl., B. G. Teubner, Stuttgart 1996, S. 363-365


F. Spieweck: Flüssigkeiten, Dichtebestimmung. In: F. Kohlrausch, Praktische Physik, Bd. 1, 24. Aufl., B. G. Teubner, Stuttgart 1996, S. 370-373 und Bd. 3, S. 347; 349-350


H. Bettin, M. Gläser, F. Spieweck, H. Toth, A. Sacconi. A. Peuto, K. Fujii, M. Tanaka, Y. Nezu: International Intercomparison of Silicon Density Standards. IEEE Trans. Instrum. Meas. 46 (1997), S. 556-559


P. De Bièvre, S. Valkiers, R. Gonfiantini, P. D. P. Taylor, H. Bettin, F. Spieweck, A. Peuto, S. Pettorruso, M. Mosca, K. Fujii, M. Tanaka, Y. Nezu, A. J. Leistner, W. J. Giardini: The Molar Volume of Silicon. IEEE Trans. Instrum. Meas. 46 (1997), S. 592-595


R. Belac, H. J. Groß, H. Bettin, F. Spieweck: Volumetric laboratory instruments. PTB Testing Instructions 21 (1997), S. 1-60


H. Bettin, A. Emmerich, F. Spieweck, H. Toth: Dichte wäßriger Glucose-, Fructose- und Invertzuckerlösungen (Density data for aqueous solutions of glucose, fructose and invert sugar). Zuckerindustrie 123 (1998), Nr. 5, S. 341-348

.......usw



--------------------------------------------------------------------------------
© Physikalisch-Technische Bundesanstalt
Erstellt am: 2004-04-01, letzte Änderung: 2007-07-24, Daniela Klaus



www.ptb.de/de/org/3/33/333/literatur.htm

pauli
30.04.08, 15:55
Mit der wissenschaftlichen Leistung und Qualifikation von Urinator und Pauli kann F. Spieweck natürlich nicht mithalten.
na das will ich meinen, Franz Spielwech ist nicht mehr im Rennen, wir schon

Querkopf
30.04.08, 16:03
Bei mir ist das mit der ART schon etwas her, aber wenn mich meine grauen Zellen nicht täuschen spricht Orca doch von Gravitationsrotverschiebung, bzw. von radialen lichtartigen Kurven.
Ich bin jetzt kein Experte und hab's auch nicht nachgerechnet, aber so falsch das ich das jetzt auf den ersten Blick sehen könnte ist es zumindest nicht.

Uranor
30.04.08, 16:29
Mit der wissenschaftlichen Leistung und Qualifikation von uwebus, Urinator und Pauli kann F. Spieweck natürlich nicht mithalten.
Ungebackener Witzbold, du spielst mit deinem Leben!

orca
30.04.08, 16:41
Bei mir ist das mit der ART schon etwas her, aber wenn mich meine grauen Zellen nicht täuschen spricht Orca doch von Gravitationsrotverschiebung, bzw. von radialen lichtartigen Kurven.
Ich bin jetzt kein Experte und hab's auch nicht nachgerechnet, aber so falsch das ich das jetzt auf den ersten Blick sehen könnte ist es zumindest nicht.

Was daran falsch ist, läßt sich leicht feststellen, im alten Forum ist nämlich der Inhalt des 7. Kapitels "Lichtablenkung im Gravitationsfeld" von F. Spieweck zu finden.



7. Lichtablenkung im Gravitationsfeld

Wie die relativistische Verkürzung einer Länge lässt sich auch die relativistische Verringerung einer Geschwindigkeit, wie insbesondere der Lichtgeschwindigkeit, nicht direkt beobachten. Während in der speziell‑relativistischen Physik eine Beurteilung der Lichtgeschwindigkeit gegenüber einem Inertialsystem K durch einen Beobachter in einem anderen Inertialsystem K' in Einheiten des Bezugssystems K', nicht möglich ist, da eine derartige Lichtgeschwindigkeit weder direkt noch indirekt messbar wäre, ist in der allgemein‑relativistischen Physik die Verringerung der Lichtgeschwindigkeit indirekt nachweisbar, weil aus ihr eine Lichtablenkung im Gravitationsfeld resultiert und z.B. Radarimpulse verzögert werden.

So konnte erstmalig im Jahre 1919 während einer Sonnenfinsternis experimentell festgestellt werden, dass Sternenlicht durch die Sonne um 1,7 Bogensekunden abgelenkt wird [Einstein 1969]. Dies zeigte, dass "das schon oft erwähnte Gesetz von der Konstanz der Vakuumlichtgeschwindigkeit, das eine der .. grundlegenden Annahmen der speziellen Relativitätstheorie bildet, keine unbegrenzte Gültigkeit beanspruchen kann. Eine Krümmung der Lichtstrahlen kann nämlich nur dann eintreten, wenn die Ausbreitungsgeschwindigkeit des Lichtes mit dem Orte variiert" [Einstein 1969]. Für einen "entfernten" Beobachter in einem gravitationsfeldfreien Euklidischen (Beobachter‑)Raum resultiert daher eine verringerte Lichtgeschwindigkeit im Gravitationsfeld [Einstein 1916], die von Einstein [1965] mit L bezeichnet wurde. Natürlich hängt die Geschwindigkeit aller elektromagnetischen Wellen, also auch von Radarimpulsen, von der Stärke des Gravitationspotentials längs des Weges ab, so dass Radarechos von Venus oder Mars die Erde verzögert erreichen sollten, wenn die Radarstrahlen in der Nähe des Sonnenrandes verlaufen [Shapiro 1964], siehe unten.

Wird in einem lokalen Nichtinertialsystem K, wo das Gravitationspotential F besteht, unter Verwendung von Einheiten des Bezugssystems K die Geschwindigkeit
(7.1) u = Ds/Dt
gemessen, so wird diese Geschwindigkeit gegenüber dem lokalen Nichtinertialsystem K von einem Beobachter im Bezugssystem K', wo das Gravitationspotential Psi = 0 ist, unter Verwendung von Einheiten seines Bezugssystems K' als verringerte Geschwindigkeit
(7.2) u' = Ds' / Dt'
beurteilt. Wie bereits im Abschnitt 2 erwähnt, bedeutet hier u' nicht ‑ wie sonst üblich ‑ die Geschwindigkeit gegenüber dem Bezugssystem K', sondern die in Einheiten des im Bezugssystem K' befindlichen Beobachters ausgedrückte Geschwindigkeit gegenüber dem Bezugssystem K.
Da u eine beliebige Geschwindigkeit sein kann, gilt auch für Vorgänge, die in einem lokalen Nichtinertialsystem K ‑ also z.B. am Sonnenrand ‑ mit der Geschwindigkeit
(7.3) u = c,
also mit Lichtgeschwindigkeit ablaufen, für einen Beobachter im Bezugssystem K' die verringerte ‑ von Einstein [1965) mit L bezeichnete ‑ Lichtgeschwindigkeit
(7.4) c' = Ds' / Dt'.

Wird jetzt angenommen, dass für eine elektromagnetische Welle "vor Ort", d.h. im Gravitationsfeld, eine Verkürzung der Wellenlänge gemäß der Beziehung (6.8) und eine Dehnung der Schwingungsdauer gemäß der Beziehung (5.9) gelten, so ergibt sich auf Grund der Gleichungen (7.1) bis (7.4) für einen Beobachter im Bezugssystem K' eine sogenannte doppelt verringerte Lichtgeschwindigkeit im Gravitationsfeld,
(7.5) c' = c (1 + 2 Psi / c² ),
aus der sich bekanntlich die Lichtablenkung im Gravitationsfeld berechnen lässt [Einstein 1916; Bagge; Spieweck 1988]. Hierbei kann angenommen werden, dass das Licht in einem Gravitationsfeld wie in einem Medium mit der (von Ort zu Ort ‑ ähnlich wie im Fall einer Luftspiegelung ‑ sich kontinuierlich ändernden) Brechzahl
(7.6) n(Psi) = c / c'
läuft [Bagge; Spieweck 1988]. Ein vom Wert n = 1 verschiedener Brechungsindex im interstellaren Raum erklärt natürlich auch den Gravitationslinseneffekt [D'Inverno].
Für den Brechungsindex in Abhängigkeit vom Gravitationspotential folgt aus den Beziehungen (7.5) und (7.6)
(7.7) n(Psi) = 1 / (1 + 2 Psi / c²).
Gemäß der Beziehung (2.15) ergibt sich dann für den Brechungsindex in Abhängigkeit vom Abstand r von einem Gravitationszentrum mit der Masse m
(7.8) n(r) = 1 / [1 - 2Gm/(r c²)].
Am Sonnenrand (r = 6,96.108 m, m = M = 1,989.1030 kg,
G = 6,674.1011 m³kg-1s-2 [Zimmermann, Weigert], c = 299792458 m/s) hat der Brechungsindex somit den Wert n = 1,0000042.

Die doppelte Lichtablenkung am Sonnenrand ergibt sich auch auf Grund folgender Überlegung. In Sonnennähe passieren einen betrachteten Ort zum einen pro Zeiteinheit (des Beobachters) weniger Wellenfronten als in größerer Entfernung, und zum andern verkürzt sich (für den Beobachter) der Abstand der Wellenfronten im Vergleich zu einem Ort in größerer Entfernung von der Sonne.

Experimentell bestätigt wurde die Verringerung der Ausbreitungsgeschwindigkeit elektromagnetischer Wellen in neuerer Zeit vor allem durch Laufzeitmessungen dicht an der Sonne vorbeigehender Radarsignale [Shapiro 1966; Fomalont, Sramek].


Spieweck, Frank:
Relativistische Effekte - neu erklärt
Aachen: Shaker, 2000 S.33-35
ISBN 3-8265-7889-9


MfG
Orca

orca
30.04.08, 17:02
Vorwort


Während die Ergebnisse der speziellen und allgemeinen Relativitätstheorie heute weitgehend bekannt und experimentell gut bestätigt sind, ist der Weg zum Verständnis insbesondere der allgemeinen Relativitätstheorie bisher nur Physikern mit umfangreichen mathematischen Kenntnissen vorbehalten.

Seit mit Hilfe des Mössbauereffekts gezeigt werden konnte, dass allgemein‑relativistische Effekte, die bisher im wesentlichen nur für die kosmische Physik von Bedeutung waren, bereits in einem Laboratorium auf der Erde die Energieniveaus von Atomkernen beeinflussen, besteht bei Schülern und Studenten ‑ mit nur "durchschnittlicher" mathematischer Vorbildung ‑ großes Interesse daran, nicht nur die speziell‑, sondern auch die allgemein*relativistische Physik zu verstehen. Daher bemühte sich der Autor, der 33 Jahre in der Metrologie (hauptsächlich mit der Entwicklung von Längen‑ und Frequenz‑Normalen) beschäftigt war, ‑ seit 1971 ‑ um eine einfache Herleitung der relativistischen Physik. Die Arbeit "Ein einfacher Zugang zur allgemein‑relativistischen Physik" aus dem Jahr 1988 ermöglicht bereits Gymnasial‑ und Hochschullehrern eine Erklärung der allgemein‑relativistischen Effekte auf der Grundlage der speziell-relativistischen Formeln für die Längenverkürzung und Zeitdehnung. Inzwischen ist es gelungen, die relativistischen Effekte auf einem von der Relativitätstheorie de facto unabhängigen ‑"metrologischen" ‑ Weg zu erhalten, wobei nur Kenntnisse der Elementarmathematik benötigt werden.

Für wertvolle Hinweise auf diesem Weg sei Herrn Prof. Dr. R.F. Werner (Institut für Mathematische Physik der TU Braunschweig) und Herrn Prof. Dr. J. Hesse (Institut für Metallphysik und Nukleare Festkörperphysik der TU Braunschweig) gedankt.

Braunschweig, im Sommer 2000 F. Spieweck


Spieweck, Frank:
Relativistische Effekte - neu erklärt
Aachen: Shaker, 2000 S.33-35
ISBN 3-8265-7889-9

pauli
30.04.08, 17:18
Hast du schon GOM und der messerschärfsten Logikerin aller Zeiten deine Entdeckung gemeldet? Sie wird sicher gerne begutachten, ob neue meteorologische Wege in die Kritikerliste aufgenommen werden können.

rafiti
30.04.08, 17:25
Spieweck, Frank:
Relativistische Effekte - neu erklärt
Aachen: Shaker, 2000 S.33-35
ISBN 3-8265-7889-9

Ja, die ist sowas von mathematisch schwierig, dass man an MIT gewesen sein muss, um sie zu verstehen, aber dank diverser Light-RT-Anleitungen, ist es uns nun endlich gelungen sie jedem zugänglich zu machen, und das ganze tolle daran ist, es geht ganz ohne eigenes Gehirn. Los geht's! Viel Spaß! Das ALDI-TEAM.


gruss
rafiti

uwebus
30.04.08, 18:48
Uranor,
----
Aber ausgerechnet Nichtforscher wissen ganz genau, wie die Natur aufgebaut ist.
----
Warum versuchst Du nicht einmal, einen Text zu lesen und auch zu verstehen? Ich schrieb doch gerade erneut:

Was ich mit meinem Substanz-PM Arche versuche, ist zu einer Übereinstimmung zwischen Modell und Beobachtung zu gelangen, um festzustellen, ob die Annahme eines einzigen das Universum konstituierenden Mediums gangbar ist. Und bisher scheint sie es zu sein, deshalb stelle diese Idee auch hier vor.

Ich spreche von Beginn an von einem Modell, nicht davon, den Stein der Weisen gefunden zu haben, das überlasse ich Leuten wie Dir.


pauli,
----
Orca versteht von Physik genausoviel wie du, ich, Verona Feldbusch, Tim Mälzer oder Karl Dall.
----
Es geht nicht darum, wie viel man von Physik versteht, sondern wie folgerichtig man zu denken in der Lage ist. Wenn Physiker meinen, eine mathematische Formel (die Riemanngeometrie) auf ein physisches Gebilde übertragen zu können, dann zeigen sie damit zwar, daß sie die Mathematik beherrschen, aber nicht in der Lage sind, ihrer eigenen Wahrnehmung zu vertrauen, denn diese Wahrnehmung beweist ihnen tagtäglich, daß ein endliches Volumen gerade deshalb endlich ist, WEIL es eine Begrenzungsfläche aufweist. Nur eine Begrenzungsfläche ermöglicht ein endliches Volumen. Und bei den Stringtheorien ist es ähnlich, hochgestochene Mathematik zwar, aber unsinnige Annahmen, denn ein Merkmal des Universums ist seine Ausdehnung und man kann machen was man will, mit ausdehnungslosen Strings läßt sich kein Universum bauen. Die unendliche Summe mathematischer Punkte, Linien oder Flächen hat nun mal das Volumen NULL und die physische Wirkung ist ebenfalls Null. Mathematik wirkt und errichtet nicht, sie beschreibt nur.

Ein weiteres typisches Beispiel physikalischer Unvernunft sind die sog. Inertialsysteme. Man kann zwar bei gering ausgedehnten Gebilden in dynamischem Gleichgewicht (Satelliten) deren eigene Gravitationswirkung sowie den Gravitationspotentialunterschied des Umfeldes vernachlässigen, aber deshalb sind es noch lange keine Inertialsysteme. Physiker rechnen häufig mit Idealisierungen oder Vereinfachungen, was vernünftig ist, tun dann aber so, als seien diese Idealisierungen real existent.

orca,
----
Daß der geostationäre Satellit einer Zentrifugalbeschleunigung unterliegt, ist richtig, sonst würde er abstürzen, aber diese Beschleunigung steht mit der Gravitation im Gleichgewicht, insofern kann man für den Satelliten als Ganzes von “beschleunigungsfrei“ reden. Aber der Satellit unterliegt einer in Richtung Erde ansteigenden Beschleunigung, d.h. innerhalb des Satelliten sind Gegenstände unterschiedlichen Beschleunigungen ausgesetzt, verdeutlichen läßt sich dies an einem Beispiel mit gedachten sehr genau gehenden Atomuhren:

http://uwebus.de/F/inertial-2.gif und http://uwebus.de/F/inertial-3.gif

----
.... wobei nur Elementarmathematik erforderlich ist.
----
Richtig, das ist auch meine Überzeugung. Es reichen die drei kartesischen Koordinaten sowie die Mathematik einschließlich Infinitesimalrechnung, um die Physis zu beschreiben.


Querkopf,
----
.... Gravitationsrotverschiebung....
----
Mit was mißt die Physik die Lichtfrequenzen? Mit Atomuhren! Und diese Dinger unterliegen dem G-Potential. Ich weise auch hier auf den Zirkelschluß der Physik hin:

Meter:
“Das Meter ist seit 1983 über die Lichtgeschwindigkeit mit der Einheit Sekunde verknüpft. Das Meter ist damit die Strecke, die Licht im Vakuum im 299.792.458sten Teil einer Sekunde durchquert. Das Meter ist damit streng genommen keine Basiseinheit mehr, sondern eine von der Sekunde abgeleitete Einheit.“

Sekunde:
“Ein bestimmtes Isotop von dem Alkalimetall Cäsium (133Cs) absorbiert zum Beispiel Strahlung, die genau 9.192.631.770 mal pro Sekunde schwingt (das sind 9,192 Gigahertz).“

Es ist also sehr schwer bis unmöglich, hier Frequenzverschiebungen des Lichtes und Frequenzgangunterschiede einer Atomuhr auseinander zu halten, denn das c(vakuum) = konstant ist ein Postulat. Mit was willst Du das beweisen?

Gruß

ingeniosus
30.04.08, 20:19
Photon ==> Elektron + Positron.

Die kinetische Energie eines Teilchens ist definiert als E=p²/2m.
Mit m = hf/c² und p=f·h/c erhält man für Photonen

Ekin=(fh/c)²/(2hf/c²)=fh/2=mc²/2.



Diese Ableitungen sind für mich nicht nachhvollziehbar.

1) E=p²/2m ist klassisch
2) m= hf/c² bewirkt bei mir einen Sturm an Widersprüchen
die Formel von Max Planck E = hf gilt nur für Photonen (masselos = masseneutralisiert) wird bisher als rein kinetische Energie gehandelt und kann m.E. nicht einfach mit der Gesamtenergie in der Einsteinsche Formel E = mc² gleichgesetzt werden. Wenn Sie eine Masse verwenden, muss differenziert werden zwischen Positronen und Elektronen oder von einer Massensumme gesprochen werden.
3) Ergebnis ist daher auch unlogisch : Ekinetisch eines Teilchens mit Masse grösser Null aber mit Lichtgeschwindigkeit (eindeutig gegen die ART von Einstein)

critically, bitte rechnen Sie das in einer ruhigen Minute nach!

Uli
30.04.08, 20:21
Das ist mir schon klar (ich habe solche Prozesse nämlich schon durchrechnen dürfen).

Deine Aussage war aber, das beim Zusammenstoß von zwei Photonen (wie es z.B. bei der Überlagerung von zwei Lichtquellen ständig der Fall sein sollte) ein Elektron - Positron – Paar frei wird.
Da habe ich auch für höhere Energien meine Zweifel. Du kannst mich aber gerne durch ein Feynmandiagram überzeugen.
...


Ich denke, so etwas gibt es schon. Denk dir das untenstehende Feynmandiagramm für die Delbrück-Streuung in der Senkrechten halbiert sodass das e- e+ Paar reell wird und ausläuft.
Ich wüsste nichts, was kinematisch gegen so einen Prozess spricht, wenn denn die beiden Photonen die entsprechende Schwellenergie zur Paarerzeugung überschreiten.

http://upload.wikimedia.org/wikipedia/commons/thumb/2/25/Delbruke_scattering_lowest.svg/120px-Delbruke_scattering_lowest.svg.png

Gruß,
Uli

criptically
30.04.08, 20:25
criptically,
----
Ein Antiproton hat noch niemand gesehen!
----
http://de.wikipedia.org/wiki/Antiproton
“Das Antiproton wurde erstmals 1955 im Lawrence Berkeley National Laboratory mit einem Protonenstrahl von 6,3 GeV, der auf ein Kupfertarget traf, künstlich erzeugt.“
...

Gruß

Das ist eine gewagte Behauptung, denn ein Antiproton müsste mit einem Proton sofort "zerstrahlen", was man aber nicht beobachten konnte!


Emilio Segrè erhielt 1959 zusammen mit Owen Chamberlain dafür den Physik-Nobelpreis „für ihre Entdeckung des Antiprotons“, dessen Existenz sie mithilfe von Clyde E. Wiegand und Thomas Ypsilantis nachweisen konnten. Mit einem Proton zerstrahlt es nicht – wie erwartet – sofort in elektromagnetische Strahlung, sondern es werden mehrere, freie Pi-Mesonen erzeugt.


mfg

Uli
30.04.08, 20:47
Das ist eine gewagte Behauptung, denn ein Antiproton müsste mit einem Proton sofort "zerstrahlen", was man aber nicht beobachten konnte!

mfg

Warum sollte es das müssen ?
Ein Proton (Antiproton) ist nun einmal nicht elementar, sondern besteht aus Quarks (Antiquarks) - Teilchen, die im Gegensatz zu Elektronen und Positronen auch stark wechselwirken. Es ist keineswegs gesagt, dass die rein elektromagnetische Wechselwirkung mit Entstehung von Photonen die Paarvernichtung eines Baryon-Antibaryon-Paares dominiert. Schliesslich heisst die starke Wechselwirkung nicht umsonst stark. :)

Bei Wechselwirkung von Baryonen entstehen nun einmal sehr gerne Pionen (gebundene Paare von Quarks-Antiquarks). Da spielt die starke Wechselwirkung eine große Rolle.

Aber sicherlich findet die von dir erwartete direkte elm. Vernichtung in Photonen auch gelegentlich statt - nur nicht so häufig wie die pionische Annihilation; es ist halt eine Frage der Wahrscheinlichkeiten.

Kein vernünftiger Mensch bezweifelt die Existenz von Antiprotonen: jedes elektrisch geladene Teilchen hat sein Antiteilchen.

Gruß,
Uli

ingeniosus
30.04.08, 20:50
Das ist leider ein (absichtlicher) "Fehlschluss" um RT zu retten!

Hätten sie keine Masse gebe es keine Ablenkung im Gravitationsfeld (Gravitationskraft zieht nur Massen an).

mfg

Ich bin erst seit 2 Wochen wieder dabei bei den Theoretischen Physikern. Daher denke ich vielleicht noch etwas klassisch.

Der Satz über die Rettung der RT ist m.E. schon etwas gewagt, aber Physik ist keine Religion....

Die Ablenkung im Gravitationsfeld beschäftigt mich auch schon seit Jahren. Einstein hatte damit ja bekanntlich 1919 seinen weltweiten Durchbruch. Ich denke dabei eventuell auch an ein elektromagnetisches Feld....

Uranor
30.04.08, 21:01
Uranor,
----
Aber ausgerechnet Nichtforscher wissen ganz genau, wie die Natur aufgebaut ist.
----
Warum versuchst Du nicht einmal, einen Text zu lesen und auch zu verstehen? Ich schrieb doch gerade erneut:

Was ich mit meinem Substanz-PM Arche versuche, ist zu einer Übereinstimmung zwischen Modell und Beobachtung zu gelangen, um festzustellen, ob die Annahme eines einzigen das Universum konstituierenden Mediums gangbar ist. Und bisher scheint sie es zu sein, deshalb stelle diese Idee auch hier vor.

Ich spreche von Beginn an von einem Modell, nicht davon, den Stein der Weisen gefunden zu haben, das überlasse ich Leuten wie Dir.
Hattu gepennt? Ich hatte dein Post nicht gelesen und fand es köstlich, dass @pauli mit Treffsicherheit immer wieder besondere Leckerli rauszugreifen versteht. Ich hoffe mal einfach, du hast keine Probleme damit. Hier noch mal der zitierte Text:

Ist es nicht phantastisch, was Physiker so alles wissen? Wenn man sie fragt, was Gravitation ist und wie sie zustande kommt, müssen sie passen, aber sie wissen ganz genau, was in einem Gravitationsfeld mit Licht passiert, obwohl sie auch das Licht noch nicht richtig verstehen.
Ün nü? :p Lass doch mal die Physiker in Ruh, wenn du ihnen schon nicht das Wasser reichen kannst. Kommst mir ähnlich vor wie der Locus. Der tut immer wunder wie schlau, hat aber noch nicht mal die einfachste Ebene der Relativität kapiert. Bei sowas :D ich eben. Ich hoffe, du hast keine Probleme damit. Falls doch, liegt es an dir. nicht immer Millionen Leute zu schmähen, die du gar nicht kennst. Sehr viele haben im Gegensatz zu dir was rausgefunden und passende Modelle entwickelt.

"die Annahme eines einzigen das Universum konstituierenden Mediums" gehört zu den Grundannahmen der Forschung. Man nennt es physikalisch nicht Medium, weil es physikalisch nicht Medium ist. Man nennt es Quantenvakuum (QV), weil es QV ist. Schon wieder vergessen, nie gelernt oder ignoriert? ;)

Bring's halt nicht zu knüppeldick und Gruß
Uranor

criptically
30.04.08, 21:04
... Schliesslich heisst die starke Wechselwirkung nicht umsonst stark. :)

...

Kein vernünftiger Mensch bezweifelt die Existenz von Antiprotonen: jedes elektrisch geladene Teilchen hat sein Antiteilchen.

Gruß,
Uli

1. Starke WW ist nur eine em-WW! Siehe Yukawa Potential.
Das Yukawa-Potential (auch abgeschirmtes Coulomb-Potential genannt). http://de.wikipedia.org/wiki/Yukawa-Potential

2. Es gibt keine Antiteilchen, sondern nur 3 stabile Teilchen: Proton, Elektron und Positron.

mfg

Uli
30.04.08, 21:08
1. Starke WW ist nur eine em-WW! Siehe Yukawa Potential.


2. Es gibt keine Antiteilchen, sondern nur 3 stabile Teilchen: Proton, Elektron und Positron.

mfg

Sorry, auf so einen Schwachfug antworte ich nicht. Lies halt einfach mal was anstatt immer nur drauflos zu plappern. Die unterschiedlichen Eigenschaften der 4 fundamentalen Wechselwirkungen samt den entsprechenden experimentellen Hinweisen kannst du in jeder Einführung in die Teilchenphysik - und sicher auch im Netz - leicht finden, z.B.
http://www-ekp.physik.uni-karlsruhe.de/~wagner/tp/Wi06/

ingeniosus
30.04.08, 21:10
Die Ablenkung im Gravitionsfeld ist unabhängig von der Masse der Photonen, in der klassischen Formel taucht die Photonenmasse nicht auf.

Die Formel lautet n(r) = c / c' = 1 / [1 - 2Gm /(rc²)]

Dabei ist m die Masse des Gravitationszentrums.

Die doppelte Ablenkung im Gravitationsfeld beruht nicht auf der Gravitationskraft, sondern auf der Änderung der Frequenz und der Wellenlänge des Lichts durch das Gravitationsfeld

Das kann dir ingenius mit seinem auf die Naturwissenschaft losgelassenen messerscharfen Struktur- und Systemverstand sicher bestätigen!

MfG
Orca

Danke Orka für ihre Erwähnung, eben habe ich einen echten messerscharfen Schluss von Ihnen gelesen.

Die Änderung der Frequenz (Wellenlänge) ist eine Änderung der Kinetischen Energie. Es bleibt jetzt immer noch die Frage, wie kann Gravitation das bewirken?

Licht als EM-Welle kann nur durch ein EM-Feld abgelenkt werden. Gravitation nur durch Gravitation.

Andere Theorien wären ja ein Beweis von Übergang von Gravitation zu Elektromagnetismus....oder umgekehrt("Neue Weltformel" usw...).

criptically
30.04.08, 21:19
Diese Ableitungen sind für mich nicht nachhvollziehbar.

1) E=p²/2m ist klassisch
2) m= hf/c² bewirkt bei mir einen Sturm an Widersprüchen
die Formel von Max Planck E = hf gilt nur für Photonen (masselos = masseneutralisiert) wird bisher als rein kinetische Energie gehandelt und kann m.E. nicht einfach mit der Gesamtenergie in der Einsteinsche Formel E = mc² gleichgesetzt werden. Wenn Sie eine Masse verwenden, muss differenziert werden zwischen Positronen und Elektronen oder von einer Massensumme gesprochen werden.
3) Ergebnis ist daher auch unlogisch : Ekinetisch eines Teilchens mit Masse grösser Null aber mit Lichtgeschwindigkeit (eindeutig gegen die ART von Einstein)

critically, bitte rechnen Sie das in einer ruhigen Minute nach!

1) E=p²/2m ist sowohl klassisch als auch quantenmechanisch (Hamiltonoperator) http://de.wikipedia.org/wiki/Hamiltonoperator#Beispiele

2) Die Formel E = mc² stamt nicht von Einstein sondern von Poincaré (1900). Einstensche Formel lautet E ≈ mc² .

Bei der Massensumme muss der Massendefekt berücksichtigt werden, so dass immer m+m < 2m wird.

3) SRT und ART sind falsch, deswegen diskutieren wir ja hier.

mfg

Lambert
30.04.08, 21:43
Andere Theorien wären ja ein Beweis von Übergang von Gravitation zu Elektromagnetismus....oder umgekehrt("Neue Weltformel" usw...).

sqt ergibt den (durch u.a. SRT stabilisierten) Übergang von Gravitation zu Elektromagnetismus. Es ist ein mengentheoretischer Zusammenhang.

Gruß,
Lambert

criptically
30.04.08, 21:48
...

Die Ablenkung im Gravitationsfeld beschäftigt mich auch schon seit Jahren. Einstein hatte damit ja bekanntlich 1919 seinen weltweiten Durchbruch. Ich denke dabei eventuell auch an ein elektromagnetisches Feld....

Für Lichtablenkung im Gravitationsfeld sind meiner Meinung nach, zwei Effekte verantwortlich: Lichtbrechung wegen gravitationsabhängiger LG und Photonen-Anziehung im Gravitationsfeld.

mfg

Uranor
30.04.08, 21:52
salve ingeniosus!
Die Ablenkung im Gravitationsfeld beschäftigt mich auch schon seit Jahren. Einstein hatte damit ja bekanntlich 1919 seinen weltweiten Durchbruch. Ich denke dabei eventuell auch an ein elektromagnetisches Feld....

Danke Orka für ihre Erwähnung, eben habe ich einen echten messerscharfen Schluss von Ihnen gelesen.

Die Änderung der Frequenz (Wellenlänge) ist eine Änderung der Kinetischen Energie. Es bleibt jetzt immer noch die Frage, wie kann Gravitation das bewirken?
Die Gravitation bewirkt das gar nicht. Als Ergebnis bekämen wir Rotverschiebung an jeder Gravitationslinse. Beim Verlassen der Linse wäre die reduzierte Energie festgeschrieben. Denn worauf würde nun eine Erhöhung auf den nichtverschobenen Wert beruhen, wie es nun mal beobachtet wird? Rotverschiebung beobachten wir einfach auf große Distanzen. Nach den Auswertungen sind gerade die enormen Weiten zwischen den Galazien für die Verschiebungen verantwortlich. Vernünftig erklärbar dürfte das nur mit Expansion sein.

Licht als EM-Welle kann nur durch ein EM-Feld abgelenkt werden. Gravitation nur durch Gravitation.
Nicht unbedingt. Eine physikalische Erklärung für die EM-Ablenkung konnte (bisher) nicht gelingen. Ergo wird man nur zur geometrisch beobachteten Ablenkung stehen können/müssen. Falls die Gravitation fluktuationen-basiert ist, ist die Forschung derzeit (noch) nicht in der Lage, drauf eingehen zu können. Ein kleiner, ansich unbedeutender Paradigmenwechsel wird sicher erst vollzogen werden müssen. Es wurde niemals und nirgends festgestellt, dass Gravitation tatsächlich anziehend wirkt. Vom virtuellen Geschehen ausgehend wäre Druck zu erwarten. Alle anstehenden Fragen sollten sich in Verständnis auflösen, wenn die Zusamenhänge tatsächlich erkannt werden könnten. Derzeit suchen nur ganz wenige danach.

Ja. Was in jedem Fall bleibt, ist die geometrische Auslenkung, eine regelrechte Raumkrümmung. Im Rahmen der ART rechnen Fachleute nur mit der Krümmung und kommen offenbar damit problemlos klar.

Gruß Uranor

ingeniosus
30.04.08, 21:58
2) Die Formel E = mc² stamt nicht von Einstein sondern von Poincaré (1900). Einstensche Formel lautet E ≈ mc² .

Bei der Massensumme muss der Massendefekt berücksichtigt werden, so dass immer m+m < 2m wird.

3) SRT und ART sind falsch, deswegen diskutieren wir ja hier.



Hallo Mr. Einstein New!

Die Poincare'sche Zeitparallelität zu Einstein ist bekannt.

Also SRT und ART sollten einmal studiert werden und dann deren Falschheit echt auch wissenschaftlich belegt werden.

Bisher stimmen alle Berechnungen der Relativität, die gesamte Raumfahrt wäre nicht möglich gewesen, hätten wir keine SRT oder ART.

Sie dürfen plappern, was Sie wollen, nur müssen Sie auch allmählich lernen, seriös wissenschaftlich zu werden!

Nichts gegen neue Ideen......mich phasziniert die Paarbildung des Photons derzeit auch....gerade in der Nähe der Absoluten Temperatur zeigt das Photon ja auch besonderes Verhalten.....aber alles exakt und logisch sauber.....

orca
30.04.08, 22:03
Danke Orka für ihre Erwähnung, ....



Keine Ursache, habe ich doch gern gemacht!;)


Die Änderung der Frequenz (Wellenlänge) ist eine Änderung der Kinetischen Energie. Es bleibt jetzt immer noch die Frage, wie kann Gravitation das bewirken?


Wie die potentielle Energie eine Änderung der Frequenz bewirkt,
ergibt sich aus der Planckschen Formel E = hf.


Roman Sexl und Herbert K. Schmidt schreiben in ihrem Buch auf Seiten 48-52

R. Sexl / H. K. Schmidt: Raum - Zeit - Relativität
3. durchges. Aufl., Nachdr. - Braunschweig;
Wiesbaden: Vieweg, 1991



5.5 Uhren im Schwerefeld

(...) Wir werden im folgenden mit verhältnismäßig einfachen Überlegungen den Einfluß der Gravitation auf den Gang von Uhren herleiten. Dabei soll gezeigt werden, daß eine Uhr auf einem Berg schneller geht als eine Uhr im Tal. Dieser (...) Effekt der Gravitation ist allerdings so klein, daß er nur mit Atomuhren nachgewiesen werden kann.
Um Mißverständnissen vorzubeugen, sei betont, daß es hier nicht darum geht, den Einfluß eines unterschiedlichen Gravitationsfeldes auf solche Uhren zu untersuchen, die ihr Zeitmaß mit Hilfe der Erdanziehung erzeugen. Sanduhren scheiden also ebenso aus, wie alle Uhren mit Schwerependel. Von den hier benutzten Uhren verlangen wir, daß ihr Gang unberührt davon bleibt, ob wir die Uhren auf den Kopfstellen, oder in irgendeine andere Lage bringen. Ebenso wie Atomuhren erfüllen Uhren mit elektrischen Schwingkreisen als Unruh diese Forderung. Wir betrachten zwei gleiche Uhren, die aus je einem hochfrequenten Schwingkreis bestehen. Um den synchronen Gang der beiden Uhren zu überprüfen, stellen wir die beiden Schwingkreise nebeneinander auf und vergleichen deren elektrische Schwingungen mit Hilfe eines Oszillographen. Sind die beiden Uhren voneinander entfernt aufgestellt, so ist ein Vergleich möglich, wenn die beiden hochfrequenten Schwingkreise Radiowellen ausstrahlen. Über eine Antenne können wir an jedem Ort die beiden Radiowellen empfangen und deren Frequenz f messen. Da die Frequenz der Radiowellen gleich der Frequenz der Schwingkreise ist, vergleichen wir auf diese Weise indirekt den Gang der beiden Uhren.
Für die folgenden Überlegungen benötigen wir das Teilchenbild der elektromagnetischen Strahlung. Photonen, wie man die Lichtteilchen nennt, haben die Energie E = h f, wobei f die Frequenz der Strahlung und h = 6,625 10-34 Js das Plancksche Wirkungsquantum ist. Die Masse eines Photons ergibt sich zu m = E / c² = h f / c².
(....) Wir betrachten nun die beiden von den Schwingkreisen ausgesandten Radiowellen im Photonenbild. Ein Schwingkreis ist im Tal und ein Schwingkreis auf dem Berg aufgestellt. Der Empfänger befindet sich auf halber Höhe zwischen den beiden Schwingkreisen (....). Photonen, die der Sender im Tal ausstrahlt, müssen den Berg 'hinaufsteigen'. Dazu benötigen sie Energie, die sie ihrer Photonenenergie E = h f entnehmen. Beim Empfänger angekommen, ist daher die Photonenenergie kleiner und mit E = h f auch die Frequenz der Radiowelle. Für die Photonen, die der Sender auf dem Berg aussendet, sind die Verhältnisse gerade umgekehrt. Demnach treffen beim Empfänger zwei Radiowellen mit verschiedener Frequenz ein. Die Frequenz der von oben kommenden Radiowelle ist größer als die Frequenz der vorn unten kommenden Welle.(....)

In der Höhe gehen Uhren schneller

Neben dem (...) Geschwindigkeitseffekt gibt es einen (...) Gravitationseffekt. Wir leiten den quantitativen Zusammenhang her: Befindet sich der Empfänger in der Höhe H', so ist die Photonenenergie der aufsteigenden Photonen um DE_Tal = m g H' kleiner, wenn sie beim Empfänger ankommen; m ist die Masse eines Photons und g = 9,8 m/s2 ist die Erdbeschleunigung. Die Energie E = m g H ist die potentielle Energie eines Körpers im Schwerefeld der Erde. Die Photonen, die von der Höhe H des Berges `hinabfallen', gewinnen den Energiebetrag

DE_Berg = m g (H - H').

Beim Empfänger angekommen, unterscheidet sich die Energie der beiden Photonen um
DE = DE_Berg + DE_Tal = m g (H - H') + m g H' = m g H
Für den Frequenzunterschied Df der beiden Wellen folgt aus der Gleichung
E = h f

DE = h Df.

Lösen wir diese Gleichung nach Df auf und setzen DE = m g H ein, so ergibt sich

Df = DE / h = m g H / h

Dieser Frequenzunterschied ist unabhängig von der Höhe H', in der sich der Empfänger befindet. Die Photonenmasse m ersetzen wir mit Hilfe der Gleichung
m = E / c² = h f / c² und erhalten

Df = (g H / c²) f.

Eine Uhr, deren Uhrwerk mit der Frequenz f geht, hat in der Höhe H eine um Df höhere Frequenz. Die Zeitanzeige t der Uhr ist proportional zu ihrer Frequenz f. Die Abweichung in der Zeitanzeige Df ist daher auch proportional zur Frequenzänderung Df. Es gilt
Df / f = Dt / t.
Damit folgt aus obiger Gleichung Dt_g = (g H / c²) t.

Nach der Zeit t geht eine Uhr in der Höhe H um die Zeit

Dt_g = (g H / c²) t

vor im Vergleich zu einer Uhr am Boden.

Der Gang einer Uhr hängt also nicht allein von ihrer Bewegung ab, sondern auch von dem Gravitationsfeld, in dem sie sich befindet. Im Weltall, fern von schweren Massen, gehen Uhren schneller, während sie in der unmittelbaren Nachb*****aft von Sternen langsamer gehen.
Im Gravitationsfeld der Erde ist dieser Effekt jedoch so klein, daß man ihn zunächst für nicht meßbar hielt. So geht zum Beispiel eine Uhr auf der Spitze des Montblanc (H = 4 180 m) in 50 Jahren im Vergleich zu einer Uhr auf Meereshöhe nur um Dt_g = 0,7 ms vor:


Dt_g = (g H / c²) t = 2,28 * 10^-11 a = 0,7 ms

Sie sehen, es besteht kein Anlaß für jemand, der in den Bergen wohnt, wegen dieser 'Lebensdehnung' in die Ebene zu ziehen.
Obwohl der Effekt so klein ist, gelang den Amerikanern Pound, Rebka und Snider in den Jahren 1960 bis 1965 der experimentelle Nachweis. Dies war möglich mit Hilfe des von Rudolf Mößbauer entdeckten und nach ihm benannten Effekts. Mößbauer hatte erkannt, daß man bei der Absorption von Gammastrahlung äußerst geringe Frequenzunterschiede nachweisen kann. Die von einem Atomkern emittierte Gammastrahlung wird von einem anderen Kern gleicher Art nur dann absorbiert, wenn die Frequenz der Strahlung nahezu unverändert ist. Pound, Rebka und Snider hatten am Boden eines etwa 25 m hohen Turms der Harvard-Universität eine y-Strahlungsquelle aufgestellt und oben im Turm einen Absorber angebracht. Wegen der Frequenzverschiebung, bedingt durch das Aufsteigen der Photonen, konnte die Strahlung nicht absorbiert werden. Bewegt man jedoch die Quelle ein wenig in Richtung des Absorbers, so gibt man den Photonen etwas Energie. Diese zusätzliche kinetische Energie erhöht die Frequenz und die Strahlung wird absorbiert. So konnte man eine relative Frequenzverschiebung von Df / f = 2,5 * 10^-15 nachweisen und die Formel für den Gravitationseffekt mit einer Meßgenauigkeit von 1 % bestätigen. Um eine Vorstellung von der Präzision dieser Messungen zu erhalten, sei bemerkt, daß die Frequenz von sichtbarem Licht (f =10^-15 Hz) bei einer Höhendifferenz von 20 m nur um 1 Hz verändert wird. Der Gravitationseffekt macht sich in der Umgebung von Neutronensternen und den sogenannten 'Schwarzen Löchern' besonders stark bemerkbar. Diesen Sternen ist man erst im letzten Jahrzehnt mit neuen Radioteleskopen und durch Satellitenbeobachtung auf die Spur gekommen. Neutronensterne und 'Schwarze Löcher' waren ursprünglich Riesensterne, die nach dem Ausbrennen ihrer Kernenergievorräte unter der Last ihrer Gravitationswirkung in sich zusammenstürzten. Dabei werden sogar die Atomkerne dicht aneinandergepackt. Dies führt zu einem unvorstellbar großen Gravitationsfeld, aus dem keinerlei Strahlung mehr entweichen kann. Daher die Bezeichnung 'Schwarzes Loch'. (...)

Prof. Dr. Roman Sexl war Vorstand am Institut für Theoretische Physik der Universität Wien und Abteilungsleiter am Institut für Weltraumforschung der Österreichischen Akademie der Wissenschaften.

Dr. Herbert Kurt Schmidt ist Oberstudienrat am Ludwig-Georgs-Gymnasium zu Darmstadt

MfG
Orca

Lambert
30.04.08, 22:03
sqt ergibt den (durch u.a. SRT stabilisierten) Übergang von Gravitation zu Elektromagnetismus. Es ist ein mengentheoretischer Zusammenhang.

Gruß,
Lambert

ach so

Gruß,
Lambert

criptically
30.04.08, 22:03
Wenn die Lichtgeschwindigkeit von der Photenmasse abhängig wäre, dann müßte in der Formel für die Lichtablenkung im Gravitationsfeld die Photenmasse erscheinen.

Wenn es vor Ort schnelle (leichte) und langsame (schwere) Photonen geben würde, dann könnte man das leicht feststellen, weil sie bei gleichem Lichtweg unterschiedliche Laufzeiten hätten.

...


MfG
Horst


Ich dachte, wenn man in der Formel n(r) = c / c' = 1 / [1 - 2Gm/(rc²)] nur Gm/(rc²) betrachtet, sieht man dass da etwas "fehlt" - die Energie des Photons E. Bringt man E hinein, bekommt man ein Ausdruck für potentielle Energie des Photons im Gravitationsfeld: GmE/(rc²)=(Gm/r)(E/c²)=Gmµ/r, wobei µ die Masse des Photons ist.

mfg

Lambert
30.04.08, 22:08
sqt ergibt den (durch u.a. SRT stabilisierten) Übergang von Gravitation zu Elektromagnetismus. Es ist ein mengentheoretischer Zusammenhang.

Gruß,
Lambert

ach so ist das

Ach so, deswegen sind die Formel der elktrische Feldkraft und der Gravitation so ähnlich. Ach so.

Gruß,
Lambert

ingeniosus
30.04.08, 22:09
Ja. Was in jedem Fall bleibt, ist die geometrische Auslenkung, eine regelrechte Raumkrümmung. Im Rahmen der ART rechnen Fachleute nur mit der Krümmung und kommen offenbar damit problemlos klar.



Salve Uranor!

Soweit ich die ART und SRT verstehe, wäre die Raumkrümmung eben die Berücksichtigung der Gravitation.

Also doch Gravitation?

Aber ich bin ehrlich, ich mache nur logische Schlüsse und habe wenig experimentelle Erfahrung. Die Paarbildung habe ich heute erstmals (!) gelesen, ich muss das noch geniessen und verarbeiten....

Salve, ingeniosus

criptically
30.04.08, 22:13
...
Bisher stimmen alle Berechnungen der Relativität, die gesamte Raumfahrt wäre nicht möglich gewesen, hätten wir keine SRT oder ART.
...


Sie werden immer lustiger! :D :D :D

mfg

criptically
30.04.08, 22:22
ach so ist das

Ach so, deswegen sind die Formel der elktrische Feldkraft und der Gravitation so ähnlich. Ach so.

Gruß,
Lambert

Ist Spam in Q-Forum erwünscht, oder? :D

mfg

criptically
30.04.08, 22:29
...

Soweit ich die ART und SRT verstehe, wäre die Raumkrümmung eben die Berücksichtigung der Gravitation.

...

Und wie wird die Trägheitskraft berücksichtigt?

Oder, wie krümmt man die Trägheitskraft? :D

mfg

orca
30.04.08, 22:30
Ich dachte, wenn man in der Formel n(r) = c / c' = 1 / [1 - 2Gm/(rc²)] nur Gm/(rc²) betrachtet, sieht man dass da etwas "fehlt" - die Energie des Photons E. Bringt man E hinein, bekommt man ein Ausdruck für potentielle Energie des Photons im Gravitationsfeld: GmE/(rc²)=(Gm/r)(E/c²)=Gmµ/r, wobei µ die Masse des Photons ist.

mfg

Wenn man die Einheiten für G * m / (r * c²) einsetzt, dann erhält man eine einheitenlose Zahl und kann den Brechungindex c/c' berechnen.

Wenn man die Einheiten für G * m * E / (r * c²) einsetzt, dann erhält man keine einheitenlose Zahl und kann den Brechungindex c/c' nicht berechnen.

Man sieht durch den Einheitenvergleich, daß da nix "fehlt", so einfach ist das!

MfG
Horst

ingeniosus
30.04.08, 22:33
Also ORCA, sie machen mich mutig, Dann würde ich dasselbe auch Hr. Prof SEXL sagen. Er müsste dann ja ein Vorgänger des berühmten Prof.Zeilinger sein....

Er hat recht mit der Gravitation, aber rechnet auch mit der Masse eines Photons und die ist meines Wissens ja echt physikalisch 0 - zumindest derzeit.

....aber es gilt auch Sokrates: ich weiss, dass ich nicht(s) weiss...

Lambert
30.04.08, 22:47
Die einfachste Lösung ist oft die richtige.

So einfach kann die Welt sein: Mengenlehre auf virtuelle Teilchen (ob Volumenteilchen (oder für wer will auch Energieteilchen)) hetzen. Kompliziert wird 's geradezu von alleine. Und GUT ist's.

Gruß,
Lambert

orca
30.04.08, 22:48
Er hat recht mit der Gravitation, aber rechnet auch mit der Masse eines Photons und die ist meines Wissens ja echt physikalisch 0 - zumindest derzeit.




Das tut er eben nicht(!):

Die Photonenmasse m ersetzen wir mit Hilfe der Gleichung
m = E / c² = h f / c² und erhalten

Df = (g H / c²) f.

In der Formel Df = f g H / c² kommt die Photonenmasse überhaupt nicht vor (!), oder habe ich Tomaten auf den Augen?

Kannst du nicht lesen, oder verstehst du nicht, was du da gerade liest?

MfG
Orca

criptically
30.04.08, 22:50
Wenn man die Einheiten für G * m / (r * c²) einsetzt, dann erhält man eine einheitenlose Zahl und kann den Brechungindex c/c' berechnen.

Wenn man die Einheiten für G * m * E / (r * c²) einsetzt, dann erhält man keine einheitenlose Zahl und kann den Brechungindex c/c' nicht berechnen.

Man sieht durch den Einheitenvergleich, daß da nix "fehlt", so einfach ist das!

MfG
Horst

Es muss selbstverständlich die ganze Formel umgeformt werden.

Aus n(r) = c / c' = 1 / [1 - 2Gm/(rc²)] erhält man

E-2GmE/(rc²)=c'E/c .

Das ist also eine "Energierelation", wobei unklar ist woher Faktor 2 kommt (normal wäre Faktor 1).

mfg

ingeniosus
30.04.08, 22:54
Und wie wird die Trägheitskraft berücksichtigt?

Oder, wie krümmt man die Trägheitskraft? :D

mfg

Trägheitskraft ist doch eine Masseeigenschaft, nur so auf die schnelle....
einfache klassische Mechanik!

Die gekrümmten Grössen sind : Raum(Länge. Breite, Höhe), Zeit und Masse bzw daraus folgende Gravitation!

Wo bereitet die Trägheit Schwierigkeiten? Kraft = Masse mal Beschleunigung

Die Krümmung gilt nur für Sichtweisen eines Beobachters....usw....Einstein lässt grüssen....

criptically
30.04.08, 22:57
Trägheitskraft ist doch eine Masseeigenschaft, nur so auf die schnelle....
einfache klassische Mechanik!

Die gekrümmten Grössen sind : Raum(Länge. Breite, Höhe), Zeit und Masse bzw daraus folgende Gravitation!

Wo bereitet die Trägheit Schwierigkeiten? Kraft = Masse mal Beschleunigung

Die Krümmung gilt nur für Sichtweisen eines Beobachters....usw....Einstein lässt grüssen....

Läuft eine Masse auf krummen Bahnen, gibt es so etwas wie Zentrifugalkraft!

Newton grüßt Einstein! :D

mfg

ingeniosus
30.04.08, 23:09
Es muss selbstverständlich die ganze Formel umgeformt werden.

Aus n(r) = c / c' = 1 / [1 - 2Gm/(rc²)] erhält man

E-2GmE/(rc²)=c'E/c .

Das ist also eine "Energierelation", wobei unklar ist woher Faktor 2 kommt (normal wäre Faktor 1).

mfg

Gleichungsrechnung ist Sache von Gymnasiasten, dort hat man das gelernt. Wissenschaftler sollten das können.

Auch ich erlaube mir höflichst und freundlich darauf hinzuweisen, dass criptically sehr "kryptisch" ist! ..... E kann man auch wieder wegkürzen !!!!
Cogito et ago - inte ago et inte cogito! Fasse, wer es fassen kann !

ingeniosus
30.04.08, 23:11
Newton grüßt Einstein! :D

mfg

Also hopp auf, auf den Weg von Newton zu Einstein! Es ist sehr spannend!

criptically
30.04.08, 23:16
Gleichungsrechnung ist Sache von Gymnasiasten, dort hat man das gelernt. Wissenschaftler sollten das können.

Auch ich erlaube mir höflichst und freundlich darauf hinzuweisen, dass criptically sehr "kryptisch" ist! ..... E kann man auch wieder wegkürzen !!!!
Cogito et ago - inte ago et inte cogito! Fasse, wer es fassen kann !

Klar, dass man das kann! Ich habe beide Seiten mit E multipliziert! :D

mfg

criptically
30.04.08, 23:20
Also hopp auf, auf den Weg von Newton zu Einstein! Es ist sehr spannend!

Wieso Newton zu Einstein? Einstein ist absolut überflüssig (überflüssig = "superfluid"). :D

mfg

orca
30.04.08, 23:36
Es muss selbstverständlich die ganze Formel umgeformt werden.

Aus n(r) = c / c' = 1 / [1 - 2Gm/(rc²)] erhält man

E-2GmE/(rc²)=c'E/c .

Das ist also eine "Energierelation", wobei unklar ist woher Faktor 2 kommt (normal wäre Faktor 1).

mfg

Ich will mal nicht beleidigend werden, aber mir reichts!:eek:

criptically
01.05.08, 00:00
Ich will mal nicht beleidigend werden, aber mir reichts!:eek:

Entschuldigung aber ich versuche die Formel zu "analysieren". Betrachten wir noch einmal die Formel n(r) = c / c' = 1 / [1 - 2Gm/(rc²)]. Wenn 2Gm/(rc²) in der obigen Formel größer als 1 wird (sehr große Masse oder sehr kleines r), wird c/c' bzw. n negativ. Wie ist physikalische Deutung dazu?

mfg

Uranor
01.05.08, 01:58
salve ingeniosus!
Soweit ich die ART und SRT verstehe, wäre die Raumkrümmung eben die Berücksichtigung der Gravitation.

Also doch Gravitation?
Oh ja ist es die Gravitation. Die Auswirkung ist, als würden Murmeln auf einem Spiralweg in ein Murmelloch rollen. Jedes Fermion krümmt den Raum problewmlos allein. Intensiver klappt das mit größeren Ansammlungen. Die Effekte ergeben sich, weil sie nicht linear aufeinander zurollen. Sie umkreisen masse-/trägheitsbedingt den gemeinsamen Schwerpunkt.

Gravitation und Trägheit sind in der Kraftwirkung äquavilent. Dabei steuert Masse die aktuell gemessene Masse an, die Trägheit, auch Beharrung genannt, strebt die Fortsetzung der erreichten Dynamiksituation an. Beides ergänzt sich zum gegenseitigen Umkreisen des gemeinsamen Schwerpunktes.


Das Licht dürfte von der Gravitationssituation gar nix mitbekommen. Es wird ja davon ausgegangen, dass die Gravitationskraft auf Bosonen beruht. Und die können nun mal beliebig überlagern.

Andererseits stellen Bosonen die Signalverbindung zwischen Massen dar. Die Signalvermittlung geschieht via Impulstausch. Die Massen reagieren tatsächlich kinetisch, setzen die Schwingungsanregung in Bewegungsdynamik und in Infrarot um. Sofern Licht von allen Seiten eintrifft, werden sie nur Infrarot, keine lineare Dynamik umsetzen können. Aber der Part ist jetzt nicht so interessant. Eins nur, das Postulat des Gravitationsdruckes würde das Andrücken des Lichtes an die Raumkrümmung gerade erwarten lassen. Den extrem viele Bosonen werden doch hinreichend viele auffindbar machen, die in den Eigenschaften den Signal-Bosonen sehr nahe kommen. Geringe gegenseitige Beeinflussung wäre die Folge, das aber in etwa stetig. Nach der QM werden dazu genügend Situationen untersucht. Quantenphysiker sollten also die Gravitation bestmöglich verstehen können. Nur hat ausgerechnet die Gravitation in der Quantenphysik nahezu keine Bedeutung.


Die Gravitonen sollen eine andere Art Bosonen sein. Von den Fermionen werden sie nicht ausgesandt. Masseangeregter Zustand wird nicht beobachtet. Auch im starken Gravitationsfeld ändern die Fermionen ihre Masse (das meint per Konvention immer die Ruhmasse) nicht. Sie erhöhen die äquavilente Masse, die Trägkeit.

Das ist nun ein eigentümliches Verhalten. Und doch ist es bekannte Routine. Angenommen, ein Objekt beschleunigt im noch schwach angedeuteten Kreis gegen die Erde. Die Beschleunigung vermittelt zunehmende Trägheit. Also neigt das Objekt grundsätzlich dazu, eine Kreisbahn anzusteuern. Allerdings ist es schon zu sehr genähert bzw. für stabilisierende Trägheit extrem zu langsam.

Kurz, die Trägheit würde den Winkel
Objekt->Ziel, Objekt->gemeinsamer Schwerpunkt
verdoppeln. Folgerichtig würde man doch Gravitonen vom Schwerpunktspartner ausgehend und "Trägionen" :p vom Doppelwinkel her erwarten müssen.

Zumindest ich oh Laie vermag keine Organisation zu beschreiben, die passend platziert wäre, genau das richtige tun zu können. Ich müsste wie wilkürlich postulieren. Welcher Physiker kann nun die Zusammenhänge zumindest nach Vernunft beschreiben?


Nur wenn ich mich zumindest in etwa an Rueda und Haisch orientiere, Masse aus dem Nichts (http://www.wissenschaft.de/wissen/news/256525.html), will sich alles sauber zusamen fügen. Das Überprüfungsangebot ist, dass gerade dort, wo die wenigsten Massen angetroffen werden, die dichtesten Fluktuationenmengen entstehen. Ordnende Feldwirkung aus der Raumkrümmung, das muss nicht unlogisch sein. Man kann auf der Schiene auch weiter denken und gelangt zu verblüffend klaren Fragestellungen an die Naturbeobachtung. Also Fragen und Antworten harmonieren miteinander.

Ergo, in Analogie zum Casimir-Druck würden die Massen gerade dort hinstreben, wo die wenigsten Fluktuationen entstehen. Genau im gemeinsamen Schwerpunkt zwischen Erde und Mond, Erde und Sonne oder beliebige andere würde der maximale Unterdruck erwartet werden müssen. Die Druckperspektive zeigt sich somit scharf vektororientiert, und das präzise auf die Beobachtungen bezogen.

Jo. Wer kann das mit allen sich ergebenden Folgen durchkalkulieren? Ich kann es nur streng geordnet vom Schema her anbieten. Überprüfen müssen es Kenner. ART, Mathe, QM, QFT wären angesagt. Die Meister sollen beweisen, dass die Gravitation anzieht! Dabei sollen sie die hier geforderte Fluktuationen-Situation scharf ausschließen. Man wird vorbehaltlos sein und eine hohe Wahrscheinlichkeit für die Antwort finden.

- Zum Glück hat es nix zu sagen, dass ich über genügend solide Zweifel doch immer unerschrockener genau das beschriebene unbefangen fordern muss. Es will sich keine auch nur stümperhafte Alternative zeigen. Und genau darum erlaube ich mir, immer sicherer zu werden, wenngleich ich gar nis aktiv zu tun in der Lage wäre.

Ole. Wer die Annahme einreißen will, dann bitte mit Schmackes. Auch die Erkenntnis, dass man offensichtlich weniger weiß als 1/2 Ameise, soll Laune machen können. Dann würde ja die wirklich starke Erkenntnis noch ausstehen. Physik eben. Sie wird immer spannend bleiben.

Gruß Uranor

orca
01.05.08, 07:41
Entschuldigung aber ich versuche die Formel zu "analysieren".
mfg

Das hört sich schon besser an.

Betrachten wir noch einmal die Formel n(r) = c / c' = 1 / [1 - 2Gm/(rc²)]. Wenn 2Gm/(rc²) in der obigen Formel größer als 1 wird (sehr große Masse oder sehr kleines r), wird c/c' bzw. n negativ. Wie ist physikalische Deutung dazu?

mfg

Normalerweise nimmt die Vakuum-Lichtgeschwindigkeit mit der Entfernung vom Gravitationszentrum zu. In unendlich großer Entfernung von allen Massenzentren würde die Lichtgeschwindigkeit unendlich. Im Universum entfernt sich das Licht aber von einem Massenzentrum, um sich zugleich anderen Massenzentren zu nähern, darum ist die Lichtausbreitung im Universum nicht instantan, sondern zwischen den Gravitationsmassen annähernd gleich. Sie nimmt auf der Erde in Richtung Sonne mit der Entfernung zunächst zu, gerät dann aber in den Einfluß der Sonne und nimmt mit Annäherung an die Sonne wieder ab.
Entfernt sich das Licht in Gegenrichtung zugleich von der Erde und von der Sonne, dann addieren sich die Effekte. Diese Effekte sind außerordentlich klein, der Brechungsindex an der Sonne beträgt z.B. n(r) = 1,000.004.2

Wird der Ausdruck 2Gm/(rc²) kleiner 1, dann wird auch die Brechzahl n(r) = c/c' kleiner 1. Dann würde die Lichtgeschwindigkeit mit dem Abstand vom Gravitationzentrum nicht gößer sondern kleiner. Ich glaube nicht, daß dieser Fall entreten kann, weil in starken Gravitationsfeldern zugleich die Lichtgeschwindigkeit c² kleiner wird.

Denn für den Brechungsindex in Abhängigkeit vom Gravitationspotential gilt in der klassischen Mechanik

n(Psi) = c / c' = 1 / (1 + 2 Psi /c²). [F. Spieweck a.a.O.]

Das heißt, die Lichtgeschwindigkeit ist von der Gravitationsbeschleunigung g abhängig, weil Psi = g H. Auf einem leichten Körper ist die Vakuum-Lichtgeschwindigkeit groß, auf einem schweren Körper ist die Vakuum-Lichtgeschwindigkeit klein.
Die Lichtgeschwindigkeit ist im Inertialsysteme der Erde, im Vakuum und auf Meereshöhe exakt 299.792.458 m/s schnell.
In allen Inertialsystem breitet sich das Licht wie in einem ruhenden "Äther" aus, das Ausbreitungsmedium ist das Gravitationsfeld.
Im Nichtinertialsystemen ist sie außerdem von der Bewegung gegenüber dem Inertialsystem abhängig.

Das ist ziemlich kompliziert, aber die Raumzeitkrümmungstheorie des irrationalen Postulators ist auch nicht ganz einfach.

MfG
Orca

Lambert
01.05.08, 09:15
Mir kommt's so vor, als dass Ihr allen zu kompliziert denkt.

Wie Wheeler schon sagte:
Die Lösung ist vermutlich unerwartet einfach.

Sich um Einstein zu kloppen, ist verschwendete Energie. :)

Gruß,
Lambert

PS. wer in die Nacht schaut, sieht Sterne aber nur wenig Licht.

pauli
01.05.08, 10:56
Orca und criptically in einem Thread vereint, das ist wie Pat + Patachon, wie Lolek + Bolek.
Im Universum entfernt sich das Licht aber von einem Massenzentrum, um sich zugleich anderen Massenzentren zu nähern, darum ist die Lichtausbreitung im Universum nicht instantan, sondern zwischen den Gravitationsmassen annähernd gleich. Sie nimmt auf der Erde in Richtung Sonne mit der Entfernung zunächst zu, gerät dann aber in den Einfluß der Sonne und nimmt mit Annäherung an die Sonne wieder ab.
Das erklärt die Bremsspuren des Lichtes überall im Universum, schließlich muss es ja vor jeder Sonne eine quasi-Vollbremsung ausführen.

orca
01.05.08, 12:49
Moin Horst,

Eine der besten falschen Antworten!

In der 'klassischen Mechanik" (ohne RT) gibt es keine Konstanz der Lichtgeschwindigkeit, da diese in diesem Gebiet unendlich ist. So, wie jede andere Signalausbreitungsgeschwindigkeit. Deshalb hieß Einsteins ursprüngliche Lichtgeschwindigkeit auch V=∞.

Und wie, überhaupt, gelingt es einem Inertialsystem inertial zu sein, wenn es von der Gravitation abhängt?

Bei deiner Antwort bedarf es weder Inertialsystemen noch einer Konstanz von Irgendetwas, sondern es gilt Absolutes.

Das Trägheitgesetz lautet, jeder Körper beharrt im Zustand der Ruhe oder der geradlinig gleichförmigen Bewegung, solange keine äußeren Kräfte auf ihn einwirken.
Bezugssystem, die ruhen oder die sich geradlinig gleichförmig bewegen, weil sie frei von äußeren Kräften sind, nennt man Inertialsysteme.

Wenn wir doch nur wüßten, was innere und äußere Kräfte sind!

Innere Kräfte, also z.B. die eigenen Gravitationskräfte, (sowie mechanische Zug-, Druck- und Scherspannungen) können den Bewegungszustand eines Bezugssystems nicht ändern.

Auf ein Inertialsystem können deshalb beliebige innere Kräfte wirken, weil nur äußere Kräfte Beschleunigungen bewirken.

Das Dynamische Grundgesetz bringt das zum Ausdruck: F = ma.

Hierbei ist F die Summe oder Resultierende der äußeren Kräfte. Ist die Summe der äußeren Kräfte Null, dann ist die Beschleunigung auch Null, also ruht der Körper oder er bewegt sich geradlinig gleichförmig.

Aus dem galileischen Relativitätsprinzip folgt, daß man auch mit Hilfe des Lichts nicht zwischen Ruhe und geradlinig gleichförmiger Bewegung unterscheiden kann. Deshalb muß die Lichtgeschwindigkeit in Inertialsystemen konstant (örtlich in alle Richtungen gleich) sein.

Wirken in einem Inertialsystem überhaupt keine Kräfte, dann ist die Lichtgeschwindigkeit unendlich.
Wirken in einem Inertialsystem äußere Kräfte, dann ist es kein Inertialsystem mehr, dann ist die Lichtgeschwindigkeit von der Bewegung abhängig.
Wirken in einem Inertialsystem innere Kräfte, z.B, in Form der eigenen Gravitationskräfte, dann ist die Lichtgeschwindigkeit von der Masse und vom Abstand vom Gravitationszentrum abhängig.

n(Psi) = 1 / (1 + 2 Psi /c²)

n(r) = 1 / [1 - 2 G m /(r c²)]

Unter extremen Bedingungen ist die Lichtgeschwindigkeit sogar Null (Schwarzschildradius).

Das ist alles seit Jahrhunderten bereits bekannt und wird nur immer wieder von der Popphysikern falsch dargestellt.
Der "Äther" widerspricht dem galileischen Relativitätsprinzip, der Grundlage der klassischen Mechanik.


MfG
Orca

rafiti
01.05.08, 19:40
rafiti,
----
Was soll an deiner Substanz empirisch dran sein?
----
Gar nichts, sie ist mein Ursprungspostulat. Der Gläubige beruft sich auf Gott, worauf Physiker sich berufen, habe ich bis heute noch nicht herausbekommen, anscheinend gibt es da noch keine einheitliche Meinung, weil wohl Physiker generell keine Warum- und Woher-Fragen stellen.


Wie gesagt, ich bin kein Physiker, und auf der einen Seite finde ich es schade, dass du dir die Mühe machst, um Modelle zu erstellen, die sicher Zeit fressen, nur um bspw. den Urknall ad acta zu legen. Das sind schwierige Fragen für einen Melonenzüchter. ;)
Was hindert dich daran nur das Not-From-This-World zu betrachten, das beim Urknall keine solche Substanz im Universum zuließ?

gruss
rafiti

criptically
01.05.08, 21:09
...

Das erklärt die Bremsspuren des Lichtes überall im Universum, schließlich muss es ja vor jeder Sonne eine quasi-Vollbremsung ausführen.

Das stimmt auch teilweise! Beim Eintritt in ein IS wird das Licht auf LG relativ zum Gravitationsfeld abgebremst. Aus Impulserhaltungssatz ergibt sich dann die "relativistische Massenformel".

Impuls des Photons in Sternsystem: p=mc;
Impuls des Photons in IS der Erde (die Erde bewegt sich senkrecht zur Ausbreitungsrichtung des Lichts): p'=m'c';

Wegen Impulserhaltung gilt: p=p' bzw. mc=m'c';

Die LG im IS der Erde bezogen auf IS des Sterns ist nach Pythagoras: c'=√(c²-v²);

Daraus erhält man die relativistische Masse:

m'c'=mc ;
m'√(c²-v²)=mc;

==>

m'=m/√(1-v²/c²)

Newton grüßt Einstein! :D :D :D

mfg

criptically
01.05.08, 21:31
...In allen Inertialsystem breitet sich das Licht wie in einem ruhenden "Äther" aus, das Ausbreitungsmedium ist das Gravitationsfeld...

MfG
Orca

Daraus kann geschlossen werden, dass der "Äther" vom Gravitationsfeld "großer" Massen mitgeführt wird! Laut Gravitationsgesetz werden alle Massen gleich stark beschleunigt, unabhängig davon wie groß die beschleunigte Masse ist (von m=0 (für Äther) bis m=∞): Beschleunigung a=GM/r². Oder haben Sie eine andere Meinung dazu?

mfg

orca
01.05.08, 22:18
Laut Gravitationsgesetz werden alle Massen gleich stark beschleunigt, unabhängig davon wie groß die beschleunigte Masse ist (von m=0 (für Äther) bis m=∞): Beschleunigung a=GM/r². Oder haben Sie eine andere Meinung dazu?

mfg

Ich habe dabei ein kleines Problem:Ich kann mir ja eine klitzekleine Masse vorstellen, ich kann mir aber keine Masse mit Null kg vorstellen, eine Masse muß eine Masse haben, eine Nullmasse ist keine Masse.

Die Gravitation wirkt nicht einfach direkt anziehend (oder auch zurückhaltend) auf das Licht, sondern indirekt auf die Wellenlänge und die Frequenz und somit doppelt auch auf die Lichtgeschwindigkeit

c' = f' * lampda'

(wie F. Spieweck s.o. beschrieben hat).

MfG
Orca

uwebus
02.05.08, 16:04
rafiti,
----
....nur um bspw. den Urknall ad acta zu legen.......... Was hindert dich daran nur das Not-From-This-World zu betrachten, das beim Urknall keine solche Substanz im Universum zuließ?
----
Was willst Du denn machen, wenn Du Dich selbst zu verstehen suchst? Entweder Du kaufst Dir ´ne Bibel, ´nen Koran oder ´ne anderweitige Schöpfungsgeschichte oder Du nimmst Dich als Bestandteil des Universums zur Kenntnis.

Ohne Schöpfer kein Anfang und kein Ende, also muß das Zeug, aus dem das Universum sich konstituiert, als ewig betrachtet werden. Egal wie Du nun dieses Zeug nennst, es ist verantwortlich für Massen, Vakuum, Dynamik sowie Leben einschließlich Bewußtsein (letzteres bezeichnen Philosophen auch gern als Geist). Und nun kannst Du anfangen zu modellieren, um aus dem Zeug etwas zu basteln, was mit den Beobachtungen der Naturwissenschaften vereinbar ist, mehr habe ich nicht getan mit meiner Arche.

Ich gehe nun davon aus, daß die Evidenz dabei beachtet werden muß, sonst reise ich ins Fantasialand. Physiker lassen die Evidenz außen vor und erfinden Dinge, die nie beobachtet wurden und auch nie beobachtet werden, z.B. hüllenlose endliche Volumina, volumenlose kräfteerzeugende Strings und verlustfreie Energieübertragung durchs Vakuum. Genauso gut kann ich an Teufel und Engel glauben, hier stehen sich Religion und Physik in nichts nach.

Was meinen Kritikern stinkt ist die Tatsache, daß ich mit einer einfachen quantitativen und qualitativen Verknüpfung von Vakuum und Masse ein in sich einfaches, nachvollziehbares Grundmodell geschaffen habe, mit dem sich Fragen beantworten lassen, auf die bis zum heutigen Tage die Physik noch keine Antworten hat. Weder die Entstehung der Gravitation noch die Entstehung der Dynamik sind bisher von Seiten der Physik erklärt, das Vakuum hängt bezüglich seiner Beschaffenheit noch “in der Luft“, und die Entstehung von sog. Teilchen ist ebenfalls noch unbeantwortet.

Wenn hier das Quantenvakuum der Physik angeführt wird, um mir zu sagen, mein Modell sei damit hinfällig, dann ist dazu zu sagen, daß mein Modell die Gravitation erklärt, das Modell der Quantengravitation hierzu aber noch aussteht.

Aus dem Internet:
“Die Physik ist bisher bis an das sogenannte "Quantenvakuum" herangekommen, das ein Teil des Hyperraums ist, und eine Welt mit 6 Dimensionen darstellt, wo sich Strings (das sind kleine, in 6 Dimensionen schwingende Fäden, welche eine Revolution im physikalischen Denken darstellen) aufhalten.“

Mein Kommentar:
Der Vatikan ist bisher an das sog. Himmelreich herangekommen, in dem sich körperlose Seelen in einer Hydromassagewelt die Sünden vom Astralleib waschen und zu frei schwingenden SEX-dimensionalen Engeln werden. Dies ist ein Revolution im religiösen Denken, welches uns die Hölle erspart und unsere Vorfreude aufs Jenseits anstachelt.

Siehst Du da irgendeinen Unterschied? :confused:
Lies mal, was da so alles unter Quantenvakuum im Internet zu finden ist, ganz tolles Zeuchs muß das sein. Da kann ich natürlich mit meinem 3-dimensionalen Vordenker Aristoteles und dessen simpler Substanz nicht mithalten.

Gruß

criptically
02.05.08, 20:02
Ich habe dabei ein kleines Problem:Ich kann mir ja eine klitzekleine Masse vorstellen, ich kann mir aber keine Masse mit Null kg vorstellen, eine Masse muß eine Masse haben, eine Nullmasse ist keine Masse.

...

MfG
Orca

Mein Gedanke war, dass bei "Paarvernichtung" die beiden Teilchen nicht ganz verschwinden, sondern "masselos" weiterleben (als Bauteile des "Äthers" oder "Vakuums" sozusagen). Da diese Paare (e- e+) überall "entstehen" können, dachte ich weiter, dass sie überall schon "existent" aber "unsichtbar" sind.

Die folgende Graphik zeigt Feynman-Diagramm von Bildung und Zerfall eines Photons!

A Feynman diagram of a positron and an electron annihilating into a photon which then decays back into a positron and an electron.

http://upload.wikimedia.org/wikipedia/commons/thumb/a/a4/Electron-positron-annihilation.svg/200px-Electron-positron-annihilation.svg.png

Annihilation (http://en.wikipedia.org/wiki/Annihilation)

mfg

Uli
02.05.08, 22:35
Mein Gedanke war, dass bei "Paarvernichtung" die beiden Teilchen nicht ganz verschwinden, sondern "masselos" weiterleben (als Bauteile des "Äthers" oder "Vakuums" sozusagen). Da diese Paare (e- e+) überall "entstehen" können, dachte ich weiter, dass sie überall schon "existent" aber "unsichtbar" sind.

Die folgende Graphik zeigt Feynman-Diagramm von Bildung und Zerfall eines Photons!



http://upload.wikimedia.org/wikipedia/commons/thumb/a/a4/Electron-positron-annihilation.svg/200px-Electron-positron-annihilation.svg.png

Annihilation (http://en.wikipedia.org/wiki/Annihilation)

mfg


Feynman-Diagramme basieren auf der quantisierten Form der Speziellen Relativität, die auch QED genannt wird: alles esoterischer Quatsch von einem Geschichtenerzähler aus Schilda und seinen hirnlosen Jüngern, die sich die QED zusammenphantasiert haben. Vergiss es.

Gruss, Uli

pauli
02.05.08, 22:58
Feynman-Diagramme basieren auf der quantisierten Form der Speziellen Relativität, die auch QED genannt wird: alles esoterischer Quatsch von einem Geschichtenerzähler aus Schilda und seinen hirnlosen Jüngern, die sich die QED zusammenphantasiert haben. Vergiss es.
rofl

Danke für diesen "Post des Tages"!

criptically
03.05.08, 11:10
Feynman-Diagramme basieren auf der quantisierten Form der Speziellen Relativität, die auch QED genannt wird: alles esoterischer Quatsch von einem Geschichtenerzähler aus Schilda und seinen hirnlosen Jüngern, die sich die QED zusammenphantasiert haben. Vergiss es.

Gruss, Uli

Und die quantisierte Form der Speziellen Relativität ist nichts anderes als die gefälschte Äthertheorie des Maxwells (luminferous aether etc.)!

mfg

Uranor
03.05.08, 12:11
Und die quantisierte Form der Speziellen Relativität ist nichts anderes als die gefälschte Äthertheorie des Maxwells (luminferous aether etc.)!

mfg
"Ist ja nur 2D.
erwischt..." (frei genutzt nach Clever&Smart :p )

Uli
03.05.08, 13:37
Und die quantisierte Form der Speziellen Relativität ist nichts anderes als die gefälschte Äthertheorie des Maxwells (luminferous aether etc.)!

mfg

... und im Himmel ist Jahrmarkt ...

ingeniosus
10.05.08, 14:39
... und im Himmel ist Jahrmarkt ...

Schöne Pfingsten, Uli !

ingeniosus
10.05.08, 14:53
Mein Gedanke war, dass bei "Paarvernichtung" die beiden Teilchen nicht ganz verschwinden, sondern "masselos" weiterleben (als Bauteile des "Äthers" oder "Vakuums" sozusagen). Da diese Paare (e- e+) überall "entstehen" können, dachte ich weiter, dass sie überall schon "existent" aber "unsichtbar" sind.



Also Feynmann mag vielleicht schon wieder nicht ganz aktuell zu sein, aber der Grundgedanke ist doch m.E. wertvoll.

Das Wort "Äther" würde ich aber aus historischen Gründen tunlichst vermeiden, dieser wurde schon vor einem Jahrhundert als nicht existent bewiesen.

Vakuumsenergie und Virtuelle Teilchen, Casimireffekt weisen aber schon auf eine existierende Energieform hin, die offensichtlich nicht sichtbar ist.

Wie können wir zB. von EM-Wellen sprechen, wenn kein EM-Feld existiert? Was schwingt denn in den Photonen? Wie kann eine Beugung entstehen. Ein Photon ist kein fester Körper, es muss eine Menge mehrerer schwingender Unterteilchen sein (neues Energon?).

Die Materie ist noch nicht erforscht!

ingeniosus
10.05.08, 14:56
Und die quantisierte Form der Speziellen Relativität ist nichts anderes als die gefälschte Äthertheorie des Maxwells (luminferous aether etc.)!



Versündige Dich nicht an Maxwell, zu seiner Zeit war der Äther ein gegebenes Faktum!

Eyk van Bommel
10.05.08, 22:14
Und die quantisierte Form der Speziellen Relativität ist nichts anderes als die gefälschte Äthertheorie des Maxwells (luminferous aether etc.)!
Komisch den Gedanken hatte ich auch, als ich auf www.einstein-online.info etwas darüber gelesen hatte (leider finde ich es nicht mehr, aber es ging um Raumquanten und dem „Jetzt“). Als ich das las, dachte ich mir auch das hört sich nun schon fast an wie das gute alte Äthermodell und man muss nicht mehr lange warten und sie behaupten die Raumzeitquanten befinden sich in absoluter ruhe.

Absolut ruhende Raumzeitquanten in Planckgröße? Back to the future:p

Gruß
EVB

ingeniosus
13.05.08, 14:50
... und man muss nicht mehr lange warten und sie behaupten die Raumzeitquanten befinden sich in absoluter ruhe.

Absolut ruhende Raumzeitquanten in Planckgröße? Back to the future:p

Gruß
EVB

Also das mit der absoluten Ruhe ist wohl nicht ganz vorstellbar.
Eine relative Ruhe kann ich mir vorstellen...aber was soll "absolute Ruhe" bedeuten, das wäre ja wieder gegen die Relativitätstheorie?

Die Raumzeit bringt doch nur Relativität, erst die Gravitation bringt auch die Krümmung......aber nur in unserer bildlichen Vorstellung.

orca
13.05.08, 16:33
Das Wort "Äther" würde ich aber aus historischen Gründen tunlichst vermeiden, dieser wurde schon vor einem Jahrhundert als nicht existent bewiesen.


Diesen sogenannten Beweis der "Nichtexistenz eines Äthers" würde ich gerne kennen lernen! :cool:

MfG
Orca

criptically
13.05.08, 20:43
...
Was schwingt denn in den Photonen? Wie kann eine Beugung entstehen. Ein Photon ist kein fester Körper, es muss eine Menge mehrerer schwingender Unterteilchen sein (neues Energon?).

Die Materie ist noch nicht erforscht!

Jeder weiß (vorausgesetzt er will es wissen), dass das Photon ein harmonischer Oszillator ist (bestehend aus einem Positron und einem Elektron, also ein "virtuelles" :D Elektron-Positron-Paar).

mfg

Lambert
13.05.08, 21:08
Jeder weiß (vorausgesetzt er will es wissen), dass das Photon ein harmonischer Oszillator ist (bestehend aus einem Positron und einem Elektron, also ein "virtuelles" :D Elektron-Positron-Paar).

mfg

Ich plädiere für zwei Photonen. :cool:

L

criptically
13.05.08, 21:24
Ich plädiere für zwei Photonen. :cool:

L

Was passiert mit Spinerhaltung 1/2+1/2=1? Bei zwei Photonen Spin = 2 oder 0. :cool: :D

mfg

Lambert
13.05.08, 22:11
Was passiert mit Spinerhaltung 1/2+1/2=1? Bei zwei Photonen Spin = 2 oder 0. :cool: :D

mfg

was passiert bei der Annilierung elektron + positron = 2 photonen ? :cool: :cool:

mfg L

Uranor
13.05.08, 22:22
Hach ist der Rasen so schön doppelt. :p :D :p

Eyk van Bommel
13.05.08, 22:26
Hallo ingeniosus,
Ich finde ja nur, dass die Beschreibung der Raumzeit sich immer mehr dem Äthermodell nähert. Und wenn man sich dann noch das Verwirbeln der Raumzeit durch sich drehende Massen vorstellt kommt man doch ganz schnell zum Modell des Mitgeführten Äthers? Wobei mir - dass will ich gleich sagen -nicht klar ist, ob die Stärke des Ätherwirbels (Mitführung) mit dem Raumzeitwirbel vergleich bar ist.
Eine relative Ruhe kann ich mir vorstellen...aber was soll "absolute Ruhe" bedeuten, das wäre ja wieder gegen die Relativitätstheorie?
War ja nicht so ernst gemeint, aber man kann sich schon fragen wie sich die Relativgeschwindigkeit der Raumzeitquanten zu den Objekten verhält? Ich meine, unterliegen die Raumzeitquanten selbst der Relativität?
Aber da ich kein Äther-liebhaber oder Raumzeitquanten-liebhaber bin, kann mir das relativ egal sein?

Gruß
EVB

ingeniosus
14.05.08, 13:35
Diesen sogenannten Beweis der "Nichtexistenz eines Äthers" würde ich gerne kennen lernen! :cool:

MfG
Orca

Ich empfehle Physik-Literatur um die Jahrhundertwende1900+: Lorentz, Einstein, Mach, usw...

ingeniosus
14.05.08, 13:50
War ja nicht so ernst gemeint, aber man kann sich schon fragen wie sich die Relativgeschwindigkeit der Raumzeitquanten zu den Objekten verhält? Ich meine, unterliegen die Raumzeitquanten selbst der Relativität?
Aber da ich kein Äther-liebhaber oder Raumzeitquanten-liebhaber bin, kann mir das relativ egal sein?

Gruß
EVB

Hallo Eyk van Bommel!

Also "Raumzeitquanten" sind auch für mich mal wieder so zerstückelte Häppchen von QuantenTheoretikern!

Ich kann mir Objekte, Energoi (= von mir verwendeter Begriff) oder Teilchen vorstellen. Das hat einen natürlichen Bezug (Fermionen,Bosonen).

Aber wo gibt es Raumzeitquanten. Einstein hat mit Quantum eben eine Portion oder bestimmte vorhandene Menge einer bestimmten Materie oder Energie (Photon) gemeint ....oder auch Plancksches Wirkungsquantum... das geht noch.

Also rein mathematisch kann es das mit definierten Bildungsgesetzen geben, aber in der Natur ist mir das noch nicht bekannt.

Raum, Zeit und Gravitation sind berechenbare Relativgrössen aber wozu RaumZeitQuanten?

ingeniosus
14.05.08, 14:04
Jeder weiß (vorausgesetzt er will es wissen), dass das Photon ein harmonischer Oszillator ist (bestehend aus einem Positron und einem Elektron, also ein "virtuelles" :D Elektron-Positron-Paar).

mfg

Der Vergleich "Harmonischer Oszillator" aus der Elektronik ist schon etwas nützlicher. Die Spin-Bewegungen sollten dann also doch irgendwie erhalten bleiben...

Die Massen (was immer das ist) neutralisieren sich ja.

Übrig bleiben würden kreisende (=oszillierende) Energiekreisel von EM-Energie...schon interessant!

Wo bleibt aber die gerichtetet Welle, wo der Impuls?

Lambert
14.05.08, 15:08
Raum, Zeit und Gravitation sind berechenbare Relativgrössen aber wozu RaumZeitQuanten?

ganz einfach: um u.a. den Begriff aktuale Unendlichkeit in der Physik zu
ma-thematisieren! Nur daraus entsteht die Relativität.

Lese hierzu SQT und verstehe die Zusammenhänge und die notwendigen Schulssfolgerungen. Es ändert Dein Weltbild.

Vielen Dank,
Lambert

criptically
14.05.08, 17:49
was passiert bei der Annilierung elektron + positron = 2 photonen ? :cool: :cool:

mfg L

Nochmal Feymnman-Diagramm dazu.

http://upload.wikimedia.org/wikipedia/commons/thumb/a/a4/Electron-positron-annihilation.svg/200px-Electron-positron-annihilation.svg.png

Und die Beschreibung lautet:
A Feynman diagram of a positron and an electron annihilating into a photon which then decays back into a positron and an electron.

mfg

Lambert
14.05.08, 18:03
Nochmal Feymnman-Diagramm dazu.

Und die Beschreibung lautet:

mfg

süß

hier noch eins:

http://www.quantenwelt.de/elementar/positronen.html
mit Text:
Positronen sind die Antiteilchen der Elektronen. Sie sind die ersten Antiteilchen, die entdeckt wurden und können durch Paarbildung erzeugt werden, indem man energiereiche Strahlung auf schwere Atome richtet.

Positronen zerstrahlen, wenn sie auf Elektronen treffen, unter Bildung zweier Photonen die in entgegengesetzter Richtung auseinanderfliegen und jeweils die Energie haben, die der Masse eines Elektrons entspricht.

na schau... für mich macht diese Aussage theoretischen Sinn.

L

criptically
14.05.08, 18:08
süß

hier noch eins:

http://www.quantenwelt.de/elementar/positronen.html
mit Text:
Positronen sind die Antiteilchen der Elektronen. Sie sind die ersten Antiteilchen, die entdeckt wurden und können durch Paarbildung erzeugt werden, indem man energiereiche Strahlung auf schwere Atome richtet.

Positronen zerstrahlen, wenn sie auf Elektronen treffen, unter Bildung zweier Photonen die in entgegengesetzter Richtung auseinanderfliegen und jeweils die Energie haben, die der Masse eines Elektrons entspricht.

na schau... für mich macht diese Aussage theoretischen Sinn.

L

Danke, aber von Joachim's Physik halte ich nicht besonders viel! :D

mfg

Lambert
14.05.08, 18:29
Danke, aber von Joachim's Physik halte ich nicht besonders viel! :D

mfg

dennoch kommt mir Joachim's Aussage plausibler vor als Dein Feynman Diagramm für diesen Ablauf. :cool:

criptically
14.05.08, 19:42
Der Vergleich "Harmonischer Oszillator" aus der Elektronik ist schon etwas nützlicher. Die Spin-Bewegungen sollten dann also doch irgendwie erhalten bleiben...

Die Massen (was immer das ist) neutralisieren sich ja.

Übrig bleiben würden kreisende (=oszillierende) Energiekreisel von EM-Energie...schon interessant!

Wo bleibt aber die gerichtetet Welle, wo der Impuls?

Du musst das komplette Bild vor Augen halten. Der Äther (ich verwende bewusst Ätherbegriff) kann mit einem Festkörper verglichen werden, wobei Gitterplätze durch "virtuelle" Elektron-Positron-Paare (Photonen) besetzt sind.
Grerichtete Welle entsteht auf gleiche Weise wie optische Phononenwelle im Festkörper, wobei statt unterschiedlich geladener Ionen, das Elektron und das Positron gegeneinander schwingen (Physik-Nobelpreis 2008 :D :D :D ).

mfg

ingeniosus
15.05.08, 14:02
ganz einfach: um u.a. den Begriff aktuale Unendlichkeit in der Physik zu
ma-thematisieren! Nur daraus entsteht die Relativität.


Du erklärst den schon rein mathematischen Begriff "RaumZeitQuantum" durch einen weiteren für mich neuen Begriff "aktuale Unendlichkeit".....

Für mich ist da noch kein Sinn dabei!

Unendlichkeit mathematisch zu definieren ist wohl eine Spielerei, aktuale Unendlichkeit ist für mich nur eine Steigerung einer mathematischen Begriffsverkettung .... was soll ich mir darunter physikalisch vorstellen?

Wo ist ein Bezugsobjekt in der Natur? oder milder gefragt: Welches Ziel hat diese Begriffsverkettung?

Man darf auch die SRT nicht der physischen Natur entfremden, sie hat neue Zusammenhänge gezeigt, aber der Mensch ist immer noch derselbe "Lichtwahrnehmer" mit seinen in langer Evolution entwickelten zwei Augen. Wir müssen alle physikalischen Ergebnisse so hinmanipulieren, dass wir sie mit unseren zwei Augen wahrnehmen können. Sonst kommen wir zu keinem Verständnis! (Tradierter Humanismus oder Neo-Humanorientierung!)

ingeniosus
15.05.08, 14:21
Du musst das komplette Bild vor Augen halten. Der Äther (ich verwende bewusst Ätherbegriff) kann mit einem Festkörper verglichen werden, wobei Gitterplätze durch "virtuelle" Elektron-Positron-Paare (Photonen) besetzt sind.
Grerichtete Welle entsteht auf gleiche Weise wie optische Phononenwelle im Festkörper, wobei statt unterschiedlich geladener Ionen, das Elektron und das Positron gegeneinander schwingen (Physik-Nobelpreis 2008 :D :D :D ).



Dann wären also 2 verschiedene Photonen vorhanden:

A) ein im Gitternetz schwingendes
B) ein Photon, das sich fortbewegt oder zumindest den Impuls gibt...

Das erinnert mich an die Elektronenbewegung in einem elektrischen Leiter!

Aber ein Gitternetz würde doch die Richtung verändern?

Alles nur so lose Gedanken zunächst.....

mbG

Lambert
15.05.08, 16:07
Du erklärst den schon rein mathematischen Begriff "RaumZeitQuantum" durch einen weiteren für mich neuen Begriff "aktuale Unendlichkeit".....

Für mich ist da noch kein Sinn dabei!

Wo ist ein Bezugsobjekt in der Natur?

die Galaxie

Uranor
15.05.08, 16:28
die Galaxie
Ich reduziere das aufdie Tiefe im Gravitationsfeld. Der Betrachtungsweg nach Einstein scheint mir einen ehernen Schlüssel zu beinhalten.

Wie schon angedeutet, die Gedankengänge laufen hier im Kern zusammen. Was wird die klarste, naturnächste Darstellung sein?

Gruß Uranor

Lambert
15.05.08, 17:10
Ich reduziere das aufdie Tiefe im Gravitationsfeld. Der Betrachtungsweg nach Einstein scheint mir einen ehernen Schlüssel zu beinhalten.

Wie schon angedeutet, die Gedankengänge laufen hier im Kern zusammen. Was wird die klarste, naturnächste Darstellung sein?

Gruß Uranor

die komplexe Bezugsformel Lichtgeschwindigkeit in m/Sek-Volumen_Raum_Zeit_Quantum in m³.

Gruß Lambert

Uranor
15.05.08, 18:31
die komplexe Bezugsformel Lichtgeschwindigkeit in m/Sek-Volumen_Raum_Zeit_Quantum in m³.

Gruß Lambert
Hört sich schon vom Ansatz her vielversprechend an. Nur die m³ (Kuhbückmeter :p ) kann ich nicht unterbringen. Ich sehe derzeit tatsächlich nur eine Längenangabe für die Tiefe, kein Volumen.


Volumen würde etwas ein- oder ausschließen. Ort, Länge und Vektor scheinen mir wichtige Eckwerte zu sein. Raum hat für mich keine Bedeutung. Alles (wirklich alles) ergibt sich aus Bezügen, ausgehend vom tiefsten, noch nicht physikalischen, unschafen Quantenvakuum.

- Jo, das Bild wollte sich in vielen Jahren aufbauen. Ich interessier mich für allles, was zur Überprüfung und zur Ausmist-Pruffolge geeignet scheint. Ich habe kein Weltbild. Es formt sich über das Leben verteilt. -- Der frühe Scheinpreis: In der Jugens kam ich mir durchaus als ohne Meinung vor. Flattern war logo noch möglich. Mit der Zeit kamen mehr Kenntnisse, Erfahrungen, Ausprägungen. Ich behalte bis heut nicht unkritisch in der Hand, was da wie zufällig reinflattern mag. Man ist ja nicht vom tümmlichen Zufall geprägt. ;)


Gruß Uranor

Lambert
15.05.08, 19:53
Hört sich schon vom Ansatz her vielversprechend an. Nur die m³ (Kuhbückmeter :p ) kann ich nicht unterbringen. Ich sehe derzeit tatsächlich nur eine Längenangabe für die Tiefe, kein Volumen.


Volumen würde etwas ein- oder ausschließen. Ort, Länge und Vektor scheinen mir wichtige Eckwerte zu sein. Raum hat für mich keine Bedeutung. Alles (wirklich alles) ergibt sich aus Bezügen, ausgehend vom tiefsten, noch nicht physikalischen, unschafen Quantenvakuum.

- Jo, das Bild wollte sich in vielen Jahren aufbauen. Ich interessier mich für allles, was zur Überprüfung und zur Ausmist-Pruffolge geeignet scheint. Ich habe kein Weltbild. Es formt sich über das Leben verteilt. -- Der frühe Scheinpreis: In der Jugens kam ich mir durchaus als ohne Meinung vor. Flattern war logo noch möglich. Mit der Zeit kamen mehr Kenntnisse, Erfahrungen, Ausprägungen. Ich behalte bis heut nicht unkritisch in der Hand, was da wie zufällig reinflattern mag. Man ist ja nicht vom tümmlichen Zufall geprägt. ;)


Gruß Uranor

Salve Uranor,

mathematisch sind m, m² und m³ zumindest für den hier angestrebten Zweck nicht-unterschieden.

Tja, wie kommt man weiter? Ich habe keine Ahnung. Es sind verschiedene Expertisen benötigt, um etwas Gescheites auszuarbeiten.

Gruß,
Lambert

Uranor
16.05.08, 00:35
Salve Uranor,

mathematisch sind m, m² und m³ zumindest für den hier angestrebten Zweck nicht-unterschieden.

Tja, wie kommt man weiter? Ich habe keine Ahnung. Es sind verschiedene Expertisen benötigt, um etwas Gescheites auszuarbeiten.

Gruß,
Lambert
moin Lambert,

wir können immerhin üben, schon gesichertes zusammenzulesen, es zu verarbeiten, etwa Schlagworte als Fragen reifen zu lassen. Du kenst sicher @Joachims Werk? Er tut auch nichts anderes als zusamenzutragen und auszuwerten. Da steht halt Praxis hintendran.

Und ich staune immwe wieder, dass ich explizit nichts brauche, das irgendwie erfunden und in Halbheit erknottelt ist. Das solide aufarbeiten und im Zusammenhang zu erkunden...



Jau, klar kannst du die Raumzeit betrachten. Sabei fällt halt nur immer wieder auf, dass Polyäder nix anderes sind als Bezugssituationen. Den Minischritt habe ich offenbar schon. Deswegen werde ich Raumquantisierung auch befremdlich, als unverstegbar registrieren.

In der (noch nicht) Wirklichkeit wird es doch erst mal um ein unscharfes Potential gehen. Das ist vektorbezogen nicht nachweisbar energiepotenter als andere Potenz auf einem unscharfen Vektor. Es nähert sich mit Ausgleichsziel unscharf einander an, nur um vielleicht jetzt gemeinsam potenter zu sein oder solidere Potentialität zu registrieren.

Das sind freie Ersatzbilder. Indes, wie wird die ewig unumgängliche Dynamik in der Natur realisiert? Es wird Vorstellungen benötigen, damit vielleicht Untersuchungsmethoden erarbeitet werden können.

Am tiefsten kann bisher mit dem LHC vorgedrungen werden. Dringt man noch weiter in die UHF vor, werden die Werkzeuge für feineres zu energiereich. Das zu untersuchende würde im zu großen Umfang verwischt,

Wir sehen ja hier, es ist offenbar nicht viel los. Dort, wo bei hinreichend langer Eigenzeit offenbar die Virtualitäten so dicht sind, dass sie gerade in der Raumflachheit Expansion verursachen, sind keine Massen, die etwas beobachten könnten.


Und ich denke grad an Halton Arp. Nein, nicht wie Arp und @uwebus. keine Ermüdung, sondern in der euklidischen Flachheit liegt die wahre Kraft.

Da draußen wird Unbestimtheit in Energie, in Paarbildungen, in Floktuationen, in einfache Materieformen umgesetzt.

Also wird das "Weltall" eines Tages nicht zerrissen werden. Auf den vorhandenen Strukturen werden unter Ausbreitung und lokaler Verdünnung ähnliches wie Vorstaubwölken realisiert. Wir werden dort hin gedrückt, die dunklen Energien.

Das ist alles kein Hexenwerk. Kapazitäten sollen die Gedanken nachvollziehen und sie auf hohem Kybernetik-Niveau verbindend und verstehend betrachten. Und genau das tun sie sicher bereits längst, ohne dass sie ein kleiner uranor draif hinweisen müsste. Die brauchen mich nicht. Ich denke, sie sind inzwischen selber groß. :p


Gruß Uranor

Lambert
17.05.08, 08:33
Ich reduziere das auf die Tiefe im Gravitationsfeld.

Gruß Uranor

moin :) Uranor,

Hatte ich Dir schon gesagt? Diese Deine Aussage ist nach sqt geradezu perfekt.

Bin diese Tage ziemlich unterwegs. Deswegen oft obskur. :o

Gruß,
Lambert

Uranor
17.05.08, 11:55
Hey, dann fehlen vor allem noch die Formel-Spezialisten.

Es wird dann interessant werden, den Beschreibungstext zu lesen. Alles relevante scheint bekannt zu sein. Doch in der Forschung wird nicht vorangeschritten. Man rätselt noch grundsätzlich. Das hängt mir zu hoch. Da wirken doch überal Asse. Die Thematiken haben auf Überprüfbarkeitsebene geknackt zu sein. Es scheint kein Problem zu geben.

Gruß Uranor

criptically
17.05.08, 12:29
Dann wären also 2 verschiedene Photonen vorhanden:

A) ein im Gitternetz schwingendes
B) ein Photon, das sich fortbewegt oder zumindest den Impuls gibt...

Das erinnert mich an die Elektronenbewegung in einem elektrischen Leiter!

Aber ein Gitternetz würde doch die Richtung verändern?

Alles nur so lose Gedanken zunächst.....

mbG

Warum würde ein Gitternetz die Richtung verändern? Das wäre nur dann der Fall, wenn das Gitter anisotrop wäre (siehe z.B. Doppelbrechung).

mfg

Lambert
17.05.08, 22:16
Hey, dann fehlen vor allem noch die Formel-Spezialisten.

Es wird dann interessant werden, den Beschreibungstext zu lesen. Alles relevante scheint bekannt zu sein. Doch in der Forschung wird nicht vorangeschritten. Man rätselt noch grundsätzlich. Das hängt mir zu hoch. Da wirken doch überal Asse. Die Thematiken haben auf Überprüfbarkeitsebene geknackt zu sein. Es scheint kein Problem zu geben.

Gruß Uranor

so ist es. Es fehlen nur die Formelspezialisten. Das Modell ist geklärt.

Es gibt nur keine Probleme, da sie gern unter den Tisch gefegt werden. Berufsblindheit allentwegs. Furcht vor Grenzüberschreitung.

Der Raum wird nicht ernst genommen. Nur der Spielraum. Ludent.

Gute Nacht,
Lambert

Lambert
21.05.08, 14:44
Was ist Licht?
QFT: Felder sind Operatoren die Teilchen aus dem Vakuum erzeugen und in dieses vernichten. Teilchen sind Anregungszustände des Vakuums. Die Teilchenzahl muss nicht erhalten bleiben!



Hallo Querkopf,

"Die Teilchenzahl muss nicht erhalten bleiben!"


Diese Aussage widerspricht meinem Verständnis der "Naturtheorie".
Gibt es für die Aussage experimentelle Beweise? Nämlich, dass sich die Teilchenzahl (=Zahl der Teilchen) ändert? Für mich gibt es nur Umwandlung.

Gruß,
L

uwebus
22.05.08, 13:06
Lambert,
----
Gibt es für die Aussage experimentelle Beweise? Nämlich, dass sich die Teilchenzahl (=Zahl der Teilchen) ändert? Für mich gibt es nur Umwandlung.
----
Was bezeichnest Du als Teilchen? Wenn das Universum sich aus Raumquanten konstituieren sollte, wie wir beide dies wohl annehmen, dann wäre ein solches Quant das kleinste aller denkbaren Teilchen, alles größere wären Zusammenschlüsse.

Gruß

Lambert
22.05.08, 15:18
Lambert,
----
Gibt es für die Aussage experimentelle Beweise? Nämlich, dass sich die Teilchenzahl (=Zahl der Teilchen) ändert? Für mich gibt es nur Umwandlung.
----
Was bezeichnest Du als Teilchen? Wenn das Universum sich aus Raumquanten konstituieren sollte, wie wir beide dies wohl annehmen, dann wäre ein solches Quant das kleinste aller denkbaren Teilchen, alles größere wären Zusammenschlüsse.

Gruß

ich habe mich bezogen auf eine Aussage von Querkopf, die ich auch zitierte.

Ich persönlich bezeichne Bosonen und Fermionen als "Teilchen". Diese sind in der Tat Zusammenschlüsse von Raumquanten/ Gravitonen/ Superquanten (diese verschiedenen Ausdrücke sind Synonyme, die jeweils andere Aspekte vom gleichen Ding beleuchten). Der Prozess des Zusammenschlüsses findet statt während der Rauminflation. Nach Abschluss dieses Prozesses gibt es nur noch Umwandlung und nicht, wie Querkopf m.E. schreibt, willkürliche Annilierung und Entstehung. Das ist meine niedrige Auffassung.

Gruß,
Lambert

Uranor
22.05.08, 18:56
ich habe mich bezogen auf eine Aussage von Querkopf, die ich auch zitierte.

Ich persönlich bezeichne Bosonen und Fermionen als "Teilchen". Diese sind in der Tat Zusammenschlüsse von Raumquanten/ Gravitonen/ Superquanten (diese verschiedenen Ausdrücke sind Synonyme, die jeweils andere Aspekte vom gleichen Ding beleuchten). Der Prozess des Zusammenschlüsses findet statt während der Rauminflation. Nach Abschluss dieses Prozesses gibt es nur noch Umwandlung und nicht, wie Querkopf m.E. schreibt, willkürliche Annilierung und Entstehung. Das ist meine niedrige Auffassung.

Gruß,
Lambert
Sowas wie virtuelle Realisierung aus zunächst unscharfen Energiesituationen in der Paarerzeugung? Annihilierung kann je nach beteiligter Energie auch wieder Paarbildung ergeben, etwa e+ | e- . Denkst du an solche Zusammenhänge?


Gruß Uranor

Lambert
22.05.08, 19:55
Sowas wie virtuelle Realisierung aus zunächst unscharfen Energiesituationen in der Paarerzeugung? Annihilierung kann je nach beteiligter Energie auch wieder Paarbildung ergeben, etwa e+ | e- . Denkst du an solche Zusammenhänge?


Gruß Uranor

Hallo Uranor,

Nicht ganz. Die Raumquanten/Gravitonen bzw. Superquanten sind virtuell und unter dem Energielevel. Sobald die Rede ist von Energie, ist der Energieurknall, mit dem die Rauminflation abgeschlossen wird, bereits passé.

Gruß,
L

Uranor
22.05.08, 20:57
Hallo Uranor,

Nicht ganz. Die Raumquanten/Gravitonen bzw. Superquanten sind virtuell und unter dem Energielevel. Sobald die Rede ist von Energie, ist der Energieurknall, mit dem die Rauminflation abgeschlossen wird, bereits passé.

Gruß,
L
Äh ja, ist klar. Wir können von Energie nur sprechen, wenn sie physikalisch messbar ist. Sie war unscharf virtuell und wird nun durch konkretisierte Objekte realisiert. h ist die Grenze.

Die Blickperspektiven sind irgendwo anders. Der unscharfe Putentialbereich ist für mich nicht kennbar. Falls sich eine Fluktuation ergibt, kann sie für mich real als Objekt kennbar sein. Im virtuellen Topf sehe ich nichts, nicht mal Quanten. Unter h haben die nun mal keine Möglichkeit, an der Physik teilzunehmen.

Ja, drum kenne ich auch außer Potentialtopf und Physik. Superquanten sagen mir noch lange nix. Sind sie physikalisch oder nicht?

Gruß Uranor

Lambert
22.05.08, 22:17
Äh ja, ist klar. Wir können von Energie nur sprechen, wenn sie physikalisch messbar ist. Sie war unscharf virtuell und wird nun durch konkretisierte Objekte realisiert. h ist die Grenze.

Die Blickperspektiven sind irgendwo anders. Der unscharfe Putentialbereich ist für mich nicht kennbar. Falls sich eine Fluktuation ergibt, kann sie für mich real als Objekt kennbar sein. Im virtuellen Topf sehe ich nichts, nicht mal Quanten. Unter h haben die nun mal keine Möglichkeit, an der Physik teilzunehmen.

Ja, drum kenne ich auch außer Potentialtopf und Physik. Superquanten sagen mir noch lange nix. Sind sie physikalisch oder nicht?

Gruß Uranor

gute Frage.

Superquantum sind nur physikalisch im Verbund. Nur im Verbund gibt es die Energieerhaltung und Symmetrie. Der Verbund entsteht in verschiedenen Verbänden schlagartig, wenn auch in bestimmter fester Reihefolge.

"Allein" sind Superquanten "nichts". Höchstens Volumen oder erregter Äther.
Ohne Erregung sind sie noch weniger: nichts-nichts.

Gruß,
L

PS. bin wieder einige Tage geldsammelnd vorlesend wegfahrend. Bis denne.

criptically
22.05.08, 22:44
gute Frage.

Superquantum sind nur physikalisch im Verbund. Nur im Verbund gibt es die Energieerhaltung und Symmetrie. Der Verbund entsteht in verschiedenen Verbänden schlagartig, wenn auch in bestimmter fester Reihefolge.

"Allein" sind Superquanten "nichts". Höchstens Volumen oder erregter Äther.
Ohne Erregung sind sie noch weniger: nichts-nichts.

Gruß,
L

PS. bin wieder einige Tage geldsammelnd vorlesend wegfahrend. Bis denne.

:confused: :confused: :confused:

mfg

Uranor
22.05.08, 23:40
:confused: :confused: :confused:

mfg
*stimmung*, ich bin auch neugierig geworden. :cool: || Verzweifelnis dank nixo versteho? Geduld.

Gruß Uranor

ingeniosus
26.05.08, 11:35
:confused: :confused: :confused:

mfg

Criptically!


Bitte übersetzte uns Deinen beigefügten "französischen" Satz:

La résultat des mesures montre que, dans l'espace ambiant, la lumière se propage avec une vitesse, indépendante du mouvement d'ensemble de la source lumineuse et du système optique. Cette propriété de l'espace caractérise expérimentalement l'éther lumineux.

M. G. Sagnac (1913)

Danke für Deine Codefreigabe! Es lebe der Anti-Cryptizismus!

ingeniosus
26.05.08, 12:23
Ich persönlich bezeichne Bosonen und Fermionen als "Teilchen". Diese sind in der Tat Zusammenschlüsse von Raumquanten/ Gravitonen/ Superquanten (diese verschiedenen Ausdrücke sind Synonyme, die jeweils andere Aspekte vom gleichen Ding beleuchten).

Teilchen....

Das ist auch meiner Meinung nach das beste deutsche Wort dafür. Ich verwende eben auch "Energon" (eigene Bezeichnung) in diesem Sinne.

Das Wort "Quantum" versuche ich zu entmystifizieren. Plank,Einstein haben ja ursprünglich eine Lichtenergie-Teilmenge damit gemeint.

Heute wird dieses Wort oft mehrdeutig und unsauber verwendet. Exakt müsste man eben "Teilmenge" sagen, aber es kann eben eine Teilmenge einer beliebiger Gesamtmenge sein.

Es verlangt neben der "Teilmengenbildungsbezeichnung" auch die saubere Mengenbezeichnung, wie etwa Energie, Kraft, Masse, oder eben auch die im Forum verwendete "Melone" - ein "Melonen-Quantum" ist eben eine jeweils zu bestimmende Menge einer Melone....

Querkopf
27.05.08, 13:26
Ich denke, so etwas gibt es schon. Denk dir das untenstehende Feynmandiagramm für die Delbrück-Streuung in der Senkrechten halbiert sodass das e- e+ Paar reell wird und ausläuft.
Ich wüsste nichts, was kinematisch gegen so einen Prozess spricht, wenn denn die beiden Photonen die entsprechende Schwellenergie zur Paarerzeugung überschreiten.
Gruß,
Uli
Du hast schon recht, ich war etwas zu sehr auf den effektiven Fall niedriger Energien fixiert. Wobei das beste Argument wäre das es ja Paarvernichtung gibt und bei genügend Energie der umgekehrte Prozess immer möglich sein sollte.

Die folgende Graphik zeigt Feynman-Diagramm von Bildung und Zerfall eines Photons!
Es zeigt die Streuung eines Elektron - Positron Paares in erster Ordnung Störungstheorie (Coulombwechselwirkung). Das Photon ist Einwechselwirkungsteilchen, dient also der Mathematischen Beschreibung des Prozesses.

Feynmandiagramme sind graphische Darstellungen der störungstheoretischen Behandlung von QFTheorien (nicht bloss relativistischer, sondern es gibt sie z.B. auch in der Statistischen Physik die im Gleichgewichtsfall auch eine QFT ist). Sie folgen aus dem Mathematischen Apparat dieser Theorien (es ist sehr viel Mathematik notwendig um zu beweisen, dass ich das machen kann: Stichwort wäre z.B. Wick’s Theorem). Eine Interpretation außerhalb des Apparates der QFT ist daher sinnlos.

Ich kann nicht erst 150 Seiten aus einem Buch nehmen, dann mit diesem Wissen einen Prozess über virtuelle Photonen beschreiben und dann sagen: Ätschibätsch, das interessiert mich alles nicht mehr, ich sage jetzt, das meine Photonen real sind und sowieso was ganz anderes als das was ich benutzt habe um sie herzuleiten. Das geht nicht. Wer A sagt muss auch B sagen. Wenn ich A sage das am Fundament z.B. der QED etwas falsch ist, kann ich bei B keine Feynman Diagramme aus der QED benutzen, denn dann bin ich im Widerspruch zu A.





Hallo Querkopf,
"Die Teilchenzahl muss nicht erhalten bleiben!"
Diese Aussage widerspricht meinem Verständnis der "Naturtheorie".
Gibt es für die Aussage experimentelle Beweise? Nämlich, dass sich die Teilchenzahl (=Zahl der Teilchen) ändert? Für mich gibt es nur Umwandlung.
Es gibt viele Theorien in denen die Teilchenzahl nicht erhalten bleib. Typischerweise z.B. in der Thermodynamik oder statistischen Physik.
Wenn ich ein Gasgemisch auf Molekülebene betrachte (also eine effektive Theorie) kann ich neben Energieaustausch oder Volumenänderung auch chemische Prozesse haben, die die Zahl meiner Moleküle ändern. Natürlich habe ich da den Fall, das ich wenn ich all die Elektronen und Quarks der Moleküle zahlen würde wieder Teilchenzahlerhaltung hätte.
Ähnliches habe ich bei anderen effektiven Theorien, z.B. in einem Supraleiter ist die Zahl der Cooperpaare keine Erhaltungsgröße (die der Elektronen allerdings schon), ich muss nur ein bisschen Wärme zuführen.

In der QFT aber auch in der Vielteilchenquantenmechanik beschreibe Wechselwirkungen über Teilchenerzeugung und Vernichtung. Das ist mein mathematischer Apparat. Ich kann mir in diesem Apparat einfach einen Operator definieren, der meine Teilchenzahl misst. Dann kann ich untersuchen ob diese erhalten bleibt (das kann ich alles mathematisch zeigen). Das ist für viele Systeme nicht der Fall.
Man darf sich unter einem Teilchen halt kein kleines Kügelchen vorstellen, sondern es ist in diesem Theoriegebäude ein Anregungszustand vermittelt durch einen Erzeugungsoperator.
Dieses Theoriegebäude beschreibt die Experimente. In der klassischen Mechanik kann ich z.B. die Streuung zweier Massepunkte aneinander beschreiben. Diese ändern ihre Energie und ihren Impuls. Wenn ich aber Beschleunigerexperimente mache und zwei Teilchen aufeinander schieße, dann habe ich bei höheren Energien nicht mehr nur eine Streunung, sondern meine Teilchen können in einem Feuerwerk in einen ganzen Haufen völlig anderer Teilchen mit anderen Eigenschaften und anderen Wechselwirkungen. Das bedeutet ich brauche eine Theorie die mir aus 2 Teilchen 25 machen kann und dabei auch noch sagt, wie lange die Lebensdauer ist, welche Energien die Teilchen haben und in welchem Winkel sie auseinanderfliegen. Das ist das was QFT leisten kann, Systeme mit vielen Freiheitsgraden und veränderlicher Teilchenzahl (und Art) qualitativ aber eben auch quantitativ zu beschreiben.

Ich treffe aber immer nur Aussagen innerhalb eines Theoriegebäudes. Ich kann z.B. die Starke Kernkraft auf der Ebene von Nukleonen als QFT beschreiben. Meine Nukleonen haben natürlich einen Substruktur, aber diese ist auf bestimmten Energieskalen nicht relevant (durch fleißiges Quak zählen ist also möglicherweise die ein oder andere Verletzung der Teilchenzahl zu verhindern).

Deine Idee ist ungefähr die, das es ganz kleine Subbausteine gibt und wenn ich Teilchen vernichte und neue erzeuge werden nur die Grundbausteine neu zusammengesetzt. Das ist die alte griechische Idee de unteilbaren Atomos, die sich aber in der Entwicklung der modernen Physik als nicht sehr nützlich herausgestellt hat.

Es gibt eine schone Schilderung von Feynman wie er vergeblich versucht seinem Vater Emissions – Absorptions Prozesse im Wasserstoffatom zu erklären. Nicht nur in er klassischen Theorie, sondern auch in der QM ist das äußerst mysteriös.
Man muss sich das mal vorstellen. Ich habe ein Elektron mit einer gewissen Energie irgendwo und überall um mein Proton. Nun kommt plötzlich ein Photon daher und trifft auf mein Elektron und das befindet sich plötzlich (tatsächlich Oszilliert es erst ein bisschen zwischen zwei Niveaus) auf einem Energieniveau. Frage: Wo ist mein Photon geblieben? Schlimmer noch. Nach geraumer Zeit Emittiert das Elektron wieder ein Photon und fällt auf das ursprüngliche Niveau zurück. Wo kommt plötzlich ein Photon her? Hat das Elektron einen Photonensack in dem es die Photonen einsammelt und nach geraumer Zeit wieder freilässt? Es gibt keine Mechanismus in der klassischen Physik oder der QM der dieses verschwinden und wiederauftauchen des Photons erklärt.
Man kann sich folgendes Paradoxes Experiment vorstellen.
Man nehme ein Positronium, also ein Wasserstoffähnliches Atom aus einem Elektron und einem Positron. Diese Atom hat eine begrenzte Lebensdauer, aber diese ist lang genug um Spektroskopie an ihm zu betreiben. Es gibt Universitäten, in denen Studenten das im Fortgeschrittenen Praktikum machen, es handelt sich also um keinen ganz Exotischen Versuchsaufbau.
Nun regen wir das Atom folgendermaßen an (wenn man genügend versuche macht, wird es irgendwann klappen). Wir regen es durch Absorption von Photonen zunächst in den ersten, dann in den zweiten, dann in den dritten und am ende in den vierten angeregten Zustand. Da stecken jetzt vier Photonen drin. Dann warten wir, bis es spontan unter Emission eines Photons in den Grundzustand zurückfällt. Wir haben also nur noch drei Photonen gespeichert. Das Spielchen machen wir so lange, bis unser Positronium in einem Blitz zu zwei Photonen zerstrahlt. Frage: Wo sind unsere gespeicherten Photonen geblieben????? Sind Photonen aus Photonen aufgebaut?
Das kann nicht sein, den Photonen scheinen praktisch nicht miteinander Wechselzuwirken. Licht unterschiedlicher Frequenz lässt sich durch Filter immer wieder trennen und üblicherweise gilt das Superpositionsprinzip.

Wenn ich jetzt wie jemand hier annehme, meine Photonen wären Elektron -Positron Paare, ist mein Problem noch gravierender. Mein Photon kann plötzlich aus einem Kollektiv von beliebig vielen Elektronen und Positronen bestehen, die zu allem Unglück geladen und recht massiv sind.
Der Mechanismus der QED sieht das Erzeugen und Vernichten von Teilchen als wesentlicher Teil seines Mathematischen Apparates vor und umgeht damit (unbeabsichtigt und völlig gratis) solcherlei Probleme.

Lambert
28.05.08, 22:03
Der Mechanismus der QED sieht das Erzeugen und Vernichten von Teilchen als wesentlicher Teil seines Mathematischen Apparates vor und umgeht damit (unbeabsichtigt und völlig gratis) solcherlei Probleme.

Was ist schon gratis? Inwieferne ist QED dann noch energiestabil? Das ist schon fishy.
Aber QED ist ja fishy, wenn sie auch für alle technischen Zwecke im breiteren Wahrscheinlichkeitsspektrum die Bedingungen erfüllt. Gespenstig (Einstein) wird es erst beim einzelnen Teilchen. Und da versagt die QED naturgemäß. So ist die Aussage der willkürlichen Schöpfung und Vernichtung von Teilchen nach meiner völlig unwichtigen Meinung schlicht fraglich. SQT erlaubt so etwas nicht; geregeltes Wahrscheinlichkeitsverhalten ja, aber keine chaotische Entstehungsprozesse einzelner Teilchen.

L

criptically
28.05.08, 22:12
Criptically!


Bitte übersetzte uns Deinen beigefügten "französischen" Satz:

La résultat des mesures montre que, dans l'espace ambiant, la lumière se propage avec une vitesse, indépendante du mouvement d'ensemble de la source lumineuse et du système optique. Cette propriété de l'espace caractérise expérimentalement l'éther lumineux.

M. G. Sagnac (1913)

Danke für Deine Codefreigabe! Es lebe der Anti-Cryptizismus!

In etwa so:

Die Messungen zeigen, dass sich das Licht im umgebenden Raum mit einer Geschwindigkeit verbreitet, die vom Bewegungszustand der Lichtquelle und des optischen Systems unabhängig ist. Diese Eigenschaft des Raums charakterisiert experimentell den Lichtäther.

M. G. Sagnac (1913)

criptically
28.05.08, 22:30
...
Man muss sich das mal vorstellen. Ich habe ein Elektron mit einer gewissen Energie irgendwo und überall um mein Proton. Nun kommt plötzlich ein Photon daher und trifft auf mein Elektron und das befindet sich plötzlich (tatsächlich Oszilliert es erst ein bisschen zwischen zwei Niveaus) auf einem Energieniveau. Frage: Wo ist mein Photon geblieben? Schlimmer noch. Nach geraumer Zeit Emittiert das Elektron wieder ein Photon und fällt auf das ursprüngliche Niveau zurück. Wo kommt plötzlich ein Photon her? Hat das Elektron einen Photonensack in dem es die Photonen einsammelt und nach geraumer Zeit wieder freilässt? Es gibt keine Mechanismus in der klassischen Physik oder der QM der dieses verschwinden und wiederauftauchen des Photons erklärt.

Das ist doch lächerlich. Die Photonen sind Feldquanten des em.-Felds, also wenn ein Photon absorbiert wird, ändert sich das intrinsische em-Feld des Atoms entsprechend der em-Feldstärke des Photons. Wie du siehst, alles ist klassisch erklärbar. :D

...

Wenn ich jetzt wie jemand hier annehme, meine Photonen wären Elektron -Positron Paare, ist mein Problem noch gravierender. Mein Photon kann plötzlich aus einem Kollektiv von beliebig vielen Elektronen und Positronen bestehen, die zu allem Unglück geladen und recht massiv sind.
Der Mechanismus der QED sieht das Erzeugen und Vernichten von Teilchen als wesentlicher Teil seines Mathematischen Apparates vor und umgeht damit (unbeabsichtigt und völlig gratis) solcherlei Probleme.

Es gibt kein Problem weil diese "virtuellen" Elektron-Positron-Paare masselos sind und sie befinden sich überall im Universum. Also es muss gar nichts erzeugt werden.

mfg

ingeniosus
28.05.08, 23:36
In etwa so:

Die Messungen zeigen, dass sich das Licht im umgebenden Raum mit einer Geschwindigkeit verbreitet, die vom Bewegungszustand der Lichtquelle und des optischen Systems unabhängig ist. Diese Eigenschaft des Raums charakterisiert experimentell den Lichtäther.

M. G. Sagnac (1913)

Danke für diese Übersetzung:

Das ist ja ebenfalls eine bestätigte Lichtgeschwindigkeit.

Offen bleibt natürlich die Bewegungsrichtung und die Energie, die ja gequantelt ist.

Das Wort "Lichtäther" ist wie eine Erlösung, weil damit "Äther" als einzelnes Wort eindeutig gestorben bleibt. Lichtäther wäre dann exakt der Vakuumeinergieraum mit seinen "Virtuellen Teilchen".

Uranor
28.05.08, 23:36
Es gibt kein Problem weil diese "virtuellen" Elektron-Positron-Paare masselos sind und sie befinden sich überall im Universum. Also es muss gar nichts erzeugt werden.
Inwifern ist ein Paar masselos, während die einzelnen Teilchen sehr wohl über Masse verfügen? Magie? Nö, oder?


Gruß Uranor

criptically
29.05.08, 07:27
Inwifern ist ein Paar masselos, während die einzelnen Teilchen sehr wohl über Masse verfügen? Magie? Nö, oder?


Gruß Uranor

Magie? Nö!

Die Teilchen verhalten sich "massenenergetisch" wie Nukleonen in einem Atomkern - Je näher sie zueinander angezogen werden, desto mehr Energie verlieren sie (durch Ausstrahlung). Bei diesem Vorgang spricht man in der Kernphysik von Massendefekt.
Da aber die gesamte Masse des Elektrons und des Positrons vom elektromagnetischen Feld herrührt, verschwindet ihre Masse bei hinreichen kleinem Abstand vollständig.
Sobald sie anfangen zu schwingen wird auch der Abstand vergrößert, wodurch sie wieder "massiv" werden.

So einfach ist das! :D

mfg

PS: Ich schlage mich für den diesjährigen Physik-Nobelpreis vor! :D :D :D

criptically
29.05.08, 07:38
Danke für diese Übersetzung:

Das ist ja ebenfalls eine bestätigte Lichtgeschwindigkeit.

Offen bleibt natürlich die Bewegungsrichtung und die Energie, die ja gequantelt ist.

Das Wort "Lichtäther" ist wie eine Erlösung, weil damit "Äther" als einzelnes Wort eindeutig gestorben bleibt. Lichtäther wäre dann exakt der Vakuumeinergieraum mit seinen "Virtuellen Teilchen".

Was Energie betrifft kann man sagen, dass sie nur in gebundenen Zuständen gequantelt ist (genauso wie eine gespannte Gitarrensaite) .
Aus der Beziehung E=hf folgt, dass die Energie jeden Wert annehmen kann, weil f ebenfalls beliebeig groß (bis zu einem f_max) werden kann.

mfg

Lambert
29.05.08, 09:42
In etwa so:

Diese Eigenschaft des Raums charakterisiert experimentell den Lichtäther.

M. G. Sagnac (1913)

es geht um die Beschaffenheit des Raumes, um nichts anderes. Vergiß "Energie". Sie kommt nach dem Raum in "zweiter" Stufe, wie übrigens auch die SRT zeigt.

L

Uranor
29.05.08, 13:27
PS: Ich schlage mich für den diesjährigen Physik-Nobelpreis vor! :D :D :D
Immerhin der Erste und einzige Treiber, der im alten Westen die hohe Auszeichnung überhaupt verdient gehabt hätte. ;)

:rolleyes:


Und weitermachen mit Atom, mit dem Verständnis bei Annihilation, hier Wolkenteilchen und so...

:D

mfg

Uranor
29.05.08, 13:33
Was Energie betrifft kann man sagen, dass sie nur in gebundenen Zuständen gequantelt ist (genauso wie eine gespannte Gitarrensaite) .
Und wenn sie gequantelt ist, ist die konkretisiert, hat nix mit dem Vakkum als Hintergrundfeld zu tun.

Gruß Uranor

ingeniosus
29.05.08, 13:57
Magie? Nö!

Da aber die gesamte Masse des Elektrons und des Positrons vom elektromagnetischen Feld herrührt, verschwindet ihre Masse bei hinreichen kleinem Abstand vollständig.


Also ich würde Dir den Nobelpreis wünschen, aber bis dahin ist es noch ein weiter Weg! Er ist ja für viele ein Traumziel!

Also böse blickender Criptically (Geheimnisvoller, Unbekannter ) !

Wie kann Masse von einem Elektromagnetischen Feld herrühren?

Da ist der Hund begraben!

Das mit dem Massendefekt ist m.E. denkbar (.... Starke Wechselwirkung).

Es gibt Analogien zwischen Gravitation und EM, aber so einfach kann ich das noch nicht akzeptieren.

Lambert
29.05.08, 14:11
Also ich würde Dir den Nobelpreis wünschen, aber bis dahin ist es noch ein weiter Weg! Er ist ja für viele ein Traumziel!



Das Nachstreben eines Preises ist die schlechteste Motivation für Grundlagenforschung. Verratet es, wo Du her kommst?

Das reine Hinschreiben des Wortes Energon, Monade (oder ähnlich) heißt nicht, dass man eine Theorie hat.

L.

Uranor
29.05.08, 14:12
es geht um die Beschaffenheit des Raumes, um nichts anderes. Vergiß "Energie". Sie kommt nach dem Raum in "zweiter" Stufe, wie übrigens auch die SRT zeigt.

L
Hmmm. Hier müsstest du wohl konkreter werden? Sonst verführt das zu der Annahme, Raum sei ein leerer Karton.

Also ich mag ja Energie keineswegs nur den Konkretisierungen, den physikalischen Teilchen zuschreiben. Wenn wir etwas als Hintergrundfeld erkannt haben, bleibt Heisenberg uneingeschränkt gültig und wird z.B. konkret am Casimir-Effekt bestätigt. Es gibt Bezüge, die aber noch keine Wirkung bedeuten. Wenn die nicht auf der unscharfen Energiebasis beruhen, ist Energie nicht erhalten. Sie wäre bei der Paarbildung einfach da. Die Natur druckt sich ihr Geld selbst? :D

Umgekehrt, bei der Paarvernichtung ergibt sich reine Energie. Aus ihr konkretisiert das maximale, was möglich ist. Bildbeispiel: Ein Hurrikan-Wirbel wird bei einer Wegänderung nicht zum Orkan sondern speist die folgende Wirbelsituation. Der ernährte Wirbel ist offensichtlich seine Natur. Erst bei weniger Nahrung kann die gesamte Energie nicht mehr beteiligt sein. Und wieder weg vom Bild, die Energie bildet immer kleinere Konkretisierungen bis zum Neutrino, bis zur Prozesswärme. Radiosignale sind nicht möglich?


Dabei fällt (mir) auf, bestehende Konkretisierung kann nicht mehr unscharfes Vakuum werden. Aber genau das muss doch als Forderung bestehen? Paarerzeugung gibt es. Wie erfolg die "Rückzahlung" ans Vakuum? Etwas ganz wesentliches entdecke ich grad als Wissenslücke. Wer mag füllen?


Auf jeden Fall scheint Energie der Grundstoff zu sein. Auf unscharfer Potentialbasis würde er das Vakuum vorphysikalisch strukturieren.


Gruß Uranor

Lambert
29.05.08, 15:41
Hmmm. Hier müsstest du wohl konkreter werden? Sonst verführt das zu der Annahme, Raum sei ein leerer Karton.



Gruß Uranor

Raum und Raumunbestimmtheit sind wesentliche Komponente.
Mathematisch simpelst ausgedruckt in V=Vsinft + Vjcosft

Alles andere folgt später als Kombis jenes Ausdruckes. String und Quant auf virtuellem (sagen wir: auf unmessbarem Level) Level gleichsam bedient. Und das ist gut so.

Gruß,
L

Uranor
29.05.08, 17:04
Raum und Raumunbestimmtheit sind wesentliche Komponente.
Mathematisch simpelst ausgedruckt in V=Vsinft + Vjcosft
Also du bleibst dem Karton teu, zierst ihn einfach nur mit der Unbestimmt-Eigenschaft? Worauf beziehst du die Trigonometrie?

Alles andere folgt später als Kombis jenes Ausdruckes. String und Quant auf virtuellem (sagen wir: auf unmessbarem Level) Level gleichsam bedient. Und das ist gut so.
Setzt du einen raumzeitlichen Start? Anders bekämst du kein "später". Es sollte schlichtweg unmöglich sein, Schuhkarton und Inhalt zeitlich einer Reihenfolge zuzuordnen.


Du siehst, auf deiner Darstellung verbleiben Fragen. Niemand kann vor der Forschung eine Antwort verlangen. Doch ist es nicht wichtig, die Fragen zu spezifizieren? Sollte nicht auch eine kleine, private Theorie ihren Gültigkeitsbereich definieren?

Gruß Uranor

Lambert
29.05.08, 17:21
Also du bleibst dem Karton teu, zierst ihn einfach nur mit der Unbestimmt-Eigenschaft?
1) Worauf beziehst du die Trigonometrie?


2) Setzt du einen raumzeitlichen Start? Anders bekämst du kein "später". Es sollte schlichtweg unmöglich sein, Schuhkarton und Inhalt zeitlich einer Reihenfolge zuzuordnen.




Gruß Uranor

Salve Uranor,

1) das ist die einfachste Quantumformel, bezogen auf einem einzelnen Volumenquantum; gleichzeitig eine Welle, wie Du siehst. Die "Ordnung" ist in dieser Darstellung nicht sichtbar.
2) ja, ohne jeden Zweifel. Raum vor Energie. Das nenne ich die Rauminflation. Sie kommt vór dem Energieknall. Das ist für ein Formelfreak überhaupt nicht schwer zu berechnen. Wo ist der Freak?

Gruß,
Lambert

Uranor
29.05.08, 18:05
Salve Uranor,

1) das ist die einfachste Quantumformel, bezogen auf einem einzelnen Volumenquantum; gleichzeitig eine Welle, wie Du siehst. Die "Ordnung" ist in dieser Darstellung nicht sichtbar.
salve Lambert,

dann war das ein Missverständnis, mea culpa. Ich nahm an, du beziehst dich auf das Vakuum. Raum als solcher hat für mein tristes Verständnis nicht die geringste Bedeutung. Bezüge ergeben sich auf relativen Objektpositionen. Der "Schuhkarton" wäre da sogar im allerweitesten Bildsinne krottenfalsch.

2) ja, ohne jeden Zweifel. Raum vor Energie. Das nenne ich die Rauminflation. Sie kommt vór dem Energieknall. Das ist für ein Formelfreak überhaupt nicht schwer zu berechnen. Wo ist der Freak?
Dann ist das OK. Unsere Ansichten sind absolut verschieden. Pg, das sah anfänglich so aus und hat sich jetzt klatgestellt. Kann man nix machen, ist so.


Gruß Uranor

criptically
29.05.08, 18:51
...

Wie kann Masse von einem Elektromagnetischen Feld herrühren?

Da ist der Hund begraben!

...

Man sollte zuerst verstehen aus was ein em-Feld besteht. Die Antwort ist ganz einfach, das Feld besteht aus Feldquanten die auch Photonen genannt werden. Da die Photonen "massive" Teilchen sind, geben sie dem Feld eine bestimmte Masse.
Diese Masse der Feldquanten ist für Synchrotron-Strahlung verantwortlich, da die Feldquanten wegen Massenträgheit sehr ungern um die Kurven laufen.

mfg

PS: Ich bin nicht so scharf auf Nobelpreis, sonst hätte ich diese "Ergebnisse" nicht hier "veröffentlicht". :D :D

Lambert
29.05.08, 19:53
salve Lambert,

dann war das ein Missverständnis, mea culpa. Ich nahm an, du beziehst dich auf das Vakuum. Raum als solcher hat für mein tristes Verständnis nicht die geringste Bedeutung. Bezüge ergeben sich auf relativen Objektpositionen. Der "Schuhkarton" wäre da sogar im allerweitesten Bildsinne krottenfalsch.


Dann ist das OK. Unsere Ansichten sind absolut verschieden. Pg, das sah anfänglich so aus und hat sich jetzt klatgestellt. Kann man nix machen, ist so.


Gruß Uranor

Ich kenne den Objektbezug der QED, den Du zitierst. Er ist erfolgreich aber gespenstig unvollständig (Einstein's Spuck). Dafür kannst Du aber auch nichts.

Gruß,
Lambert

Uranor
29.05.08, 20:14
Man sollte zuerst verstehen aus was ein em-Feld besteht. Die Antwort ist ganz einfach, das Feld besteht aus Feldquanten die auch Photonen genannt werden. Da die Photonen "massive" Teilchen sind, geben sie dem Feld eine bestimmte Masse.
Diese Masse der Feldquanten ist für Synchrotron-Strahlung verantwortlich, da die Feldquanten wegen Massenträgheit sehr ungern um die Kurven laufen.
Aber wenn das doch so ist... :D :D


PS: Ich bin nicht so scharf auf Nobelpreis, sonst würde ich diese "Ergebnisse" nicht hier "veröffentlicht".
Die mysteriöse Tante beim Würfelzucker zählen: "Ich sehe, ich sehe... hier hat jemand keine Probleme mit seinen Wirklichkeitsbezügen!"

:D :D


mfg

ein Uranor


:D :D

rafiti
29.05.08, 20:24
Diese Masse der Feldquanten ist für Synchrotron-Strahlung verantwortlich, da die Feldquanten wegen Massenträgheit sehr ungern um die Kurven laufen.


Habe ich dir schon die Geschichte mit der Strasse, Kurve, usw.. erzählt?

gruss
rafiti

criptically
29.05.08, 20:25
Und wenn sie gequantelt ist, ist die konkretisiert, hat nix mit dem Vakkum als Hintergrundfeld zu tun.

Gruß Uranor

Wieso denn nicht? :D :D :D

Werden Felder durch nichts übertragen?
Dann bestehen sie aus Korpuskeln! :D

mfg

PS: Also, Nobelpreis würde ich vermutlich nicht unbedingt ablehnen. :D

Uranor
29.05.08, 20:25
Ich kenne den Objektbezug der QED, den Du zitierst. Er ist erfolgreich aber gespenstig unvollständig (Einstein's Spuck). Dafür kannst Du aber auch nichts.

Gruß,
Lambert
In der Tat, das sind so die Kurzdarstellungen, die mir nix sagen. Mitunter kenne ich da völlig andere Zusammenhänge.

Einstein's Spuck aus der unvollständigen QED? Hmmm. Ist das so?

Gruß Uranor

criptically
29.05.08, 20:26
Habe ich dir schon die Geschichte mit der Strasse, Kurve, usw.. erzählt?

gruss
rafiti

Ich kann mich dunkel erinnern. :D :D

mfg

Uranor
29.05.08, 20:39
Wieso denn nicht?

Werden Felder durch nichts übertragen?
Dann bestehen sie aus Korpuskeln! :D

mfg

PS: Also, Nobelpreis würde ich vermutlich nicht unbedingt ablehnen. :D
Die besten Korpuskeln weit und breit verkauft Frau Pömmikke auf dem Wochenmarkt. :D :D :D

mfg

Lambert
29.05.08, 20:46
In der Tat, das sind so die Kurzdarstellungen, die mir nix sagen. Mitunter kenne ich da völlig andere Zusammenhänge.

Einstein's Spuck aus der unvollständigen QED? Hmmm. Ist das so?

Gruß Uranor

sorry: "Einsteins Spuk" heißt das Buch.
Prof. Zeilinger tut Experimente mit dem unverstandenen Phänomen.

Gruß,
Lambert

criptically
29.05.08, 20:54
...

Und weitermachen mit Atom, mit dem Verständnis bei Annihilation, hier Wolkenteilchen und so...

:D

mfg

Also, die Wolke entsteht aus elektronischen Anteilen der virtuellen Paare, die dann bestimmte "Raum-Eigenschwingungen" :D um den Atomkern ausführt. So z.B. beim Wasserstoff, "versinkt" das Elektron ins Vakuum aber dafür steigen viele klitzekleine virtuelle Fluktuationen :D als Ersatz (Tausch) aus dem Vakuum hoch.

mfg

Uranor
29.05.08, 21:35
sorry: "Einsteins Spuk" heißt das Buch.
Prof. Zeilinger tut Experimente mit dem unverstandenen Phänomen.

Gruß,
Lambert
Ah, jetzt ja. Die Korrelation gilt tatsächlich als Magengeschwür. Dabei muss die Theorie noch nicht mal unvollständig sein. Haben wir es tatsächlich mit Nichtlokalität zu tun, kann die durch physikalische Theorien nicht eher beschreibbar sein. Zumindest ist man noch lang nicht so weit.

Dabei wird doch eine weitere Dimension gem. Bellsche Ungleichung ausgeschlossen? Man kann ja nicht mal zubilligen, dass etwa Photonen bei der Paarbildung ihre Quanteneigenschaften erhalten. Ja wann und wie dann? Ich hab mir mal vorgenommen, die Thematik nicht zu verstehen. :p Ich sage nichtlokal, damit ich wenigstens eine gewisse Vorstellung habe kann.


Jo. Vor lauter Trampelriss-Kaperreligionsgequake schrecken sich die förderungspflichtigen Themen eigenständig selbst zurück. Die Mischung hier kannste jederzeit ins Asyl tragen. Bezahlt wird so oder so nix dafür.


Nicht meckern, nur wundern,
ein Uranor

:rolleyes:

Lambert
29.05.08, 22:12
Ah, jetzt ja. Die Korrelation gilt tatsächlich als Magengeschwür. Dabei muss die Theorie noch nicht mal unvollständig sein. Haben wir es tatsächlich mit Nichtlokalität zu tun, kann die durch physikalische Theorien nicht eher beschreibbar sein. Zumindest ist man noch lang nicht so weit.

Dabei wird doch eine weitere Dimension gem. Bellsche Ungleichung ausgeschlossen? Man kann ja nicht mal zubilligen, dass etwa Photonen bei der Paarbildung ihre Quanteneigenschaften erhalten. Ja wann und wie dann? Ich hab mir mal vorgenommen, die Thematik nicht zu verstehen. :p Ich sage nichtlokal, damit ich wenigstens eine gewisse Vorstellung habe kann.


Jo. Vor lauter Trampelriss-Kaperreligionsgequake schrecken sich die förderungspflichtigen Themen eigenständig selbst zurück. Die Mischung hier kannste jederzeit ins Asyl tragen. Bezahlt wird so oder so nix dafür.


Nicht meckern, nur wundern,
ein Uranor

:rolleyes:

ok, Dir steht frei, zu tun und zu denken, wie Du das für gut hältst. :)

ich bemühe den Raum zur Erklärung. :cool:

Gute Nacht, bekomme Besuch und wenig Zeit. Tage.
Bin Dir verpflichtet.
Gruß,
Lambert

rafiti
29.05.08, 23:57
Also, die Wolke entsteht aus elektronischen Anteilen der virtuellen Paare, die dann bestimmte "Raum-Eigenschwingungen" :D um den Atomkern ausführt. So z.B. beim Wasserstoff, "versinkt" das Elektron ins Vakuum aber dafür steigen viele klitzekleine virtuelle Fluktuationen :D als Ersatz (Tausch) aus dem Vakuum hoch.

mfg

Ich glaube, ich muss sie dir doch nochmal erzählen. Es war so: Die Sonne knallte mit > 10c auf mich runter, es waren 50° im Schatten, zuerst erkannte ich vor mir das Melonenfeld, sowas ähnliches wie dein lustiges Quantenfeld da, dann verstand ich es, Ernten-Schwitzen-Verkaufen. Leider kam da die Kurve und naja, den Rest kennst du ja. :D :D :D :D

gruss
rafiti

Uranor
30.05.08, 00:59
ok, Dir steht frei, zu tun und zu denken, wie Du das für gut hältst. :)

ich bemühe den Raum zur Erklärung. :cool:

Gute Nacht, bekomme Besuch und wenig Zeit. Tage.
Bin Dir verpflichtet.
Gruß,
Lambert
Jau, Wildschweinchen Schlau hat vor die Erlärung das Verstehen gesetzt. Wir grillen es dafür und schenken dazu edles Warsteiner im traditionellen Humpen aus.

Wieder mit Gag, man kann nicht genug Wissen sammeln. Ich frag mich halt, wie man drauf kommt, den Raum, wirklich den Raum, aber hoffentlich nicht als Schuhkarton erklären zu wollen, zu können. Ich sehe halt nur Bezüge. Ob die olle RT dran Schuld ist? Hätte ja sein können, das "übliche" Bild passt hier. Aber ich hab jetzt doch mehr den Verdacht, im Vergleich wird nicht mal die LQG passen. Was bleibt?

Oh ja, man vermisst die Anlehnungen, die Herleitung. Ich hab wider Erwarten Interesse gefunden und will mir das jetzt vergegenwärtigen. Was ist es nun genau? Womit starte ich die Betrachtung? Was sehe ich im Detail? Da gibt es doch etwas, das beschrieben wird? Lass das deutlich werden. Du wendest dich ja wohl kaum aus Jux und Dallerei an die Öffentlichkeit? Du müsstest zumindest mal klar sagen, was der Formalist beschreiben soll. Also musst du das Anliegen sprachlich ausdrücken, musst einen möglichst guten Sprachalgor finden. Da bisher niemand erkennt, um was es geht, wird auch niemand beim Prozess unterstützen können.

Klar, jeder denkt, was er für richtig hält. Der Gag ist, man kann nur etwas für richtig halten, das man kennt bzw. vermisst. Vor allem letzteres kann wichtig sein. Wo kein Bedarf erkannt wird, da besteht auch keiner. An solchen Punkten dreh ich rum und frag: Worauf gründest du deinen Bedarf an der Thematik? Worauf gründet dein Interesse nach Erkundung der noch unbekannten Lösungswege? Hier scheint wichtiges zu fehlen.

Dementsprechend ist auch niemand anderen verpflichtet. Im Grunde geht man doch völlig pragmatisch dran. Du behauptest... nichts. Ich denk mir, niemand wird sich die Mühe machen, nichts zu behaupten. Also ist sicherlich was. Darf man dazu näheres erfahren?


So in etwa mag übrigens das Schema sein, wie es @ingeniosus vorschwebt. Etwa Angebot, Mangel, Interesse, Nachfrage, eine Gaudi für sinnvolle Aufmerkdamkeit, wachsende Apetitanregung, also was natürliches. Und ich erinnere mich aus teilweise mieser bis tiefster tiefenlästiger Erfahrung an Drücker- und Nötigungsprinzipien. Immerhin ging es um Religion. Und mit sowas spaße ich nicht.Wenn wir die grenzenlose Barbarei nicht energisch angehen, wird die Steinzeit in den Umgangsformen immer grassieren. Ich hab von Natur aus was dagegen. Also setze ich mich ein, wenn die Situation dafür die passende ist. - Aber gut, mir ging das Nebenthema hier jetzt noch mal durch den Kopf...


Gruß Uranor

Uranor
30.05.08, 01:06
Ich glaube, ich muss sie dir doch nochmal erzählen. Es war so: Die Sonne knallte mit > 10c auf mich runter, es waren 50° im Schatten, zuerst erkannte ich vor mir das Melonenfeld, sowas ähnliches wie dein lustiges Quantenfeld da, dann verstand ich es, Ernten-Schwitzen-Verkaufen. Leider kam da die Kurve und naja, den Rest kennst du ja. :D :D :D :D

gruss
rafiti
Knöllchen wegen überfuttert am Ochsenkarrensteuer,

salve
Uranor

criptically
30.05.08, 18:24
Ich glaube, ich muss sie dir doch nochmal erzählen. Es war so: Die Sonne knallte mit > 10c auf mich runter, es waren 50° im Schatten, zuerst erkannte ich vor mir das Melonenfeld, sowas ähnliches wie dein lustiges Quantenfeld da, dann verstand ich es, Ernten-Schwitzen-Verkaufen. Leider kam da die Kurve und naja, den Rest kennst du ja. ...

gruss
rafiti

Klar, die Melonen sind einfach geradeaus weiter gerollt. Also tangential zum Kreis. :D :D

Genauso ist es mit beschleunigten Ladungen, die Melonen (Feldquanten) rollen geradeaus weiter.

Ich muss sagen, du hast es wirklich begriffen. :D :D

mfg

rafiti
30.05.08, 20:03
Klar, die Melonen sind einfach geradeaus weiter gerollt. Also tangential zum Kreis. :D :D

Genauso ist es mit beschleunigten Ladungen, die Melonen (Feldquanten) rollen geradeaus weiter.

Ich muss sagen, du hast es wirklich begriffen. :D :D

mfg

Gut, dass du es auch begriffen hast. (Keine Ahnung was) Damit kämen wir auch zum Melonenkern der ganzen Melone. Was ist dein Punkt, das Ende vom Lied, aber rede bitte klar und deutlich, damit ich dich auch verstehen kann...

gruss
rafiti

criptically
30.05.08, 21:18
Gut, dass du es auch begriffen hast. (Keine Ahnung was) Damit kämen wir auch zum Melonenkern der ganzen Melone. Was ist dein Punkt, das Ende vom Lied, aber rede bitte klar und deutlich, damit ich dich auch verstehen kann...

gruss
rafiti

Mein Punkt ist: Das Licht ist Welle und die Photonen sind Teilchen.

mfg

Uranor
30.05.08, 21:28
Mein Punkt ist: Das Licht ist Welle und die Photonen sind Teilchen.

mfg
Kann wirklich dolles Lachen manchmal auch bissele der Gesundheit nützen?

:D :D :D


mfg

Marco Polo
30.05.08, 22:54
PS: Ich schlage mich für den diesjährigen Physik-Nobelpreis vor! :D :D :D

Au ja. Ich kann mir niemanden vorstellen, der den mehr verdient hat. :D

criptically
30.05.08, 23:39
Au ja. Ich kann mir niemanden vorstellen, der den mehr verdient hat. :D

Endlich einmal hast Du Recht! :D

mfg

pauli
31.05.08, 12:37
Die smileys können nicht darüber hinwegtäuschen, dass criptically es mit dem Nobelpreis ernst meint

Eyk van Bommel
31.05.08, 13:17
Aber warum sollte criptically einen Nobelpreis für eine Widerlegung von etwas bekommen, dass selbst nicht den Wert hatte einen Nobelpreis zu bekommen?
Für die RT gab es keinen Nobelpreis - für dessen „Widerlegung“ schon? :rolleyes: :D

criptically
31.05.08, 15:02
Aber warum sollte criptically einen Nobelpreis für eine Widerlegung von etwas bekommen, dass selbst nicht den Wert hatte einen Nobelpreis zu bekommen?
Für die RT gab es keinen Nobelpreis - für dessen „Widerlegung“ schon? :rolleyes: :D

Nein, nicht für Widerlegung der RT, sondern für die Formel hier unten die beweist, dass Masse-Energie-Äquivalenz ebenfalls nicht gilt. Mehr noch, die Formel beweist, dass die Photonen eine Masse m besitzen, welche sie wenn sie einen Körper verlassen mit sich mitnehmen.

@Pauli
Natürlich dass ich es ernst meine :D :D .

Wenn Nobelpreise für verschiedenste Fälschungen (z.B. "Entdeckung" von Neutrino) vergeben werden, warum nicht endlich mal auch für eine echte physikalische Leistung. :D :D

mfg

pauli
31.05.08, 15:24
dass die Photonen eine Masse m besitzen, welche sie wenn sie einen Körper verlassen mit sich mitnehmen.
Dann müssten Photonen ja wie Känguruhs einen Beutel für die Masse m haben :confused:

criptically
31.05.08, 16:15
Dann müssten Photonen ja wie Känguruhs einen Beutel für die Masse m haben :confused:

Sie sind ja ein Stück Masse und ein Stück Ladung.

Wenn von einem radioaktiven Material Alpha-Teilchen wegfliegen, nehmen sie auch Masse mit, die genau nach E_kin=mv²/2 mit E_kin im Bezug steht. Also

m=2E_kin/v² .

Das gleiche gilt auch für Photonen

m=2E_kin/c². :D :D :D

mfg

rafiti
31.05.08, 17:03
Sie sind ja ein Stück Masse und ein Stück Ladung.

Also ein wenig was wiegen die schon, aber elektrisch geladen kann man vernachlässigen, merkt niemand weiter, schon gar nicht nachdem sie gegrillt worden sind.
:D :D :D

Wenn von einem radioaktiven Material Alpha-Teilchen wegfliegen, nehmen sie auch Masse mit, die genau nach E_kin=mv²/2 mit E_kin im Bezug steht.

Nö, die Alpha-Männchen sind nicht radioaktiv. :D :D :D

gruss
rafiti

Uranor
31.05.08, 17:12
Nein, nicht für Widerlegung der RT, sondern für die Formel hier unten die beweist, dass Masse-Energie-Äquivalenz ebenfalls nicht gilt. Mehr noch, die Formel beweist, dass die Photonen eine Masse m besitzen, welche sie wenn sie einen Körper verlassen mit sich mitnehmen.
Und darum haben sie auch Spin 1/2 und folgen dem Paulischen Ausschließungsprinzip. Wo ein Licht ist, kann kein anderes sein. Photonen eben.

:rolleyes: :rolleyes: :rolleyes:

mfg

Eyk van Bommel
31.05.08, 20:56
Nein, nicht für Widerlegung der RT, sondern für die Formel hier unten die beweist, dass Masse-Energie-Äquivalenz ebenfalls nicht gilt. Mehr noch, die Formel beweist, dass die Photonen eine Masse m besitzen, welche sie wenn sie einen Körper verlassen mit sich mitnehmen.
Wieso denn? Keiner bezweifelt doch das Photonen Masse „mitnehmen“? Sonnst wäre die Masse von Helium nicht geringer als von 2 H-Atomen (grob gesagt) Sie besitzen nur keine Ruhemasse und den Beweis einer Ruhemasse sehe ich da nicht?
Mir scheint zudem dass du hier nur die Energie/Masse der Photonen quantisierst, dagegen hat ja auch keiner was. Photonen nehmen Energie mit (=Masse) und diese Abgabe ist quantisiert.
Aber wäre deine Energie in dieser Formel nicht ohne Einheiten? :rolleyes:
Ach noch was! Gab es für E=mc^2 einen Nobelpreis? ;)
Naja- vielleicht gibt es 2-mal einen halben. RT und mc^2

criptically
31.05.08, 21:28
Wieso denn? Keiner bezweifelt doch das Photonen Masse „mitnehmen“? Sonnst wäre die Masse von Helium nicht geringer als von 2 H-Atomen (grob gesagt) Sie besitzen nur keine Ruhemasse und den Beweis einer Ruhemasse sehe ich da nicht?
Mir scheint zudem dass du hier nur die Energie/Masse der Photonen quantisierst, dagegen hat ja auch keiner was. Photonen nehmen Energie mit (=Masse) und diese Abgabe ist quantisiert.
Aber wäre deine Energie in dieser Formel nicht ohne Einheiten? :rolleyes:
Ach noch was! Gab es für E=mc^2 einen Nobelpreis? ;)
Naja- vielleicht gibt es 2-mal einen halben. RT und mc^2

Bist du neidisch oder was? :D :D

Energie ist nicht Masse! :D Wenn du eine Feder dehnst, wie viel schwerer wird die? :D

Masse hat Energie z.B kinetische wegen Bewegung, Rotationsenergie auch kinetische wegen Bewegung (Rotation), Schwingungsenergie auch kinetische wegen Schwingungsbewegung, aber die Masse bleibt konstant.

Wenn ein Photon gleichzeitig schwingen und rotieren könnte, dann wäre die "Massengleichung":

m=3E/2c², weil

E=E_kin+E_schwing+E_rot=mc²/2+hf/2+Jω²/2.

Aber ein Photon kann entweder rotieren oder schwingen und deshalb

E=mc²/2+hf/2 oder E=mc²/2+Jω²/2 was aufs gleiche hinauskommt und zwar E=mc².

mfg

criptically
31.05.08, 21:33
Und darum haben sie auch Spin 1/2 und folgen dem Paulischen Ausschließungsprinzip. Wo ein Licht ist, kann kein anderes sein. Photonen eben.

:rolleyes: :rolleyes: :rolleyes:

mfg

Wer oder was hat Spin 1/2 ?

Für Licht gilt Superposition. :D

Positronspin+Elektronspin=1/2+1/2=1 oder 0. :D :D

mfg

Optimist71
31.05.08, 21:42
1/2+1/2=1 oder 0. :D :D

Na, du machst mir Spass ...

Ærbødigst
-- Optimist